PHARMACOLOGY

Ace your homework & exams now with Quizwiz!

A 6-year-old patient was recently diagnosed with Cat I PTB. DOTS Treatment is initiated for this patient. The dosing should be: A. INH 5mg/kg + Rif 10mg/kg + Pyz 25mg/kg + Eth 15mg/kg B. INH 10mg/kg + Rif 15mg/kg + Pyz 35mg/kg + Eth 20mg/kg C. INH 15mg/kg + Rif 10mg/kg + Pyz 35mg/kg + Eth 15mg/kg D. INH 25mg/kg + Rif 10mg/kg + Pyz 25mg/kg + Eth 11mg/kg E. INH 5mg/kg + Rif 20mg/kg + Pyz 15mg/kg + Eth 10mg/kg

1 A. Know the doses of anti-TB drugs for BOTH children and adults. Adults: INH 5mg/kg + Rif 10mg/kg + Pyz 25mg/kg + Eth 15mg/kg Generally just make sure you are familiar with TB! It can come out over and over in different subjects from pharma, micro, pedia, im, patho, even gyne. Source: National TB Control Program Manual of Procedures

A 45 year old male with history of polyuria, polyphagia and weight loss was recently started on antidiabetic medications after workup confirmed DM type II. 2 months later, he notes increasing weight. Which of the following antidiabetic medications is associated with this? A. Chlorpropramide B. Glimepiride C. Exenetide D. A and B E. All of the above

10 D. The first and second generation sulfonylureas are associated with weight gain. Source: Katzung Basic and Clinical Pharmacology 11th ed p748

Which of the following prostaglandin derivatives increases outflow of aqueous humor? A. misoprostol B. alprostadil C. dinoprostone D. epoprostenol E. none of the above

100 E Latanoprost increases outflow of aqueous humor and reduces intraocular pressure in glaucoma. Katzung Pharmacology Examination and Board Review 10th edition p.176

Thru what route is the usual portal of entry in Inorganic Mercury poisoning? A. Ingestion B. Topical application C. Inhalation D. Rectal E. None of the above

101 C. The usual setting of either acute or chronic inorganic mercury poisoning is thru Inhalation of inorganic mercury vapor. Page 94 of Topnotch Handout

Using the Henderson-Hasselbach equation, in a pH above the dissociation constant, a weak acid is present in the _____________ form. A. unprotonated, uncharged B. protonated, charged C. protonated, uncharged D. unprotonated, charged E. unpredictable

102 D. According to the Henderson-Hasselbach equation Weak acid above the pKa is unprotonated and charged. Page 2 of Topnotch Handout

The rectal route is said to have partial avoidance of the first pass effect. Thru which of the following venous drainage of the rectum will NOT bypass the first pass effect: A. Superior rectal vein B. Middle rectal vein C. Inferior rectal vein D. Both B and C E. All of the above

103 A. The superior rectal vein will drain to the Inferior mesenteric vein then to the portal vein, hence having a first pass effect. The middle rectal vein will drain thru the internal iliac vein then to the inferior vena cava The inferior rectal vein will drain via the internal pudendal vein then thru the internal iliac vein then thru the inferior vena cava. Page 3 of Topnotch Handout

15 units of Drug X was given to a patient. If this drug undergoes first-order elimination how many half-lifes will it take for the drug concentration to reach 1.88 units: A. 1 B. 2 C. 3 D. 4 E. 5

104 C. In a drug undergoing first-order elimination concentration decreases by 50% for every half-life. SO if the drug was 15 units at the start, 1st half life- 7.5, 2nd half life 3.75, third half life 1.875. Page 4 of Topnotch Handout

Which of the following statements is/are true regarding the binding affinity of a drug: A. Kd is the concentration required to bind 100% of the drug receptors B. The smaller the Kd the lesser the affinity of the drug for its receptor C. A smaller Kd means that the drug is less potent D. Binding affinity refers to the fraction of receptors bound by a drug plotted against the log of the drug effect E. None of the above

105 E Binding affinity refers to the fraction of receptors bound by a drug plotted against the log of the drug concentration Kd is the concentration required to bind 50% of the receptors The smaller the Kd, the greater the affinity of a drug for its receptor, the more potent it is. Page 4 of Topnotch Handout

A drug is 90% cleared by the liver and 10% by the kidney. The normal dosage of the drug is 500mg/d. If this drug is to be given for a patient with a creatinine clearance of 30ml/min, what should be the corrected dose: A. 15 B. 50 C. 450 D. 465 E. 500

106 D. Normal creatinine clearance for healthy men is 97-137 mL/min. Normal creatinine clearance for healthy women is 88-128 mL/min. In this patient his crea clearance is only 30ml/min. Hence to compute for the corrected dose the formula is: Corrected dose= Average dose (applied only to the part of the dose cleared by the kidney) x patient's crea clearance/ 100ml/min. In this case 500 x 0.10= 50; 30/100= 0.3; ).3 x 50= 15. Corrected dose is 500 x 0.90=450+ 15=465. Page 7 of Topnotch Handout

Which of the following teratogenic drug and effect is correctly paired? A. Sulfonamides: ototoxicity B. Penicillamine: cutis laxa C. Misoprostol: Ebstein's anomaly D. Phenytoin: Mobius sequence E. Lithium: kernicterus

107 B. Sulfonamides : kernicterus; Misoprostol : Mobius sequence; Phenytoin: Fetal hydantoin syndrome; Lithium: Ebstein's anomaly Page 9 of Topnotch Handout

This drug inhibits the transport of Acetylcholine into vesicles for storage by VAT: A. Hemicholinium B. Botulinum C. Neostigmine D. Betanechol E. Vesamicol

108 E Storage of Ach into vesicles is inhibited by vesamicol. Page 11 of Topnotch Handout

A 55 y/o male presented at the ER with complaints of 3 day diarrhea, frequent urination, increased salivation and sweating. You noted his pupils to be constricted. He works in a factory manufacturing pesticides for farms. Which of the following drugs will you give this patient? A. Pyridostigmine B. Edrophonium C. Pilocarpine D. Atropine E. Varenicline

109 D. The patient is experiencing symptoms of organophosphate poisoning. Remember the DUMBBELSS mnemonic! Diarrhea, Urination, Miosis, Bronchospasm, Bradycardia, Excitation (skeletal muscle and CNS), Lacrimation, Sweating and Salivation. The antidote to be given in organophosphate poisoning is Atropine (Cholinergic antagonist, muscarinic) and Pralidoxime (Cholinesterase regenerator) Page 12 of hand out

Which of the following is not associated with pulmonary fibrosis? A. Bleomycin B. Busulfan C. Amiodarone D. Methotrexate E. None of the above

11 E All of the following are associated with pulmonary fibrosis. Source: Topnotch handout on Pharmacology

Your patient is in cardiogenic shock. Which of the following is the drug of choice to increase cardiac contractility without the vasoconstricting effects? A. Norepinephrine B. Dopamine C. Epinephrine D. Phenylephrine E. Dobutamine

110 E In cardiogenic shock the DOC is Dobutamine. It is a selective Beta 1 Agonist hence it has no Alpha 1 vasoconstricting effects. Page 15 of Topnotch Handout

Which of the following beta-blockers are less likely to cause bronchospasm in patients with asthma due to its partial agonist activity? A. Carvedilol B. Labetalol C. Pindolol D. Propanolol E. Nadolol

111 C. The two beta blockers with intrinsic sympathomimetic activity are Pindolol and Acebutolol. Page 16 of Topnotch Handout

A 65 y/o male on Clonidine for a few months for his hypertension abruptly discontinued his medication because he felt he was feeling better. After 2 days, during which he had strong headaches. Three days after withdrawal, the patient had home blood pressure levels of 140-150/95-105 mmHg. Ten days after, the patient went to the emergency room with a symptomatic hypertensive crisis at 220/130 mmHg. What is the drug of choice to treat the rebound hypertension? A. Phentolamine B. Hydrochlorothiazide C. Propanolol D. Hydralazine E. Nicardipine

112 A. For rebound hypertension due to clonidine withdrawal the DOC is Phentolamine. Clonidine is an alpha 2 agonist, sympathetic outflow blocker. Phentolamine: its primary action is vasodilation due to alpha1 blockade, but can also lead to reflex tachycardia because of hypotension and alpha2 inhibition, which increases sympathetic tone. The primary application for phentolamine is for the control of hypertensive emergencies. Unlike the α1 selective blockers, phentolamine also inhibits the α2 receptors, which function predominantly as presynaptic negative feedback for norepinephrine release. By abolishing this negative feedback phentolamine leads to even less regulated norepinephrine release, which results in a more drastic increase in heart rate Page 17 of Topnotch Handout

By which mechanism does Angiotensin converting enzyme inhibitors cause cough? A. By inhibiting ACE and formation of angiotensin II B. By blocking the breakdown of bradykinin C. By facilitating the release of norepinephrine from sympathetic adrenergic nerves D. By inhibiting norepinephrine reuptake E. By blocking angiotensin II stimulation of aldosterone secretion

113 B. ACE inhibitors produce vasodilation by inhibiting the formation of angiotensin II. ACE also breaks down bradykinin (a vasodilator substance). Therefore, ACE inhibitors, by blocking the breakdown of bradykinin, increase bradykinin levels, which can contribute to the vasodilator action of ACE inhibitors. The increase in bradykinin is also believed to be responsible for a troublesome side effect of ACE inhibitors, namely, a dry cough. Angiotensin II also facilitates the release of norepinephrine from sympathetic adrenergic nerves and inhibits norepinephrine reuptake by these nerves. This effect of angiotensin II augments sympathetic activity on the heart and blood vessels. ACE inhibitors promote renal excretion of sodium and water (natriuretic and diuretic effects) by blocking the effects of angiotensin II in the kidney and by blocking angiotensin II stimulation of aldosterone secretion. This reduces blood volume, venous pressure and arterial pressure. Page 19 of Topnotch Handout

This drug belongs to a class of antiarrhythmic drugs which prolongs AP duration, PR interval, QRS duration and QT interval: A. Procainamide B. Lidocaine C. Flecainide D. Propanolol E. Dofetilide

114 A. Under the Singh- Vaughn Williams classification. The class being described are Class 1A anti-arrhythmics. Classified under which is Procainamide Quinidine and Disopyramide. Page 24 of Topnotch Handout

A patient was diagnosed with familial combined hypercholesterolemia, which of the following synergistic drug combination will be most suitable for this patient: A. Niacin + Statin B. Statin + Ezetimibe C. Niacin + Resin D. Statin + Resin E. Fibrate + Resin

115 C. A synergistic combination for patients with familial combined hypercholesterolemia (overproduction of VLDL) are niacin+resin and statin+fibrate. Fibrate and resin combinations are disadvantageous because they confer an increased risk of cholelithiasis. Statin and resin combinations cause impaired statin absorption. Niacin and statin combinations are more for familial hypercholesterolemia where the defect is in the LDL receptors hence increased LDL. Page 29 of Topnotch Handout

This prostaglandin F2 alpha analog is commonly used off label for its side effect which is lengthening of eyelashes: A. Beraprost B. Dinoprostone C. Alprostail D. Latanoprost E. Gemeprost

116 D. Latanoprost is commonly used off label for eyelash lengthening. It is a prostaglandin F2alpha analog that activates FP receptors, increases outflow of aquaeous humor and reduces intraocular pressure. Its other side effects are blurred vision, eye irritation, or tearing darkening of eyelid skin color, increase in brown color in colored part of eye. Page 32 of Topnotch Handout

Which of the following statements is true for cromolyn: A. It reduces synthesis of leukotrienes B. It has no bronchodilator action C. It binds IgE antibodies on mast cells D. It reduces the expression of cyclooxygenase E. It prevents vagal-stimulated bronchoconstriction

117 B. Cromolyn is a mast cell stabilizer by the prevention of calcium influx, therefore preventing degranulation and release of histamine, leukotrienes and other mediators. A is moa of zileuton C is for omalizumab D is for corticosteroids E is for Ipratropium - muscarinic receptor antagonist. Page 33 of Topnotch Handout

A 23 y/o female previous RHD patient s/p valve replacement is being maintained on warfarin. 3 days prior she had symptoms of PUD for which she self medicated. She now presented at the ER due to black tarry stools, epigastric pain, coffee ground emesis, gum bleeding and dizziness. Laboratory showed prolonged PT. Which of the following drugs caused a drug interaction with warfarin prolonging its anticoagulant effect? A. Cimetidine B. Omeprazole C. Bismuth D. Sucralfate E. Maalox

118 A. CYP 450 inhibitors prolong/increase warfarin's anticoagulant effectsince they inhibit drug clearance. Cimetidine is a CYP450 inhibitor. Page 39 of Topnotch Handout.

A 30 y/o male patient developed methemoglobinemia after receiving prilocaine during a minor surgery. What should be administered to this patient? A. NAC B. Methylene blue C. EDTA D. Dimercaprol E. Deferoxamine

119 B. For methemoglobinemia the antidote is methylene blue. Page 52 of Topnotch Handout

What is the treatment for malignant hyperthermia? A. Dantrolene B. Nitroglycerin C. Paracetamol D. Halothane E. All of the above

12 A. Source: Topnotch handout on Pharmacology

Which nondepolarizing neuromuscular blocker undergoes rapid breakdown by Hoffman elimination? A. Vecuronium B. Mivacurium C. Atracurium D. Tubocurarine E. Rocuronium

120 C. Atracurium is broken down to inactive metabolites by (minor) ester hydrolysis and spontaneous Hoffman elimination (major pathway) to Laudanosine. Page 53 of Topnotch Handout

These are agents or drugs that bind to a different receptor, producing an effect opposite to that produced by the drug it is antagonizing: A. Competitive antagonists B. Irreversible antagonists C. Chemical antagonists D. Physiologic antagonists E. Partial agonists

121 D. Competitive/Reversible antagonist - bind to receptors in a reversible way without activating the effector system; Non-competitive/Irreversible antagonist - cause downward shift of the DRC; Chemical antagonist - interact directly with the drug being antagonized; PHYSIOLOGIC ANTAGONIST is the answer. Examples are histamine and epinephrine, propranolol and thyroid hormone.

The patient is being maintained on oral anticoagulant. Which of the following if taken by the patient will decrease the effect of the drug he is presently taking? A. Phenobarbital B. Sulfamethoxazole C. Valproic acid D. Ketoconazole E. Vancomycin

122 A. Phenobarbital, a barbiturate, is a Cytochrome P450 inducer which will increase clearance of other drugs, thus, decreasing their effects.

Which of the following is a mechanism of action of Hydralazine? A. Alters intracellular calcium B. Opens potassium channel causing arteriolar and venular dilatation C. Blocks alpha-1 adrenergic receptor D. Competitively bloocks Nn nicotinic receptor E. Blocks voltage-gated L-type calcium channels

123 A. Hydralazine binds to and activates potassium channels on vascular smooth muscle resulting to efflux of potassium and subsequent hyperpolarization of the cell. This prevents calcium- mediated activation and constriction of smooth muscle, resulting in vasodilation. It dilates arterioles, but not veins.

The drug of choice for paroxysmal Supraventricular tachycardia is: A. Procainamide B. Verapamil C. Amiodarone D. Lidocaine E. Adenosine

124 E

The most common anti-arrythmic drug used for cardiac arrest is: A. Lidocaine B. Adenosine C. Flecainide D. Sotalol E. Amiodarone

125 A. Lidocaine is the drug of choice for ventricular arrythmias post-MI, and digoxin-induced arrythmias. It is the least cardiotoxic among conventional anti-arrythmics

Which of the following diuretics may cause hyperkalemia? A. Amiloride B. Ethacrynic acid C. Indapamide D. Mannitol E. Acetozalamide

126 A. Amiloride (also Triamterene, Eplerenone, Spironolactone) is a potassium sparing diuretics. All other options are potassium-wasting.

Which of the following anesthetic agent has analgesic property and used as an adjunct to other anesthetics? A. Halothane B. Nitric oxide C. Etomidate D. Desflurane E. Fentanyl

127 E Nitrous oxide, not nitric oxide, is used as an anesthetic agent with analgesic property. Fentanyl is often used for anesthesia and analgesia.

A 50-year old male with DM type II had been on metformin for nine years and presented with diarrhea, paresthesia, and muscular weakness. On examination, he has hyperactive DTRs. Blood electrolytes showed serum Na 138 mEq/L, Serum K= 3.4 mEq/L, serum Ca=8.2 mg/dL and Mg 0.8 mEq/L. Which of the following medications should you avoid to prevent exacerbation of his symptoms? A. Spironolactone B. Magnesium sulfate C. Furosemide D. Indomethacin E. Acetazolamide

128 C. This is a case of hypomagnesemia, which can be exacerbated by administration of a loop diuretic Furosemide is a loop diuretic that acts on the thick ascending limb of the loop of henle. This tubular segment is responsible for a significant sodium chloride reabsorption, as well as the site for calcium and magnesium reabsorption. Side effects of Furosemide: Hypokalemia, hypocalcemia, hypomagnesemia, hyperuricemia, dehydration, metabolic alkalosis, ototoxicity, sulfa allergy, nephritis

Dicycloverine is used for: A. Paralytic ileus B. Biliary colic C. Intestinal spasm D. Constipation E. Urinary retention

129 C. Dicycloverine is an antispasmodic, prescribed for gastrointestinal tract spasm and irritable bowel syndrome. It blocks the activity of acetylcholine on muscarinic receptors. It should NOT be used in patients with PARALYTIC ILEUS, myasthenia gravis, narrow angle glaucoma, enlarged prostate, or pyloric stenosis. Constipation is a side-effect of this drug.

Which of the following is not a 3rd generation cephalosporin A. Ceftazidime B. Ceftriaxone C. Ceftizoxime D. Cefixime E. Cefepime

13 E 1st gen Cefazolin Cephalotin Cephalexin Cephradine Cephapirin 2nd gen Cefaclor, Cefuroxime, Cefoxitin, Cefotetan, Cefamandole, Cefonicid, Cefprozil, Cefometazole 3rd gen Ceftriaxone, Cefoperazone, Cefotaxime, Cefixime, Ceftazidime, Ceftizoxime, Cefpodoxime proxetil, Cefdinir, Cefditoren, Ceftibuten 4th gen Cefepime Source: Katzung Basic and Clinical Pharmacology 11th ed

Which of the following is NOT an effect of Naloxone A. It will cause respiratory depression B. It may cause nausea and vomiting C. It induces abstinence syndrome D. It decreases constipation E. None of the above

130 A. Naloxone is an opioid antagonist used for opioid overdose. It competitively blocks opioid receptors and rapidly reverses effects of opioid agonists. In individuals who are acutely depressed by an overdose of an opioid, the antagonist effectively normalizes respiration, LOC, pupil size, bowel activity, and awareness of pain. In dependent subjects who appear normal while taking opioids, naloxone or naltrexone almost instantaneously precipitates an abstinence syndrome.

The combination of metronidazole and alcohol will most likely cause? A. Ataxia B. Blurring of vision C. Dizziness D. Nausea and vomiting E. Pancreatitis

131 D. Metronidazole has a disulfiram-effect, so that nausea and vomiting can occur if alcohol is ingested during therapy.

The mechanism of action of Nedocromil: A. Activates beta-2 receptors in bronchial smooth muscle B. Prevents calcium influx and stabilizes mast cells preventing release of histamine C. Inhibits 5-lipoxygenase D. Prevents vagal-stimulated bronchoconstriction E. Blocks cysteinyl leukotriene-1 receptor

132 B. Option A - Beta 2 agonist (Salbutamol, Terbutaline); Option C - leukotriene synthesis inhibitor (Zileuton); Option D - Muscarinic receptor antagonist (Ipratropium); Option E - Leukotriene antagonist (Zafirlukast). Cromolyn, Nedocromil, and Lodoxamide are mast cell stabilizers, which acts by preventing calcium influx and stabilizes mast cells, preventing degranulation and release of histamine, leukotrienes, and other mediators.

The mechanism of action of Allopurinol: A. Inhibits microtubule assembly B. Decreases macrophage migration and phagocytosis C. Compete with uric acid for reabsorption in the proximal tubules D. Irreversibly inhibits xanthine oxidase E. Increases uric acid excretion

133 D. Option A and B - MOA of Colchicine; Option C and E - MOA of Probenecid; MOA of Allopurinol: its active metabolite irreversibly inhibits xanthine oxidase and lowers production of uric acid. Acute gout: colchicine Prophylaxis: allopurinol

The preferred antiseizure drug for pregnant women: A. Valproic acid B. Phenobarbital C. Phenytoin D. Carbamazepine E. Topiramate

134 B. Phenobarbital is the preferred anti-seizure drug in children and pregnant women.

Which of the following is an extraluminal amoebicide? A. Metronidazole B. Diloxanide furoate C. Lumefantrine D. Pentamidine E. Nifurtimox

135 A. Tissue amebicides act in the bowel wall and liver: Metronidazole, emetine, chloroquine, tinidazole; (MECT) Luminal amebicide: Diloxanide furoate, Iodoquinol, Paromomycin. (DIP)

Sulfone in the management of leprosy causes? A. Inhibition of folic acid synthesis B. Inhibition of PABA C. Inhibition of DNA synthesis D. Inhibition of protein synthesis E. Inhibition of arabinosyl transferase

136 A. Drugs used in leprosy: Dapsone, Rifampicin, and Clofazimine. Dapsone, a sulfone, is the most active drug used against M. leprae. It is bacteriostatic and inhibits folic acid synthesis. Clofazimine acts by binding to guanine bases in bacterial DNA.

Which of the following antiretroviral would cause hyperlipidemia and hyperglycemia? A. Enfuvirtide B. Indinavir C. Zidovudine D. Delavirdine E. Didanosine

137 B. Indinavir (Protease Inhibitor) - all Pis/'navirs' may cause hyperlipidemia, hyperglycemia, and insulin resistance as potential adverse events; Efuvirtide is a fusion inhibitor, SE: Hypersensitivity, increased incidence of bacterial pneumonia, injection site reaction; Zidovudine and Didanosine (NRTI) - all NRTI carry the risk of lactic acidosis with hepatic steatosis

Which of the following sulfonylurea is safest drug for elderly diabetics? A. Chlorpropramide B. Tolbutamide C. Tolazamide D. Glyburide E. Glimepiride

138 B. Because of its short half-life, Tolbutamide is the safest sulfonylurea for elderly diabetics.

The danger of long term use of propylthiouracil is: A. Cholestatic jaundice B. Exfoliative dermatitis C. Liver failure D. Agranulocytosis E. Arthralgia

139 D. The most dangerous complication of thioamides is agranulocytosis, (granulocyte <500) an infrequent but potentially fatal adverse reaction. Hepatitis can also be fatal. The most common adverse effect of PTU is maculopapular rash. (Katzung).

Which of the following vasodilators can worsen Diabetes Mellitus? A. Hydralazine B. Diazoxide C. Verapamil D. Nifedipine E. Nitroprusside

14 B. The mechanism of action of diazoxide is by opening K channels causing hyperpolarization and relaxation of vascular smooth muscles. However, recall the mechanism of release of insulin in the pancreatic beta cells. Glucose enters cells resulting to increased ATP production. This closes ATP dependent K channels resulting to depolarization, entry of Calcium and release of insulin. Diazoxide causes opening of K channels resulting to hyperpolarization and inhibition of insulin release. Diazoxide is actually used in treatment of insulinomas. Source: Katzung Basic and Clinical Pharmacology 11th ed p 180

This cell-cycle specific drug is commonly used for testicular cancer. It does not cause myelosuppression, however, dose modification is recommended in the setting of renal dysfunction. What is this chemotherapeutic drug? A. Cyclophosphamide B. Cisplatin C. Busulfan D. Bleomycin E. Etoposide

140 D.

You are treating a patient with hyperthyroidism and you prescribed methimazole, you know that this drug inhibits which enzyme? A. Sodium-Iodide co-transporter B. Thyroid peroxidase C. Proteases in the thyroid gland D. All of the above E. None of the above

141 B. Thioamides like methimazole and PTU inhibit the organification of iodine to tyrosine via the enzyme thyroid peroxidase.

Which of the following anti fungals will enhance the activity of cyclosporine? A. Itraconazole B. Fluconazole C. Ketoconazole D. Griseofulvin E. Terbinafine

142 C. Ketoconazole is a potent cytochrome P450 inhibitor and therefore inhibits the metabolism of other drugs and increases their effects, griseofulvin is a cytochrome inducer and thus facilitates the metabolism of other drugs thus decreasing their effect.

Which among the following is the most potent opioid A. Ohmefentanyl B. Morphine C. Fentanyl D. Pethidine E. Remifentanil

143 A. The opioid ohmefentanyl is the most potent opioid

What is the drug of choice in acute arrhythmias A. Metoprolol B. Lidocaine C. Quinine D. Bisoprolol E. None of the above

144 B.

What is the drug of choice in preventing the recurrence of arryhthmia A. Adenosine B. Procainamide C. Lidocaine D. Carvedilol E. None of the above

145 A.

The following antibiotics eliminate intestinal flora thus causing pseudomembranous colitis except A. 3rd gen cephalosporins B. Aminoglycosides C. Clindamycin D. Erythromycin E. None of the above

146 D. Erythromycin is not associated with pseudomembranous colitis

Why is nitroglycerin preferred given sublingually? A. Giving it orally will require a higher dose B. Giving it orally will produce a greater effect C. It D. Protect it from first-pass metabolism E. None of the above

147 D. Sublingual route is preferred as there is less dose needed and it is protected from hepatic first-pass metabolism

RH, a 50 year old bank executive was brought to the ER due to loss of consciousness, upon PE you noted his breath smelling like sweet with garlicky odor, what is the appropriate drug for this case A. Dimercaprol B. EDTA C. Hydration D. Deferoxamine E. Flumazenil

148 A. The patient has been poisoned with arsenic and the drug of choice is dimercaprol

RJLC, a 30 year old female was treated for extraintestinal amebiasis, what is the drug of choice to eliminate the extraintestinal parasites? A. Paromomycin B. Diloxanide furoate C. Metronidazole D. Pyrantel pamoate E. Praziquantel

149 C. Metronidazole eliminates the trophozoite forms of the parasite in the intestine or in the tissues. DIP Intraluminal MET Extraluminal

What enzyme do statins inhibit? A. HMG-CoA synthase B. HMG-CoA reductase C. HMG-CoA lyase D. HMG-CoA transferase E. All of the above

15 B. Statins inhibit the rate-limiting step of cholesterol synthesis. Don't get confused with HMG CoA synthase which is the rate limiting step of ketogenesis. Source: Katzung Basic and Clinical Pharmacology 11th ed p 612

Allopurinol exerts its therapeutic effect by: A. Inhibiting excretion of uric acid at the PCT B. Increasing purine catabolism C. Competes with uric acid for excretion in the kidney D. Inhibiting formation of uric acid E. None of the above

150 D. Allopurinol inhibits the enzyme xanthine oxidase which metabolizes purines to form uric acid. Therefore its activity inhibits the formation of uric acid

This drug reduces the relapse rates for malaria A. Quinine B. Arthemeter-lumefantrine C. Mefloquine D. Doxycycline E. Primaquine

151 E Primaquine eradicates hypnozoites in the liver responsible for causing relapse

Chronic administration of phenobarbital causes metabolic tolerance via which mechanism? A. Stimulation of aldehyde dehydrogenase B. Stimulation of alcohol dehydrogenase C. Stimulation of MEOS D. Stimulation of Monoamine oxidase E. None of the above

152 C. Stimulation of the microsomal ethanol oxidizing system is implicated in the development of tolerance to phenobarbital aside from playing a minor role in individuals with chronic alcoholism. The rest have no role in the development of tolerance to phenobarbital

Most common side effect of clomiphene citrate A. Hot flushes B. Constipation C. Headache D. Allergic skin reactions E. None of the above

153 A. Hot flushes are the most common side effect of use of clomiphene citrate, the rest are occasional side effects

Which of the following antimalarial drugs may precipitate a hemolytic crisis in a patient with G6PD A. Lumefantrine B. Doxycycline C. Artemether D. Quinine E. None of the above

154 D. Quinine is precipitates hemolytic crisis in patients with malaria with G6PD deficiency.

A man is being treated for intestinal strongyloidiasis, you know that the drug of choice is: A. Praziquantel B. Ivermectin C. Melarsoprol D. Pyrimethamine-sulfa E. Trimethoprim-sulfamethoxazole

155 B. Ivermectin is the DOC in the treatment of strongyloidiasis

SMA, a 68 year old female with hypertension is taking captopril, you know that it is effective in hypertension because it inhibits: A. Cathepsin C B. Dipeptidyl dipeptidase-2 C. Dipeptidyl dipeptidase-4 D. Peptidyl dipeptidase E. Procarboxypeptidase

156 D. Peptidyl dipeptidase is otherwise known as ACE which is the enzyme inhibited by ACE inhibitors like captopril.

Intake of potassium supplements is contraindicated in patients taking which of the following? A. Ethacrynic acid B. Amiloride C. Hydrochlorothiazide D. Acetazolamide E. None of the above

157 B. Amiloride is a potassium sparing diuretic which therefore spares renal excretion of potassium raising the potassium levels and therefore is contraindicated in patients taking potassium supplements.

Injection of a small dose of Ach will cause what? A. Reflex bradycardia B. Hypotension C. Splanchnic stimulation D. All of the above E. None of the above

158 D. Ach causes depression of the SA and AV nodes thus slowing the heart rate, hypotension through the relaxation of arterioles, splanchnic stimulation through the stimulation of the muscarinic receptors

Though 1% is systemically absorbed when Ipratropium Bromide is given via nebulization, what is its net effect? A. Reflex tachycardia B. Bronchodilation C. Bronchoconstriction D. Laryngeal spasm E. None of the above

159 B. Bronchodilation is still the net effect of ipratropium, the systemic effect is negligent as the drug is administered to its site of action which is the lungs.

A 67-year-old male with ischemic heart disease, compliant with his medications is admitted for acute renal failure. Labs reveal hyperkalemia, a rapidly elevating creatinine, and markely increased CK and myoglobin levels. Which of the following medications he was taking could have precipitated the condition? A. Aspirin B. Carvedilol C. Losartan D. Captopril E. Rosuvastatin

16 E This is a case of acute renal failure secondary to statin induced rhabdomyolysis. Statins inhibit cholesterol synthesis. One of the intermediates of cholesterol synthesis is farnesyl. Farnesyl is needed for the synthesis of Coenzyme Q, which is important for ATP production in metabolically active cells like the muscles. Depletion of this can result to rhabdomyolysis. To prevent this, one can administer Coenzyme Q10 with statins. Source: Katzung Basic and Clinical Pharmacology 11th ed p 1122

KVM, a 29 year old female was brought to the ER due to ingestion of insecticide, you ordered pralidoxime to be given, what does it do? A. Induces the enzyme Acetylcholinesterase B. Regenerates acetylcholinesterase if given within 6-8 hours after ingestion C. Breaks the bond between organophosphate and acetylcholinesterase D. B and C only E. All of the above

160 D. Pralidoxime regenerates acetylcholinesterase and is only effective when the bond between the organophosphate and acetylcholinesterase hasn't matured which takes 6-8 hours, after which, pralidoxime even if given is no longer effective.

A patient came in for consult due to infertility. It was found out that the patient is having ovulation disorder so you decided to give here clomiphene. It is important to tell to the patient that the most common adverse effect of clomiphene is: A. hot flushes B. bleeding C. constipation D. headache E. nausea and vomiting

161 A. "the most common adverse effects in patients treated with this drug are hot flushes, which resemle those experienced by menopausal patients." Katzung 11th edition page 719

It is the antiarrythmic of choice for termination of ventricular tachycardia and prevention of ventricular fibrillation after cardioversion in the setting of acute ischemia. A. Adenosine B. Esmolol C. Lidocainne D. Procainamide E. Amiodarone

162 C. "Lidocaine is the agent of choice for termination of ventricular tachycardia and prevention of ventricular fibrillation after cardioversion the setting of acute ischemia. However, routine prophylactic use of lidocaine in this setting may actually increase total mortality, possibly by increasing incidence of asystole, and is not the standard of care. Most physicians administer IV lidocaine only to patients with arrythmia" Katzung 11th edition page 239

Jan Deo is a newly diagnosed with essential hypertension. He has no other co morbidities. Aside from lifestyle modification, he was prescribed with Enalapril as his maintainance. Which of the following BEST describes the mechanism of action of the drug. A. blocks Angiotensin receptor which will prevent rise in BP B. inhibits peptidyl dipeptidase C. direct vasodilator D. reduces calcium uptake of smooth muscles E. none of the above

163 B. Enalapril is an ACE inhibitor. One of the many alternative names for ACE is peptidyl dipeptidase.

A patient was given Nedocromil for asthma. Which of the following is the mechanism of action of the drug? A. Inhibits the release of histamine B. Direct relaxation of smooth muscle C. Inhibits leukotriene pathway D. Mast cell stabilizer E. None of the above

164 D. Katzung 11th edition page 349

Among the thioamide antithyroid drugs, which of the following is the one preferred for pregnant women because it crosses the placental barrier less readily giving lesser effect to the fetus A. Potassium iodide B. Ipodate C. Carbimazole D. methimazole E. PTU

165 E PTU is preferable because it is more strongly protein-bound and therefore crosses the placcenta less readily. Katzung 11th edition page 672

All of the following are mechanism of action of propylthiouracil EXCEPT: A. inhibits thyroid peroxidase-catalyzed reactions B. Blocks iodine organification C. Blocks coupling of the iodotyrosinase D. Block uptake of iodide by the gland E. inhibits peripheral deiodination

166 D. All of the choices are mechanisms of action of PTU except for the blocking of uptake of iodide by the gland. Inhibition of thyroid peroxidase-catalyzed reaction and the blocking of iodine organification. Katzung 11th edition page 672.

Fluoxetine (prozac) is one of the most commonly use antidepressants worldwide. Which of the following is the mechanism of action of the drug? A. Selectively inhibits serotonin reuptake B. Inhibits serotonin and norepinephrine reuptake C. selectively inhibits reuptake of norepinephrine D. directly antagonize serotonin E. directly antagonize epinephrine

167 A. SSRIs like Fluoxetine are agents that have their primary action the inhibition of serotonin transporter. Katzung 11th edition page 513.

Which among the following drugs deliver its action through selectively inhibiting reuptake of serotonin? A. venlafaxine B. imipramine C. selegiline D. fluoxetine E. trazodone

168 D. Fluoxetine is a SSRI. Venlafaxine is a SNRI. Imipramine is a TCA. Selegiline is MAOI. Trazadone is a serotonin antagonist. Katzung 11th edition page 514.

All of the following drugs are considered disease-modifying antirheumatic drugs (DMARDs) except: A. Abatacept B. Azathioprine C. Chloroquine D. Methotrexate E. Ketoprofen

169 E Ketoprofen is a non selective COX inhibitor

Colchicine is a widely used drug in the treatment of gout. Which among the following describes the mechanism of action of the drug? A. Binding to intracellular tubulin to exert anti inflammatory effect B. It increases the excretion of uric acid through urine C. inhibits xanthine oxidase catalyzed reactions D. non purine xanthine oxidase inhibitor E. none of the above

170 A. B. are uricosuric agents like probenecid and sulfinpyrazone C. Allopurinol D. Febuxostat

A patient came in at the ER having severe headache and heat intolerance. Examination revealed enlarged thyroid, high blood pressure and severe tachycardia. On history, his relative told you that he also has frequent attacks of asthma. In your findings, you are suspecting that he is having thyroid storm. Which of the following drugs can be given immediately to relieve hypertension and tachycardia? A. propanolol B. esmolol C. diltiazem D. nifedipine E. clonidine

171 C. Take note, the patient has asthma If propanolol is contraindicated by the presence of severe heart failure or asthma, then hypertension and tachycardia may be controlled with diltiazem, 90-120mg orally three or four times daily or 5-10 mg/h by IV infusion Katzung 11th edition page 677

A post stroke patient is taking in warfarin. Which among the following drugs will most likely increase the possibility of having warfarin toxicity when added in his management? A. Smoking B. Barbiturates C. Carbamazepine D. Isoniazid E. Ethanol

172 D. Isoniazid in a P450 inhibitor which will delay the excretion of warfarin when given. Other choices are P450 inducers

A G7P7 (6107) came in for consult. She asks you that she wants to take oral contraceptive pills as family planning. If given in conjunction with OCP, which among the following will decrease the effectivity of OCP? A. Isoniazid B. Cimetidine C. Ketoconazole D. Erythromycin E. Rifampicin

173 E Rifampicin is a CYP450 inducer which hastens the excretion of OCP

This drug inhibits intestinal absorption of phytosterol and cholesterol which will eventually decrease the level of LDL. A. rosuvastatin B. ezetimibe C. Niacin D. Gemfibrozil E. Simvastatin

174 B.

All of the following reactions are considered Phase 2 reactions in drug metabolism except: A. Glucoronidation B. Hydoxylation C. Glutathione conjugation D. Sulfation E. Methylation

175 B. Other choices are phase 1 reaction

Which of the following correctly describes FDA Drug category B? A. Either animal studies revealed adverse effects on the fetus and there are no controlled studies in women or studies in women and animals are not available B. Control studies in women fail to demonstrate a risk to the fetus in the first trimester and the possibility of fetal harm is remote C. There is positive evidence of human fetal risk but the benefits from use in pregnant women may be acceptable despite the risk D. Animal reproduction studies have shown an adverse effect that was not confirmed in controlled studies in women in the first trimester E. None of the above

176 D. A: Category C B. Category A C. Category D

Which of the following antineoplastic drugs can cause hemorrhagic cystitis? A. Cyclophosphamide B. Cisplatin C. Vincristine D. Methotrexate E. Bleomycin

177 A. Cisplatin are nephrotoxic and ototoxic. Vincristine causes peripheral neuropathy. Methotrexate usually causes myelosuppression. Bleomycin is usually associated with pulmonary fibrosis

A cancer patient is scheduled for methotrexate chemotherapy. Which of the following rescue agent/s is used to alleviate the toxic effect of the drug? A. Dexrazoxane B. Mesna C. Leucovorin D. Amifostine E. None of the above

178 C. Dexrazoxane is a rescue therapy for doxorubicin; Mesna is for cyclophosphamide; Amifostine is for cisplatin

Which of the following antibiotics is a protein synthesis inhibitor at 30S sub unit? A. Ciprofloxacin B. Azithromycin C. Clindamycin D. Linezolid E. Doxycycline

179 E Ciprofloxacin is a fluoroquinolone which inhibits topoisomerase. The other choices inhibits at 50s subunit.

Which of the following drugs are beneficial in delaying organ remodeling such as in cardiac remodeling in heart failure and reduces progression of DM nephropathy? A. Captopril B. Losartan C. Eplerenone D. A and B only E. All of the above

18 E Source: Topnotch Handout on Pharmacology

Which of the following cephalosporins can be used to patients with Pseudomonas infection? A. Cefuroxime B. Cefazolin C. Cefoxitin D. Ceftazidime E. Ceftriaxone

180 D. Antipseudomonal cephalosporins are ceftazidime, cefepime, and cefoperazone

What is the mechanism of action of metronidazole? A. Inhibits DNA replication by binding to DNA gyrase B. Reactive reduction by ferredoxin forming free radicals C. Forms multiple reactive intermediates when acted upon by bacterial enzyme D. Unknown E. None of the above

181 B. A- fluoroquinolones. C- nitrofurantoin.

59 year old male, hypertensive for 22 years, came in the ER due to difficulty of breathing. PE revealed crackles on bilateral lower lobes of the lungs and bipedal edema. Which of the drugs used in CHF can prolong survival of patients? A. carvedilol B. atenolol C. digitalis D. furosemide E. A and B

182 A. My mnemonics: improves QUANTITY of life in CHF- ABS (ACEI/ARBS, Beta-Blockers, Spironolactone). Improves QUALITY of life- Digital Film (Digitalis, Furosemide). Only 3 beta blockers are found to be useful in CHF: Carvedilol, metoprolol and bisopolol.

Your patient requires a local anesthetic with long duration of action. What will you give? A. procaine B. lidocaine C. cocaine D. bupivacaine E. None of the above

183 D. My mnemonics for this: BeT-long. Local anesthetics with long duration of action are Bupivacaine and Tetracaine. The longest acting is Ropivacaine ("at the end of a long ROPe")

What anti-retroviral agent causes peripheral neuropathy and pancreatitis? A. Zidovudine B. Zalcitabine C. Didanosine D. Abacavir E. Foscarnet

184 C. Zidovudine causes bone marrow suppression. Abacavir causes hypersensitivity. Didanosine causes peripheral neuropathy and pancreatitis. Stavudine and Zalcitabine causes peripheral neuropathy.

Which of the following anti-microbial agent is bacteriostatic? A. Co-trimoxazole B. Pyrazinamide C. Metronidazole D. Aminoglycoside E. Tetracycline

185 E Bacteriostatic agents: All protein synthesis inhibitors except aminoglycoside; ethambutol, nitrofurantoin; those with anti-metabolite mechanism of action- sulfonamide and trimethoprim (but they become bactericidal when combined).

Drug of choice for hydatid disease? A. Niclosamide B. Praziquantel C. Albendazole D. Mebendazole E. Pyrantel pamoate

186 C. Albendazole is the drug of choice for hydatid disease and cysticercosis.

Which of these drugs will be the fastest to be eliminated from the body? A. 100mg drug with zero order kinetics; elimination rate is 25mg/2hrs B. 100mg drug with first order kinetics; elimination t 1/2 is 2 hours C. 150mg drug with zero order kinetics; elimination rate is 25mg/hr D. 150mg drug with first order kinetics; elimination t 1/2 is 2 hours E. All of these will be eliminated at the same time

187 C.

Local anesthetics are not effective in inflamed infected tissues because? A. The cationic form is increased B. Protonated form predominates C. The drug is more water soluble D. A and B E. All of the above

188 E Local anesthetics are weak bases, therefore, in an acidic environment like in infected tissues, the protonated or cationic form of the drug predominates. This is water soluble so the drug undergoes rapid clearance rather than being able to cross biological membranes.

The following are adverse effects of amiodarone except? A. hyperthyroidism B. hypothyroidism C. photodermatitis D. pulmonary fibrosis E. None of the above

189 E Adverse effects of amiodarone: bradycardia, heart block, pulmonary fibrosis, elevated liver enzymes, photodermatitis, corneal microdeposit, halos in peripheral visual field, optic neuritis, hypothyroidism (blocks the peripheral conversion of T4 to T3) and hyperthyroidism (because it is also a potential source of large amounts of inorganic iodine)

For the treatment of brain tumors, it is important for a drug to be able to penetrate the blood brain barrier. Which of the following antineoplastic agents cannot cross the blood brain barrier? A. Procarbazine B. Dacarbazine C. Streptozocin D. Carmustine E. 5-FU

19 B. Procarbazine, Dacarbazine, Nitrosureas (Lomustine, Carmustine, Streptozocin) are chemotherapeutic agents which are able to penetrate the blood brain barrier. Source: Topnotch Handout on Pharmacology

This opioid antagonist is used in opioid and alcohol dependence A. Nalmefene B. Naltrexone C. Naloxone D. Nalbuphine E. A and C

190 B. Naltrexone reduces craving in alcohol dependence. Note: there are probably 4 questions about opiates in our boards.

What is the mechanism of action of PTU? A. Blocks iodination and organification B. Inhibits iodide trapping C. Blocks coupling reaction D. Blocks peripheral conversion of T4 to T3 E. Inhibits secretion

191 D. There are 5 steps in thyroid hormone synthesis. 1. iodide trapping; 2. iodination/organification; 3. coupling; 4. secretion; 5. peripheral conversion. Steps 2 to 5 are actually blocked by PTU but the main effect among these is the inhibition of peripheral conversion.

This 3rd generation cephalosporin can cross the blood brain barrier A. ceftizoxime B. cefpodoxime C. ceftibuten D. cefaclor E. cefuroxime

192 A. All 3rd generation cephalosporins can cross the blood brain barrier except: ceftibuten, cefpodoxime, cefixime, cefoperazone.

These drugs increase the outflow of aqueous humor used in glaucoma except? A. timolol B. latanoprost C. epinephrine D. pilocarpine E. physostigmine

193 A. Drugs that increase outflow of aqueous humor: pilocarpine, physostigmine, latanoprost, epinephrine. Drugs that decrease aqueous humor secretion; mnemonics- TAMAD: Timolol, Acetazolamide, Mannitol, Apraclonidine, Dorzolamide.

Patient came in the clinic complaining of amenorrhea and galactorrhea. History revealed intake of an anti-psychotic drug. What could have caused the symptoms of the patient? A. Clozapine B. Olanzapine C. Risperidone D. Haloperidol E. Chlorpromazine

194 C. Risperidone causes hyperprolactinemia.

Which has the highest partition coefficient among the inhalation anesthetics? A. methoxyflurane B. Nitrous oxide C. isoflurane D. sevoflurane E. desflurane

195 A. Methoxyflurane has the highest partition coefficient which means that it has the slowest onset and recovery.

Patient is diagnosed with Lennox-Gaustat Syndrome and is taking maintenance drug. He later on developed urolithiasis. The patient is probably taking what drug? A. topiramate B. lamotrigine C. ethosuximide D. carbamazepine E. levetiracetam

196 A. Topiramate is used as an adjunct in Lennox-Gaustat Syndrome. Its adverse effects are unusual/weird for anti-seizure drugs (ex. Urolithiasis, myopia, glaucoma)

Escitalopram is an antidepressant agent that belongs to which drug class? A. SSRI B. SNRI C. TCA D. MAOI E. 5HT2 antagonist

197 A. Drugs that belong to SSRI: fluoxetine, sertraline, paroxetine, fluvoxamine, citalopram, escitalopram

7 year old male was noted to have difficulty sustaining attention in tasks or play activities in school and at home. He often leaves his seat inside the classroom and runs about and climbs excessively. What is the treatment of choice for this case? A. sibutramine B. methylphenidate C. diazepam D. behavioral therapy E. magnesium pemoline

198 B. Methylphenidate is the drug of choice for ADHD in children 6 years old and above. Mechanism of action is unknown but it mainly acts as a CNS stimulant similar to amphetamines.

What drug acts on epithelial sodium channels in cortical collecting ducts and is used to reduce potassium excretion in cases of hypokalemia? A. furosemide B. acetazolamide C. spironolactone D. triamterene E. hydrochlorothiazide

199 D. Triamterene and Amiloride are potassium sparing diuretics that act on epithelial sodium channels (ENaC) in cortical collecting duct, causing decreased Na reabsorption and K excretion. Spironolactone and eplerenone are also potassium sparing diuretics but they act on aldosterone receptors.

Which of the following anti-TB drug is matched incorrectly with its adverse effects? A. Most hepatotoxic - Pyrazinamide B. Psychosis - Isoniazid C. Most ototoxic - Streptomycin D. Most associated with skin rashes due to hypersensitivity - Rifampicin E. Most associated with visual impairment - Ethambutol

2 D. Anti-TB drug most associated with skin rashes is streptomycin. Source: National TB Control Program Manual of Procedures

Which of the following is not considered a controller drug for asthma? A. Budesonide B. Terbutaline C. Montelukast D. Nedocromil E. Omalizumab

20 B. Terbutaline is a bronchodilator used for acute relief of asthma exacerbations. Source: Topnotch handout on Pharmacology

Which of the following beta-blockers have local anesthetic effect? A. Nadolol B. Penbutolol C. Pindolol D. Carteolol E. Atenolol

200 C. Beta-blockers with partial agonist effect on adrenergic receptors have intrinsic sympathetic activity: LAPPCC (Labetalol, Acebutolol, Penbutolol, Pindolol, Carteolol, Celiprolol). Beta-blockers with membrane stabilizing activity have local anesthetic effect: LAMPP (Labetalol, Acebutolol, Metoprolol, Pindolol, Propranolol)

In the stomach, aspirin will exist predominantly as the ______________ form. A. Hydrophilic, ionized, polar B. Lipophilic, unionized, non-polar C. Hydrophilic, unionized, non-polar D. Lipophilic, ionized, polar E. Hydrophilic, unionized, polar

201 B. Review the Henderson-Hasselbach equation. Aspirin (acetylsalicyclic acid) is acidic in nature. In an acidic environment like the stomach, it will exist predominantly as the lipid-soluble, unionized, non- polar form.

In the CHOP regimen for non-Hodgkin's lymphoma, which chemotherapeutic drug acts by inhibiting the mitotic spindle? A. Cyclophosphamide B. Hydroxyurea C. Oncovin D. Prednisone E. Doxorubicin

202 C. Oncovin is vincristine.

A patient with leprosy is taking a drug that casts a red color to the skin. He is probably taking: A. Dapsone B. Rifampin C. Clofazimine D. Ethambutol E. Phenylbutazone

203 C. Rifampin causes red discoloration of body fluids but not the skin.

What part of the pharmacokinetics of penicillin is affected by the drug probenecid? A. Glomerular filtration B. Tubular reabsorption C. Tubular secretion D. Hepatic uptake E. Protein-binding with albumin

204 C. Probenecid raises plasma concentration of penicillin by inhibiting its tubular secretion in the renal tubules.

Hoffman elimination is exhibited by: A. Succinylcholine B. Diazepam C. Phenobarbital D. Phenytoin E. Atracurium

205 E Hoofman elimination aka exhaustive methylation

In a patient suffering from organophosphate poisoning with a cardiac rate of 45, what drug should be given? A. Neostigmine B. Atropine C. Epinephrine D. Pancuronium E. Edrophonium

206 B. Because of its antimuscarinic effect on the heart, atropine (the antidote for organophosphate poisoning) has a positive chronotropic effect.

Patients receiving cyclophosphamide should also be given: A. Mesna B. Leucovorin C. Vitamin B6 D. Thiamine E. Amifostine

207 A. Leucovorin for methotrexate. Pyridoxine for isoniazid. Thiamine for alcoholics.

Which statement is correct regarding the aminoglycosides? A. They work in anaerobic conditions. B. They exhibit concentration-dependent killing. C. Most drugs of this class are myelosuppresive. D. They cannot be given to patients allergic to penicillins. E. They are acidic in nature.

208 B. Aminoglycosides only work in arebic conditions. They do not have cross-reactivity with the penicillins. Most drugs are nephrotoxic and ototoxic.

What is the second messenger system for the receptors blocked by metoprolol? A. cAMP B. IP3 C. DAG D. cGMP E. PLC

209 A. Beta-2 receptors are coupled to the cAMP second messenger system.

A 45-year-old man with a history of medication-controlled hypertension presents to your office with complaints of a painful, swollen big toe on the left foot. You suspect gout and check his uric acid levels, which are elevated. From looking at the list of the medications the patient is taking, you realize that one of the medications may be the cause of his current symptoms. Which medication might that be? A. Acetazolamide B. Amiloride C. Spironolactone D. Hydrochlorothiazide E. Mannitol

21 D. Hydrochlorothiazide, a thiazide diuretic, can precipitate a gouty attack in predisposed individuals. This is because these agents increase serum uric acid as a result of competition for the organic acid carrier. Loop diuretics can have this effect too. Acetazolamide is a carbonic anhydrase inhibitor; this agent does not have a significant impact on the levels of uric acid. Amiloride and spironolactone are potassium-sparing diuretics, and they do not have a significant impact on the levels of uric acid either. The same is true for mannitol, an osmotic diuretic.

In the treatment of Parkinson's disease, what drug is given with levodopa to increase the drug's bioavailability? A. Entacapone B. Selegiline C. Carbidopa D. Fluoxetine E. Pyridoxine

210 C. Carbidopa peripherally inhibits the enzyme DOPA decarboxylase, the enzyme that converts DOPA to dopamine. Dopamine itself is unable to cross the blood-brain barrier but DOPA can. Inhibition of the enzyme results in more DOPA available to enter the CNS. Within the CNS, DOPA is converted to dopamine by the same enzyme. However, it is not inhibited by carbidopa at this location.

Which drug used in the treatment of depression possess a tricyclic ring? A. Imipramine B. Fluoxetine C. Trazodone D. Venlafaxine E. Bupropion

211 A. Imipramine is a tricyclic antidepressant.

Which antiretroviral drug acts through the M2 ion channel? A. Efavirenz B. Nevirapine C. AZT D. Amantadine E. Indinavir

212 D. Amantadine is also effective for influenza.

Drug X has greater affinity for albumin than Drug Y. Considering all other parameters are the same, what significant drug-drug interaction is expected to take place when both drugs are administered simultaneously? A. Drug X will have a higher plasma concentration than Drug Y. B. Drug Y will not be filtered by the renal glomerulus. C. Drug Y will not be available to interact with its receptors in the tissues. D. The free plasma drug concentration of Drug Y will increase. E. Drug X can exert its full biologic activity.

213 D. Only the free, unbound drug can interact with receptors, exert biologic activity, and be excreted by the kidneys.

Dissociative anesthesia is effectively achieved by the combination of: A. Thiopental, diazepam, ketamine B. Ketamine, nitrous oxide, fentanyl C. Succinylcholine, thiopental, diazepam D. Nitrous oxide, halothane, fentanyl E. Succinylcholine, halothane, thiopental

214 B.

Among the inhaled anesthetics, nitrous oxide has the _______ MAC and the ______ potency. A. Lowest, lowest B. Lowest, highest C. Highest, lowest D. Highest, highest E. Intermediate, intermediate

215 C. The MAC is a measure of an inhaled anesthetic's potency. The relationship is inverse.

Which properties of digoxin make it an effective treatment in cases of heart failure? A. Positive inotrope, positive chronotrope B. Positive inotrope, negative chronotrope C. Negative inotrope, negative chronotrope D. Negative inotrope, positive chronotrope E. None of the above

216 B. Digoxin also slows AV conduction (negative dromotrope)

In anticoagulation, warfarin therapy is usually overlapped with heparin for the first 1-2 days. Why is this so? A. To achieve supraoptimal anticoagulation during critical periods of illness as warfarin and heparin have synergistic effects. B. To prevent bleeding as heparin partially counteracts warfarin's hemorrhagic property. C. Warfarin is metabolized slowly thus leading to a delay in anticoagulation if heparin is not also given. D. To compensate for warfarin's initial prothrombotic property. E. Heparin decreases the clearance of warfarin thus achieving greater plasma drug concentration of warfarin.

217 D. Warfarin acts by vitamin K anatagonism. The initial prothrombotic effect of warfarin is because the synthesis of protein S, an anticoagulant, also depends on vitamin K. Thus, protein S levels decline initially and the effect of protein C (for which it serves as a co-factor) is also diminshed.

A commercial product for colds contain the following: Phenylephrine, chlorpheniramine, and paracetamol. Which acts as the decongestant? A. Paracetamol B. Chlorpheniramine C. Phenylephrine D. Both chlorpheniramine and phenylephrine E. This product does not contain a decongestant.

218 C. Phenylephrine constricts blood vessels through stimulation of adrenergic receptors. This acts as a decongestant.

Which antimalarial drug should not be given to a patient with glucose-6-phosphate dehydrogenase deficiency? A. Chloroquine B. Quinine C. Atovaquone-proquanil D. Mefloquine E. Primaquine

219 E

A 24-year-old student has been taking over-the-counter diphenhydramine for her allergy symptoms most of her life. Lately, however, she has had more frequent symptoms, so he increased the dose of the medication. She now asks her friend, who is a medical student, to explain to her how exactly this agent makes her more sleepy lately. What is the most likely answer regarding diphenhydramine? A. It blocks H1-receptors in the brain B. It modulates the release of dopamine and serotonin C. It acts peripherally, since it does not cross the blood-brain barrier D. It exerts its effects via muscarinic-cholinergic agonist activity E. It contains tryptophan, which produces sedation

22 A. Diphenhydramine blocks H1-receptors in the brain, thereby producing sedation. The release of dopamine and serotonin is modulated via H3- receptors. Diphenhydramine readily crosses the blood-brain barrier. This agent has muscariniccholinergic agonist properties. It is not known to contain tryptophan.

What drug should be given to an immunocompromised patient with CMV retinitis? A. Acyclovir B. Vidarabine C. Indinavir D. Ganciclovir E. AZT

220 D. Ganciclovir is the drug of choice for CMV. Foscarnet is an alternative.

What is the pharmacologic basis of using Timolol in open angle glaucoma? A. Increaed outflow via dilatation of uveoscleral veins B. It leads to opening of trabecular meshwork C. It suppresses the ciliary epithelium from producing aqueous humor D. lt decrease production of aqueous humor by decreasing levels of bicarbonate through cAMP mediated pathway. E. none of the above

221 C. MOA: A- alpha agonist and epinephrine; B- Cholinomimetics ( physostigmine, pilocarpine) D- there is no such thing cAMP dependent decrease in bicarbonate; however inhibition of Carbonic anhydrase leads to decrease humor production via decreased bicarbonate levels.

A 7 yo male was brought to ER secondary to symptoms suggestive of botulinum toxin ingestion, which of the following are likely included in the symptoms of the patient? A. Muscle fasciculation B. Bronchoconstriction C. Increased bowel movement D. blurred near vision E. hypertension

222 D. Botulinum toxin inhibits the release of vesicular contents from all types of cholinergic nerve endings. Since both ganglionic/autonomic and peripheral motor synapse are affected. Patient will present with hypotension and lack of muscular contraction. Blurred near vision is a result of paralysis of ciliary muscle aka cycloplegia

Which of the following will increase the arrythmogenic side effect of digoxin? A. Decrease sympathetic discharge B. Increase serum Potassium C. Decrease serum Calcium D. Increase Parasympathetic discharge E. None of the above

223 D. Hypokalemia, hypomagnesemia, hypercalcemia, and increase vagal tone increases the risk for digoxin induced arrhythmia

Which of the following is correct regarding isosorbide mononitrate? A. Can be use along with PDE inhibitors compared to other forms of nitroglycerin B. It has a similar mechanism of action with Nitroprusside by phosphorylation of myosin light chain phosphate C. It has no direct effect on cardiac muscle D. The primary mechanism for decrease in BP is due to decrease in afterload E. B and C are correct

224 C. ISMN is the oral form of nitroglycerin. Its mechanism is by activating guanylyl cyclase causing an increase in cGMP eventually leading dephosphorylation of MLC. Its antihypertensive effect is primarily via venodilation. It has no direct inotropic or chronotropic effect to the heart. all forms of nitroglycerin should not concomitantly use with anyform of PDEI ( sildenafil) since it will cause profound hypotension and hypoperfusion of critical organs

This drug when given IV and supraphysiologic dose will cause conduction block on AV node via increasing K efflux? A. Verapamil IV B. Adenosine C. Fleicanide D. Amiodarone E. None of the above

225 B. Verapamil can also decrease AV node conduction however via blocking L type Ca channels and not K channels. Fleicanide is a class IC drug that affects primarily Na channels present in both atria and ventricles. Amiodarone is the most efficacious of all antiarrhythmic drug since it blocks Na, Ca, K and Beta receptors. the question is specific on its action in reference to AV node. The best answer is adenosine.

Which of the following drugs can cause prolonged QT interval leading to Torsade de Pointes arrhytmia? A. Esmolol B. Propafenone C. Procainamide D. Verapamil E. None of the above

226 C. Antiarrhytmic drugs that affects the K channel at phase 3 causes prolongation of Action Potential leading prolonged QT interval. These drugs are of the Class IA and III. Class IA drugs - procainamide, quinidine, disopyramide. Class III- Ibutilide, sotalol, dofetilide

Which in the following drugs is classified as 5HT1D agonist? A. Tegaserod B. Ketanserin C. Ondansetron D. Naratriptan E. None of the above

227 D. Naratriptan and its prototype Sumatriptan are 5HT 1D agonist use for the treatment of acute migraine and cluster headache. Tegaserod indicated for constipation is a 5HT4 partial agonist. Ondansetron is a 5-HT3 antagonist ise for antiemesis in post operative and postchemotherapy patients. Ketanserin is a 5HT2 antagonist use for controlling the systemic effects of carcinoid syndrome.

The primary indication of Epoprostenol? A. Open angle glaucoma - increase outflow of aqueous humor B. Severe pulmonary hypertension C. Postpartum bleeding D. All of the above E. None of the above

228 B. Epoprostenol is a Prostacyclin analog use for dialysis to prevent platelet aggregation and also for pulmonary hypertension. Latanoprost or PGF2a analog is the one use for glaucoma. Misoprostol or RU 486 (PGE1 analog) has an off label indication for induction of labor. It primarily acts as an abortifacient.

What is the site of action of corticosteroids in terms of its antiinflammatory effect? A. Cyclooxygenase 1 and 2 B. Phospholipase C C. Phospholipase A2 D. Thromboxane synthase E. All of the above

229 C. Corticosteroids exerts its anti inflammatory effect by inhibiting the Phospholipase A2 enzyme which is responsible for generation of arachidonic acid from membrane lipids. Phospholipase C is not connected with corticosteroids and its function is to generate IP3 and DAG. Cyclooxygenase and thromboxane synthase is primarily inhibit by Coxibs and NSAIDS.

Somatostatin interacts with a A. Gq-protein-coupled receptor B. Gi-protein-coupled receptor C. Ligand-activated ion channel D. Receptor-activated tyrosine kinase E. Intracellular nuclear receptor

23 B. Somatostatin (growth-inhibiting factor) binds to Gi-coupled protein receptor, initiating exchange of GTP for GDP, which inhibits AC and leads to reduced cAMP production.

The major systemic side effects of systemic corticosteroids are much more likely to occur if at least the duration of treatment is more than_____? A. 14 days B. 4 weeks C. 3 weeks D. 20 days E. 2 months

230 A. Katzung Review of pharmacology 8th ed pp 171.

What is the most serious side effect of Methanol ingestion? A. Retinal damage B. Hepatotoxicity C. Blood dyscrasia D. Nephrotoxicity E. Neurotoxicity - status epilepticus

231 A. Methanol or wood alcohol is metabolized by alcohol dehydrogenase to produce a product called formaldehyde and formic acid. This metabolite causes severe acidosis and prominent retinal damage. Nephrotoxicity is a major side effect of ethylene glycol poisoning via its oxalic acid metabolite. Treatment for both alcohols is by administration of fomepizole (inhibitor alcohol dehydrogenase) and ethanol (competitive substrate for alcohol dehydrogenase).

Which of the following antiseizure drugs can inhibit cytochrome isoenzymes leading to increase plasma concentration of other drugs? A. Phenobarbital B. Lorazepam C. Oxcarbazepine D. Valproic acid E. None of the above

232 D. Oxcarbazepine, carbamazepine, Barbiturates, Phenytoin are all drug metabolizing inducers. Only Valproic acid in the choice is an inhibitor. Benzodiazepines do not affect the function of the cytochromes.

Which of the following anti-Parkinsonian drugs can cause exacerbation of psychosis and produce ergot related effects such as pulmonary fibrosis and erythromelalgia? A. Amantadine B. Bromocriptine C. Entacapone D. Haloperidol E. None of the above

233 B. Katzung Review of pharmacology 8th ed pp 233. Both Levodopa and bromocriptine can cause exacerbation of schizophrenia by increasing brain dopamine levels. However, only bromocriptine can cause ergot related side effects.

Prolonged treatment with a PPAR alpha ligand in the treatment of hyperlipidemia will increase the risk for the following condition? A. Cutaneous flushing B. Hyperuricemia leading to gout C. Intestinal malabsorption D. Gallstones E. hepatotoxicity

234 D. PPAR alpha ligands is the MOA of fibric acid derivatives (gemfibrozil and fenofibrate). It increases the supersaturation of bile leading to cholelithiasis. Risk for hepatotoxicity only significantly increase if gemfibrozil is combined with HMG CoA inhibitor. Cutaneous flushing and hyperuricemia are toxicity profile of niacin. Intestinal malabsorption is a primary side effect of resins (cholestyramine, colestipol, colsevelam)

Which of the following is not true in the pharmacologic profile of Aminoglycosides? A. The primary mechanism of resistance is due to plasmid mediated group transferases B. Bactericidal action continuous even if the plasma drug concentration is below measuring levels C. It does not have an activity against anaerobic organisms D. high dose can cause respiratory paralysis E. none of the above

235 E Aminoglycoside exerts concentration dependent killing and post antibiotic effect. It also has curare like action at high dose leading to respiratory paralysis. Aminoglycosides needs Oxygen dependent transport to exerts inhibitory effect 30s ribosome. therefore, it has no activity to anaerobes. acetylation by plasmid mediated group transferase is the major mode of resistance.

Which of the following is a drug of choice for daily dose prophylaxis in AIDS patient with a CD4 count of 50/UL? A. Cotrimoxazole B. Kanamycin C. Azithromycin D. Erythromycin E. Clarithromycin

236 E AIDS patient with a CD4 count of less than 50/UL are prone to have Mycobacterium Avium Intracellulare infection. The only approved prophylactic treatment are daily dose of clarithromycin with or without rifabutin and once weekly dose of azithromycin.

Which in the following choices can be primarily use for the treatment of hemolymphatic stage of trypanosomiasis and for Pneumocystis jiroveci infection A. Sodium Stibogluconate B. Metronidazole C. Nifurtimox D. Pentamidine E. Cotrimoxazole

237 D. Sodium stibogluconate - treatment of all forms of leishmaniasis infection; Metronidazole is not effective in the said infection, it is use for giardia and trichomonas infection. Nifurtimox is for chagas diseae (T cruzi infection). Cotrimoxazole is used for P jiroveci but not effective in Trypanosomiasis infection.

What is the drug of choice for all forms of schistosomiasis and paragonimiasis? A. Diethylcarbamazine B. Praziquantel C. Ivermectin D. Albendazole E. Piperazine

238 B. DEC is the drug of choice for loa loa infection, it is also an alternative drug for elephantiasis. Ivermectin - DOC for onchocerciasis, cutaneous larva migransm and strongyloides. Albendazole- ascariasis, hookworm, pinworm, whipworm, cysticercosis and hydatid disease. Piperazine is an alternative drug for ascariasis.

In what phase of the cell cycle does vinca alkaloid exerts its effect A. G0 B. G1 C. S D. G2 E. M

239 E Vinca alkaloids ( vinblastine, vincristine, vinorelbine) blocks the formation of mitotic spindle by preventing the assembly of tubin dimers into microtubules. Therefore it acts on the M phase.

A 50-year man with mild hypertension complains of discomfort in his chest. He has slightly enlarged fat deposits in his breasts with prominent nipples. Which of the following medications might be causing this adverse effect? A. Amiloride B. Spironolactone C. Metolazone D. Hydrochlorothiazide E. Acetazolamide

24 B. Spironolactone antagonizes the action of the mineralocorticoid, progesterone, and androgen receptors. Inhibition of androgen receptors can lead to gynecomastia and breast tenderness, most often in men.

Which of the following drugs/drug regimen is primarily used for Hodgkin's Lymphoma A. ABVD - adriamycin, bleomycin, vincristine, dacarbazine, prednisone B. PEB - cisplatin, etoposide, bleomycin C. Gemcitabine + 5 FU + radiation D. EMACO- etoposide, methotrexate, actinomycin D, cyclophosphamide, vincristine E. all of the above

240 A. B - testicular cancer C - pancreatic cancer D - choriocarcinoma

25 year old female came in with chief complaint of cough of 3 weeks duration, this was accompanied by night sweats, weight loss, body malaise. PE revealed slightly pale palpebral conjunctiva, multiple bilateral posterior cervical lymphadenopathies but with no adventitious breath sounds on bilateral lung fields. The rest of the physical examination was unremarkable, which of the following drugs is considered to be the most active for her condition? A. Rifampicin B. Isoniazid C. Pyrazinamide D. Ethambutol E. streptomycin

241 B. Considered as the most active anti-TB drug, in fact the only drug given to treat latent tuberculosis

A 56 year old patient diagnosed with rheumatic heart disease underwent mitral valve replacement and is placed on oral anticoagulant Warfarin, she comes to your clinic for a follow up check up and complaints of heartburn and epigastric pain, which of the following medications will you avoid prescribing? A. Cimetidine B. Aluminum Magnesium Hydroxide C. Lansoprazole D. Ranitidine E. Sucralfate

242 A. Cimetidine is a CYP inhibitor. Warfarin is majorly metabolized by CYP 2C9 Cimetidine inhibits this enzyme among others which inhibits warfarin metabolism which increases its toxicity.

Which of the following medications is considered as an inhibitor of glucocorticoid synthesis which could be used in the treatment of cushing's syndrome and some types of cancers? A. Methylprednisolone B. Anastrozole C. Danazol D. Aminogluthetimide E. Flutamide

243 D. Aminoglutethimide inhibits desmolase, blocking conversion of cholesterol to pregnenolone, which reduces synthesis of all hormonally active steroids Anastrozole inhibits only estrogen synthesis Danazol is an androgen Flutamide is androgen antagonist

Which of the following drugs can cause cataract formation? A. allopurinol B. pyrazinamide C. propanolol D. omeprazole E. ciprofloxacin

244 A.

Which of the following chemotherapeutic regimens could be used for the treatment of advanced hodgkin's disease? A. FOLFOX B. MOPP C. CHOP D. 7+3 E. BEP

245 B. MOPP nitrogen mustard, oncovin, procarbazine, prednisone Take time to know the common regimens for other cancers as well, e.g. FOLFOX (Folinic acid, fluorouracil and oxaliplatin) - colorectal CHOP (Cyclophosphamide, hydroxydaunorubicin, oncovin (vincristine), prednisone) - Non-Hodgkins BEP (bleomycin, etoposide, and cisplatin) - testicular 7+3 (7 days of standard-dose cytarabine, and 3 days of an anthracycline antibiotic or an anthracenedione, most often daunorubicin (can be substituted for doxorubicin or idarubicin or mitoxantrone)) - AML

Which of the following medications causes mydriasis but without cycloplegia? A. tropicamide B. homatropine C. cylopentolate D. phenylephrine E. atropine

246 D. Only phenylephrine among the choices causes mydriasis without affecting accommodation

When a pregnant 18 year old female delivered her child, her baby was noted to have characteristic small palpebral fissures, smooth philtrum and thin upper lip, which of the following substances when ingested by the mother during pregnancy could potentially be the cause of such changes? A. phenytoin B. warfarin C. ethanol D. captopril E. isotretinoin

247 C. Fetal alcohol syndrome Know the common syndromes and teratogens.

What is the predominant reason why nitroglycerin is generally preferred not to be given via the oral route? A. Could potentially cause dangerous systemic hypotension B. Considerably less effective when given through this route C. Immediately inactivated by exposure to low gastric pH D. There is high first pass effect E. Food in the stomach significantly retards absorption

248 D. High first pass effect is the best answer, low bioavailability through the oral route due to liver metabolism

Which of the following is an inhalational anesthetic of choice for asthmatic patient because it causes the least bronchospasm? A. halothane B. desflurane C. sevoflurane D. thiopental E. propofol

249 A. Halothane generally is the inhaled agent of choice, sevoflurane is debatable, desflurane is pungent, the others are IV anesthetics

Zileuton is useful in the treatment of asthma because it A. Inhibits prostaglandin biosynthesis B. Inhibits leukotriene synthesis C. Inhibits leukotriene receptors D. Inhibits 12-lipoxygenase E. None of the above.

25 B. By inhibiting 5-lipoxygenase, zileuton reduces leukotriene biosynthesis; it does not inhibit (and in fact it might increase) prostaglandin synthesis.

Which of the following drugs given contains an effect which suppresses nausea and vomiting ? A. propanolol B. procarbazine C. chlorambucil D. chlorpromazine E. chloramphenicol

250 D. Chlorpromazine usually is used as a antipsychotic, but has antiemetic effects belonging to the phenothiazine class, along with other drugs used to control nausea vomiting like promethazine

Which of the following medications could be given to high risk or immunocompromised patients with RSV infections? A. Acyclovir B. Fosamprenavir C. Indinavir D. Ganciclovir E. Ribavirin

251 E Ribavirin - hepatitis C, viral hemorrhagic fever, and RSV Acyclovir and ganciclovir for herpes group Indinavir and fosamprenavir for HIV Amantadine and rimantidine for influenza A Zanamivir and oseltamivir for influenza B

Using drugs to block which of the following chemical mediators could potentially decrease leukocyte chemotaxis? A. prostaglandin H B. cyclooxygenase C. bradykinin D. leukotrienes E. serotonin

252 D. Leukotrienes could also serve as chemotactic factors

A patient is taking fenofibrate for elevated trigylceride levels, which of the following laboratory examinations should the doctor request to monitor for potential side effects upon follow up after 3 to 6 months? A. CBC with differential count B. urinalysis C. AST, ALT D. fasting blood glucose levels E. serum total cholesterol and triglycerides

253 C. Fibrates can potentially increase liver enzymes

Which of the following medications can be used as part of a regimen for migraine prophylaxis? A. ibuprofen B. acetaminophen C. sumatriptan D. propranolol E. dihydroergotamine

254 D. Prophylaxis of migraine: BBs (atenolol, timolol, propranolol) (best choice) Selective H2 antagonist (pizotifen, methysergide) Tricyclic antidepressants (amitriptyline) Anticonvulsants (sodium valproate, topiramate) Calcium channel blocker (flunarizine)

Which of the following medications can decrease conduction through the AV node? A. nifedipine B. verapamil C. lithium D. prazosin E. dypyridamole

255 B. Verapamil is a non-dihydropyridine calcium channel blocker which exerts its actions more on the heart decreasing AV nodal conduction

A 24 year old female is taking oral contraceptives as family planning method. Which of the following drugs is considered a CYP450 enzyme inducer which can cause rapid metabolism of other drugs such as oral contraceptives which can potentially decrease their effectiveness? A. isoniazid B. trimethoprim sulfamethoxazole C. griseofulvin D. Amiodarone E. ketoconazole

256 C. Griseofulvin is an inducer. All the others are inhibitors. Memorize at least the most common of them inducers and inhibitors. Topnotch handouts are good reference.

Which of the following chemotherapeutic agents can produce pulmonary fibrosis as toxicity? A. bleomycin B. doxorubicin C. irinotecan D. etoposide E. carmustine

257 A. Know the characteristic adverse effects and toxicities, the peculiar ones, the ones that make the drugs famous. Pulmonary fibrosis = bleomycin

Which among the following cardiac drugs can decrease preload, afterload and contractility? A. digoxin B. captopril C. amlodipine D. hydrochlorthiazide E. propranolol

258 E Know the mechanism and understand them. Know and master normal physiology. Digoxin is a negative chronotropic, but positive inotropic which increases contractility. Captopril inhibits ACE, acts on the RAAS and decreases preload by decreasing aldosterone and sodium water retention, hence decreasing blood volume and venous return. Anlodipine is a calcium antagonist, predominantly on the vessels which decrease afterload by dilating resistance vessels. Hydrochlorothiazide decreases preload by increasing secretion of both sodium and water. Propranolol, a non selective beta blocker, decreases renin secretion hence RAAS through B1 blockage. This includes angiotensin 2, so decreased both preload and afterload, decreases contractility through B1 blockade.

Which of the following antibiotics are highly protein bound which can act to displace other substances from albumin binding and increase free drug levels? A. chloramphenicol B. ciprofloxacin C. cotrimoxazole D. tetracycline E. gentamicin

259 C. Sulfa drugs are usually highly protein bound drugs which can displace other drugs making the free drug available to easily permeate cell membranes or bind to receptors. It can also displace bilirubin from albumin raising bilirubin levels in neonates

A 59-year-old female nurse who has been diagnosed with type 2 diabetes is admitted to the emergency room. She is tachycardic, tachypneic, and appears very disoriented; she does not remember the day of the week or her address or any emergency contact numbers. She vaguely remembers taking her "sugar medicine" earlier in the day. Which of the following drugs is most likely responsible for her condition? A. Metformin B. Acarbose C. Glipizide D. Glucagon E. Pioglitazone

26 C. Any of the sulfonylureas can cause hypoglycemia which can produce shock-like symptoms. Metformin and the glycosidase inhibitors such as acarbose rarely cause hypoglycemia. Glucagon would raise plasma glucose.

Which of the following drugs with mechanism of action that inhibits protein synthesis by inhibiting binding of the tRNA to the mRNA ribosome complex? A. tetracycline B. clindamycin C. gentamycin D. erythromycin E. chloramphenicol

260 A. SIMILAR TO PREVIOUS BOARD EXAM CONCEPT/PRINCIPLE, please know the mechanism of action of each drug class

If a single dose of a known drug with first order elimination is given intravenously, how long will it take for 75% of the drug to be eliminated if the half-life is 1.5 hours? A. 1.5 hours B. 2 hours C. 2.5 hours D. 3 hours E. 6 hours

261 D. Review first-order elimination. 50% is one half-life, 75% is 2 X half-life and so on..

Which of the following teratogenic effects is associated with valproic acid use in pregnancy? A. Craniofacial anomalies B. Neural tube defects C. Ebstein anomaly D. Fetal hydantoin syndome E. Microcephaly

262 B.A is carbamazepine; C is Lithium, D is phenytoin

You are assigned to go on duty as an intern of the toxicology department of the hospital. You recall that activated charcoal will NOT be effective adsorbing which of the following? A. Iron B. Theophylline C. Phenobarbital D. Amitryptiline E. Digoxin

263 A. Iron, lithium, cyanide, alcohol are poorly adsorbed by activated charcoal

Which of the following Cholinomimetic drugs are used exclusively in the treatment of Alzheimer's disease? A. Donepezil B. Physostigmine C. Rivastigmine D. A and B E. A and C

264 E Donepezil, Rivastigmine, Tacrine used exclusively in Alzheimer's

You prescribe a cancer patient who is undergoing chemotherapy with allopurinol to address hyperuricemia. Upon reviewing his chart, you realize caution is needed because of risk of toxicity. Which of the following chemotherapeutic drus is most likely being given to the patient? A. Azathioprine B. Methotrexate C. Fluorouracil D. Cytarabine E. All of the above

265 A. Azathioprine/Mercaptopurine metabolism is inhibited by allopurinol.

Adrenoreceptor blockers such as acebutolol and pindolol are unique for having intrinsic sympathetic activity. What is meant by this characteristic? A. has a membrane-stabilizing activity B. has partial agonist activity C. has receptor selectivity D. A and B E. All of the above

266 B.

Which of the following drugs that used in gout has been associated with cataract formation? A. Colchicine B. Allopurinol C. Probenecid D. Indomethacin E. Acetaminophen

267 B. SIMILAR TO PREVIOUS BOARD EXAM CONCEPT/PRINCIPLE

You are treating a patient HIV-patient for an opportunistic viral infection affecting the eyes. If you suspect viral strains that are thymidine-kinase deficient, you decide to choose an antiviral drug that is exclusively phosphorylated by host cell kinases but which doesn't require viral kinase phosphorylation. Which of the following will you choose? A. Acyclovir B. Ganciclovir C. Cidofovir D. Foscarnet E. Lamivudine

268 C. Cidofovir is phosphorylated by host kinases while Foscarnet doesn’t reqauire phosphorylation. Both are used in thymidine-deficient resistant strains of CMV

A 63 year old male patient presenting with intermittent complaints of chest pain upon walking several blocks or climbing several flights of stairs is prescribed with nitroglycerin to used on an as needed basis. What will be the expected physiologic changes as a result of this drug? A. Increased HR and and decreased left ventricular end-systolic volume B. Decreased HR and decreased left ventricular end-systolic volume C. Decreased HR and increased left ventricular end-systolic volume D. Increased HR and increased left ventricular end-systolic volume E. None of the above

269 A. Nitroglcerin decreases afterload, thus, decreasing end systolic volume. Compensatory sympathetic activity increases heart rate.

A 53-year-old woman with breast cancer undergoes a breast-conserving lumpectomy and lymph node biopsy. The pathology report returns with mention of cancer cells in two of eight lymph nodes removed. Following radiation therapy, chemotherapy is started that includes the use of paclitaxel. Which side effect is the patient likely to complain of? A. Blood in the urine B. Easy bruising C. Hot flashes D. Shortness of breath E. Numbness and tingling

27 E Paclitaxel is often used in the treatment of breast as well as ovarian and lung cancer. Its main toxicities are myelosuppression and peripheral neuropathy that usually manifest as numbness and tingling in the distal extremities. Blood in the urine can indicate hemorrhagic cystitis, a complication of cyclophosphamide use. Easy bruising can result from mechlorethamine use. Hot flashes are a common complaint in patients using tamoxifen. Shortness of breath can result from pulmonary fibrosis secondary to busulfan or bleomycin use.

Which of the following mechanism indicates high-level resistance in a TB patient being treated with Isoniazid? A. deletions in inhA gene that encodes target enzyme B. changes in drug sensitivity to RNA polymerase C. expression of drug efflux system D. expression of inactivating enzymes E. deletion in katG gene involved in the bioactivation of the drug

270 E E is INH high level resistance; A is INH low level resistance; B is rifampicin resistance

A drug with diuretic activity is being studied based on its effects on electrolyte levels in the urine. It was found to moderately increase urine NaCl and urine K while decreasing urine Ca with associated slight increase in body pH and minimal change in urine HCO3-. This drug has characteristics similar to that of? A. Loop diuretic B. Carbonic anhydrase inhibitor C. K-sparing diuretic D. Thiazide diuretic E. Osmotic diuretic

271 D. Thiazide increases Na and K excretion and causes metabolic alkalosis. It increases Ca reabsorption

Which of the following steroid antagonists inhibits cytochrome P450 enxymes necessary for all steroids? A. Ketoconazole B. Aminogluthetimide C. Metyrapone D. Spinorolactone E. Mifepristone

272 A. Ketoconazole is a potent cytochrome p450 inhibitor. Aminoglutethimide inhibits conversion of cholesterol to pregnenolone. Metyrapone inhibits cortisol synthesis. Spinorolactone inhibits aldosterone. Mifepristone inhibits progesterone

A patient with warfarin as a maintenance medication is treated for pneumonia. Prothrombin time monitoring during treatment of the respiratory infection reveals elevated INR but it decrease back to baseline levels after treatment with the antibiotic is completed. Which of the following is most likely given? A. Amoxicillin B. Ceftriaxone C. Clindamycin D. Erythromycin E. TMP-SMX

273 D. Erythromycin inhibits metabolism of warfarin causing an increase In INR

A COPD patient with severe concomitant heart condition is being prescribed a drug that will relieve his dyspnea and shortness of breath. Which of the following bronchodilators will be the primary choice? A. Salmeterol B. Salbutamol C. Terbutaline D. Atropine E. Ipratropium

274 E Ipratoropium is an anticholinergic used in COPD because of less cardiac effects

A physician prescribes a diabetic patient with an antidiabetic agent that activates a nuclear receptor that increases transcription of GLUT-4 transporters in adipose tissue. The drug prescribed is most likely: A. Methotrexate B. Insulin C. Pioglitazone D. Glyburide E. Sitagliptin

275 C. Thiozolidinediones - activates PPAR, a nuclear receptor to improve sensitivity to insulin

A patient presented to your clinic with complaints of dyspnea and chronic cough. He states that he frequently gets short of breath and is unable to take a deep breath. History is significant for prolonged treatment of rheumatoid arthritis. Chest X-ray revealed fine reticulonodular densities. Which drug is most likely responsible? A. Glucocorticoids B. Methotrexate C. Infliximab D. Cyclosporine E. Anakinra

276 B. Methotrexate is known to cause pulmonary fibrosis

If you need to give a skeletal muscle relaxant to a burn patient, you will AVOID prescribing which of the following drugs? A. Pancuronium B. Baclofen C. Dantrolene D. Vecuronium E. Succinylcholine

277 E Succinylcholine is contraindicated in patient with rhabdomyolysis, hyperkalemia, burn patients, malignant hyperthermia

Which of the following medications exert its anticoagulant effect through the inactivation of thrombin? A. Fondaparinux B. Unfractionated heparin C. Low molecular weight heparin D. Apixaban E. All of the above

278 B. Fondaparinux and LMW heparin combines with antithrombin and selectively inhibits factor X but NOT thrombin. Apixaban is a direct Xa inhibitor

One of the following drugs is effective in reducing the risk of ulcers in chronic NSAID users. Diarrhea is a frequent side effect. Which is it? A. Sucralfate B. Ranitidine C. Omeprazole D. Misoprostol E. Cimetidine

279 D.

A 42-year-old MDS patient presents to the emergency room with mental status changes and a headache. A computed tomography scan is ordered and demonstrates a ring-enhancing lesion. You decide to treat him empirically due to the possibility of Toxoplasmosis gondii abscess. Which agent should be included in his treatment? A. Ivermectin B. Praziquantel C. Sulfadiazine D. Niclosamide E. Pyrantel pamoate

28 C. Toxoplasmosis is treated with a combination of pyrimethamine and sulfadiazine. Ivermectin is used to treat filariasis, whereas praziquantel is used to treat schistosomiasis. Niclosamide can be used to treat tapeworm infections. Pyrantel pamoate is used to treat many helminth infections.

Knowledge of potency and solubility is important in general anesthetic medications. Which of the following characteristics refers to high potency? A. Low minimum alveolar concentration B. High minimum alveolar concentration C. Low blood:gas partition coefficient D. High blood:gas partition coefficient E. None of the above

280 A. Low MAC - high potency; High MAC - low potency; Low blood:gas coefficient - low solubility; High blood:gas coeeficient - high solubility

Which of the following anti hypertensive medications when taken during pregnancy can cause hypocalvaria in the fetus? A. Carvedilol B. Amlodipine C. Captopril D. HCTZ E. Hydralazine

281 C. Intake of ACE inhibitors can produce teratogenic effect in the fetus which includes renal dysgenesis and hypoplastic skulls (hypocalvaria) thus it is contraindicated during pregnancy

A 29 year old woman presented with amenorrhea, galactorrhea and loss of sexual libido. On evaluation her serum prolactin was noted to be markedly elevated. Which of the following medications is most useful in the treatment of her condition? A. Bromocriptine B. Cimetidine C. Sumatriptan D. Ergotamine E. Ondansetron

282 A. Bromocriptine is an effective dopamine agonist in the CNS with the advantage of oral activity. The drug inhibits prolactin secretion by activating pituitary dopamine receptors

A 47 year old obese male, who is a heavy alcoholic beverage drinker, was recently diagnosed with diabetes mellitus. He was started on Metformin 500 mg/tab TID. This patient is at risk of developing which of the following complications? A. Hypoglycemia B. Disulfiram like reaction C. Congestive heart failure D. Lactic acidosis E. Diarrhea

283 D. Metformin is associated with lactic acidosis. It should be avoided or used with extreme caution in patients who are heavy alcoholics since acute ethanol ingestion increaes the risk of lactic acidosis

A 65 year old female presented to the ER due to progressive dyspnea. She is a known hypertesive but is poorly compliant with medications. On history, patient claims to experience orthopnea, paroxysmal nocturnal dyspnea and easy fatigability. On PE, her BP is 80/50. There is prominent neck vein distention. S3 gallop, bibasal crackle and a grade 3 bipedal edema were also appreciated. A 2d echo was done which showed a depressed ejection fraction of 32%. Which of the following medications should not be given at this time for this patient ? A. Furosemide B. Digoxin C. Metoprolol D. Dobutamine E. None of the above

284 C. This is a case of an acute decompensated heart failure. Due to the marked systolic dysfunction as evident by a depressed ejection fraction, beta blockers are contraindicated at this time as it may further lower the cardiac output.

A 75 year old male was brought to the ER due to sudden onset of right sided weakness accompanied by a progressive deterioration in sensorium. On PE, BP was elevated at 220/100. Neurologic exam showed a GCS of 11, an MMT of 2/5 on the both right upper and lower extremities and a positive babinski on the right. A CT scan showed a massive intracerebral hemorrhage involving the left basal ganglia. A diuretic was started to decrease the patient's intracranial pressure. Which of the following is an associated adverse effect of this medication ? A. Ototoxicity B. Hyperlipidemia C. Gynecomastia D. Pulmonary Edema E. Thrombocytosis

285 D. Mannitol is used to decrease the ICP in patient's with hemorrhagic stroke. It acts as an osmotic diuretic thus removes water from the intracellular compartment. This rapid fluid shifting may cause hyponatremia and pulmonary edema and as the water is excreted, hypernatremia may follow.

Which of the following is an orally active direct thrombin inhibitor? A. Apixaban B. Rivaroxaban C. Enoxaparin D. Fondaparinux E. Dabigatran

286 E Apixaban and Rivaroxaban are both oral Factor Xa inhibitors. Enoxaparin and Fondaparinux are both SC factor Xa inhibior.

A 48 year old female, a known case of Acute Myelogenous Leukemia (AML), who is on reinduction chemotherapy was started on anti viral prophylaxis with Valacyclovir 500 mg/tab TID. What is the mechanism of action of this anti viral medication? A. Inhibition of Viral DNA polymerase B. Blockage of M2 proton channel C. Inhibition of neuraminidase D. Inhibition of viral reverse transcriptase E. Prevents fusion of virus with the host cellular membrane

287 A. Valaciclovir is a prodrug, an esterified version of aciclovir that has greater oral bioavailability (about 55%) than aciclovir (10â€"20%). It is phophorylated three times and this form acts as a competitive substrate for DNA polymerase and it leads to chain termination

Which of the following drugs of abuse is non addictive? A. Amphetamine B. Cocaine C. Morphine D. LSD E. Heroin

288 D. Some drugs of abuse do not lead to addiction. This is the case for substances that alter perception without causing sensation of reward and euphoria. These include LSD, Phencyclidine and ketamine

A 70 year old male who is diagnosed with Colon Cancer Stage IV with metastasis to the liver who is currently on FOLFOX regimen was started on a monoclonal antibody which binds to the vascular endothelial growth factor (VEGF) receptor. What is the medication that was given to the patient? A. Imatinib B. Sorafenib C. Trastuzumab D. Bevacizumab E. Erlotinib

289 D. Imatinib is a tyosine kinase inhibitor used in CML. Sorafenib inhibit multiple tyrosne kinase receptors and are primarily used in HCC. Trastuzumab inhibits cells that overexpress the Her-2/neu recepto in Breast CA. Erlotinib inhibits epidermal growth factor receptor and is used for non small cell lung CA and pancreatic cancer

A 74-year-old man with a 100-pack/year history of smoking is evaluated for hemoptysis. A computed tomography (CT) scan of the chest shows numerous pulmonary nodules. A nodule on the pleural surface is selected for CT-guided biopsy by the interventional radiologist. The biopsy report is small-cell carcinoma of the lung, and chemotherapy containing etoposide is started. This drug works by A. Inhibiting topoisomerase II B. Inhibiting dihydrofolate reductase C. Alkylating double-stranded DNA D. Stabilizing microtubules, with resultant mitotic arrest E. Causing DNA chain scission and fragmentation

29 A. Etoposide is used in the treatment of small-cell lung carcinomas as well as testicular tumors. Its mechanism of action is related to its ability to inhibit topoisomerase II. Methotrexate inhibits dihydrofolate reductase. Alkylating agents include mechlorethamine, cyclophosphamide, and ifosfamide. Paclitaxel and docetaxel stabilize microtubules and thereby disrupt mitosis. Bleomycin causes DNA chain scission and fragmentation.

Which of the following medications can be given once a week for prophylaxis against bactermia caused by M. avium complex (MAC) in AIDS patients? A. Azithromycin B. Kanamycin C. Ethambutol D. Rifampicin E. Cycloserine

290 A. because of its long elimination half life (3-4 days), weekly administration of azithromycin has proved to be equivalent to daily administration of clarithromycin when used for prophylaxis against MAC in AIDS

A 37 year old female diagnosed with chronic schizophrenia maintained on an antipsychotic medication was brought to the clinic due to a 3 day history of fever. PE showed presence of tonsillar exudates. A CBC was requested which showed a markedly decreased WBC count. The medication was immediately discontinued by the patient's physician. What is the anti-psychotic medication that could have caused this problem? A. Haloperidol B. Quetiapine C. Clozapine D. Risperidone E. Olanzapine

291 C. Clozapine can cause agranulocytosis in a small but significant number of patients (1-2%). This serious, potentially fatal effect can develop rapidly usually between the 6th to 18th week of treatment. It appears to be reversible upon discontinuation of treatment. Because of this risk, patient on clozapine must have weekly blood counts for the first 6 months of treatment and every 3 weeks thereafter

What is the major advantage of Etomidate over other intravenous anesthetics? A. Has greater analgesic effects B. Causes minimal cardiovascular and respiratory depression C. Relatively longer duration of effect D. Has a lower incidence of postoperative nausea and vomiting E. Has a rapid onset of recovery when compared with Propofol

292 B. Etomidate has no analgesic effects. Distribution is rapid from the brain to highly perfused tissue hence a relatively short duration of its anesthetic effect. It has a high incidence of postoperative nausea and vomiting, pain and myoclonic activity. Initial recovery is less rapid compared with propofol

A 24 year old female G1P0, 12 weeks AOG, sought consult due to purulent vaginal discharge and dyspareunia. Gram stain of the vaginal discharge showed intracellular gram negative diplococci. The patient disclosed that she had a severe allergic reaction to amoxicillin a year ago. Which of the following is the safest antibiotic for this case ? A. Doxycyline B. Ciprofloxacin C. Ceftriaxone D. Gentamicin E. Azithromycin

293 E Cephalosporins should be avoided in patients with history of severe allergic reaction to penicillin. FQ and doxycycline are avoided in pregnancy since it can cause cartilage and bone damage. Azithromycin, a macrolide antibiotic is the safest drug for this case.

Which of the following is a major adverse effect associated with use of fenofibrates? A. Constipation B. Gallstones C. Hyperuricemia D. Cardiac arrythmia E. Liver damage

294 B. SIMILAR TO PREVIOUS BOARD EXAM CONCEPT/PRINCIPLE. A major toxicity of fenofibrates is increased risk of gallstone which may be due to enhanced biliary excretion

A 45 year old male presented to the ER due to sudden onset of right eye pain accompanied by blurring of vision. On examination there was note of ciliary injection over the right eye, IOP pressure was noted to be elevated. A diagnosis of acute angle closure glaucoma was made. Which of the following ophthalmic medications should be avoided ? A. Brinzolamide B. Timolol C. Apraclonidine D. Atropine E. All of the above

295 D. Atropine is a mydriatic and cyloplegic and can further increase IOP hence worsen angle closure glaucoma.

Which of the following beta blockers possess intrinsic sympathomimetic activity? A. Metoprolol B. Esmolol C. Pindolol D. Acebutolol E. Both C and D

296 E Acebutolol and Pindolol have partial agonist activity (ISA)

Which of the following drugs is used to induce ovulation in anovulatory women by selectively blocking estrogen receptors in the pituitary, thus increasing FSH and LH output? A. Raloxifene B. Clomiphene C. Fulvestrant D. Anastrozole E. Progestins

297 B. SIMILAR TO PREVIOUS BOARD EXAM CONCEPT/PRINCIPLE

A 39 year old female, known case of Graves' disease but poorly compliant with medications, presented to the ER due to palpitations and shortness of breath. She was hooked to a cardiac monitor which showed supraventricular tachycardia. Which of the following drugs would be most suitable for this case ? A. Esmolol B. Quinidine C. Flecainide D. Disopyramide E. Lidocaine

298 A. beta blockers are the most effective agents in acute thyrotixc arrythmias . Esmolo is a rapid parenteral beta blocker

A 67 year old woman, known diabetic for 20 years and maintained on Gliclazide, complained of severe bloating and post prandial fullness. Evaluation was done and she was assessed to have diabetic gastropathy. Which of the following medications would be most appropriate for this case? A. Cimetidine B. Metoclopramide C. Alosetron D. Loperamide E. Bismuth subsalicylate

299 B. Metoclopramide is DOC for diabetic gastropathy

A 49 year old made came to the ER with a 3 hour history of anginal chest pain. Past medical history is significant only for uncontrolled hypertension. Cardiac markers and ECG both confirm the diagnosis of acute myocardial infarction probably of the anterior wall. Currently the patient has stable vital signs within normal range with persistence of angina. The following should be given except: A. Lisinopril B. Simvastatin C. Ibuprofen D. Metoprolol E. None of the above. All should be administered.

3 C. Other than Aspirin, NSAIDs are absolutely contraindicated to patients with acute MI. Drugs which are administered post MI include beta- blockers (unless with contraindications), ACE inhibitors (beneficial in preventing cardiac remodeling), Statins (stabilizes the atheromatous plaque), antiplatelet therapy like aspirin or clopidogrel, aldosterone antagonist (epleronone only), nitrates (increase O2 supply).

A young couple present to their primary care physician stating that they are trying to conceive. They would like to know if the future mom-to-be needs to be on any supplements. Along with recommending a multivitamin with folic acid, the doctor also suggests an iron supplement. Pregnant women develop iron deficiency anemia because of A. Increased bleeding tendency B. Increased dietary deficiency C. Malabsorption D. Increased iron demands E. Increased excretion

30 D. Pregnancy and lactation are the states of increased iron demands. While increased bleeding tendency, dietary deficiency, and malabsorption are all true causes of iron deficiency anemia, they are not the culprits during pregnancy. Iron storage is regulated at the level of absorption, and very little of it is lost from the body.

Which drug when taken together with Azathioprine has resulted in severe myelosupression by inhibiting the enzyme responsible for its metabolism ? A. Theophylline B. Celecoxib C. Allopurinol D. Digoxin E. Rifampicin

300 C. Azathioprine is converted to mercaptopurine which is responsible for its immunosuppressant and hematotoxicity. Allopurinol inhibits xanthine oxidase, the enzyme that metabolizes mercaptopurine.

During anaphylaxis, administration of IM epinephrine is essential because of its bronchodilatory effect in the lungs. How does epinephrine acts to produce such an effect in this particular condition? A. Chemical antagonist B. physiologic antagonist C. partial agonist D. noncompetitive antagonist E. none of the above

301 B. physiologic antagonist binds to a different receptor molecules producing an effect opposite to that produced by the drug it antagoniszes. It differs from a pharmacologic antagonist, which interacts with the same receptors as the drug it is inhibiting. the antagonism of bronchoconstrictor effect of histamine ( through histamine receptors) by epinephrine bronchodilator action ( beta receptors). chemical antagonist - reacts directly with the drug being antagonized to remove it or to prevent it from reaching its target. Katzung review 8th ed p 13.

which of the following drugs exhibit zero order kinetics ? A. Ethanol B. tolbutamide C. warfarin D. heparin E. All of the above

302 E zero order kinetics refers to a contant rate of elimination of a drug regardless of its concentration. The following drugs exhibit zero order kinetics: aspirin, ethanol, warfarin,heparin, penytoin, tolbutamide, theophyline phenylbutazone, and salicylates.

The following drugs promote induction of liver CYP enzymes except ? A. Rifampin B. Carbamazepine C. Phenylbutazone D. Disulfiram E. Glutethimide

303 D. CYP inducers enhances drug metabolism. Ex. Benzopyrene, carbamazepine, chlorcyclizine, glutethimide, griseofulvin, phenobarbital, phenylbutazone, phenytoim, rifampin, ritonavir ( chronic administration, acute -acts as inhibitor), st johns wort. Katzung 11th ed p 64-65.

which of the following effects in parathion poisoning will not be antagonized by timely administration of pralidoxime? A. cyclospasm B. Urinary incontinence C. convulsion D. Negative dromotrophy E. None of the above

304 C. pralidoxime is a chemical antagonist acting as cholinesterase regenerator. It cannot enter the BBB because of its positively charged group and poor lipid solubility. Katzung review pp 69.

Among the beta adrenergic blockers, which of the following will less likely to cause bradycardia and increased VLDL concentration? A. metoprolol B. nebivolol C. carteolol D. labetalol E. None of the above

305 C. pindolol, acebutolol, carteolol, bopindolol, penbutolol,celiprolol are the beta blockers having also partial b agonist activity ( intrinsic sympathomimetic activity). The advantage of this property is that it is less likely to cause elevation of plasma lipids without affecting its antihypertensive effect. Katzung 11th ed p 159.

which of the following changes will promote digitalis induced cardiototoxicity? A. Acute tubular necrosis B. hypermagnesemia C. hyperparathyroidism D. Triamterene administration E. All of the above

306 C. any condition that can predisposed to hyperkalemia, hypermagnesemia and hypocalcemia will inhibit the binding of digitalis to Na K ATPase resulting to decrease toxicity. Triamterene and ATN can induce hyperkalemia. Katzung 11th ed p 216.

which of the following drugs is highly selective to ischemic or prolonged depolarized purkinje fibers? A. Mexiletine B. Procainamide C. Propafenone D. ibutilide E. None of the above

307 A. phenytoin, lidocaine and mexiletine are classified as class IB antiarrhythmics. This group affects primarily ischemic or depolarized ventricular tissue and does not have any use in atrial arrhytmia. B- procainamide is a class IA drug, C- propafenone is a Class IB agent, D- ibutilide is class III drugs.

which of the following effects of acetazolamide is self limiting? A. Sodium reabsorption B. Tubular bicarbonate excretion C. CSF acidosis D. Aqueous humor production E. None of the above

308 B. the major effect of acetazolamide is bicarbonate diuresis resulting to metabolic acidosis. As increased sodium is presented to the cortical collecting tubule, some of the excess sodium is reabsorbed and potassium is secreted resulting to significant potassium wasting. as a result of bicarbonate depletion, sodium bicarbonate excretion slows even with continued administration. Katzung review 8th ed p 135

A 39 y/o male went for follow up consult secondary to open angle glaucoma. He was compliant with all the topical medications given to him. Indirect ophthalmoscopy revealed brownish discoloration in his iris. What is the mechanism of action of the drug that causes such change?. A. decrease aqueous secretion due to lack of HCO3 B. increase outflow via uveoscleral veins C. increase aqueous outflow D. decrease aqueous secretion from the ciliary epithelium E. none of the above

309 C. dermanent discoloration of the iris and eyelashes is a well known side effect of latanoprost. Its mechanism of action is to increase aqueous outflow. A- brinzolamide, dorsolamide, B- epinephrine. D- timolol. Katzung review p 87.

Ephedrine can cause increased blood pressure by A. Indirect action on cholinergic receptors B. Blockade of adrenergic receptors C. Stimulation of release of epinephrine D. Inhibition of reuptake of catecholamines E. Direct action on dopamine receptors

31 C. Ephedrine acts indirectly to release norepinephrine from nerve terminals, causing effects similar to those of catecholamines, including elevated blood pressure. This potentially dangerous agent has been removed from the OTC market because of an increasing number of deaths being reported as caused by this agent. An example of an indirect- acting cholinergic agonist is edrophonium, which is used for diagnosis of myasthenia gravis. Some adrenoceptor blockers, such as atenolol, are used for the treatment of hypertension. Catecholamine reuptake inhibition is a property of some antidepressant medications. Dopamine receptor agonists are used in the treatment of Parkinson disease.

which in the following antipsychotic medications has the greatest risk of inducing Torsades de pointes arryhthmia? A. Fluphenazine B. Clozapine C. Thioridazine D. Ziprasidone E. Haloperidol

310 D. Olanzapine -weight gain, clozapine - agranulocytosis, haloperidol- extrapyramidal symptoms. Thioridazine - retinal deposits. Among the atypical antipsychotics, ziprasidone has the greatest risk of inducing QT prolongation leading to ventricular arrhythmias. Kazung review p 242.

Which of the following is true about Buspirone? A. Anxiolytic effect is secondary to direct stimulation of GABAergic neurons B. It is highly effect in acute panic attacks C. Less psychomotor retardation D. Abrupt discontinuation can lead acute withdrawal syndrome E. all of the above

311 C. Buspirone is a selective anxiolytic medication that does not cause, sedative, hypnotic, euphoric, and acute withdrawal or rebound effect. It has no effect in psychomotor skills (driving skills). The mechanism of anxiolysis is secondary to partial antagonism to 5HT1A receptors in the brain. it has no direct GABAergic activity. weeks are needed before buspirone takes effect, therefore, it is not effect in treating acute panic attacks. Katzung 11ed p374.

what is the recommended treatment for a 29 y/o G1PO female diagnosed with Gonnorrhea infection who has severe hypersensitivity to pennicilin? A. erythromycin B. Spectinomycin C. Cetriaxone D. Doxycycline E. Clindamycin

312 B. single dose administration of spectinomycin is effective in treating Gonnorhea in a patient who has severe beta lactam hypersensitivity. It has lesser risk of teratogenicity compared to conventional aminglycosides. Clindamycin is active only on gram positive and anaerobic organisms. ceftriaxone is a 3rd generation cephalosporin effective against Gonorrhea infection however it should be avoided since the patient has beta lactam hypersensitivity. Doxycycline should never be given for pregnant patients. erythromycin is not effective treatment for gonorrhea infection. Katzung 11th ed p 813.

the pungency of this inhaled anesthetics leading breath-holding limits it use in anesthesia induction? A. desflurane B. Isoflurane C. Sevoflurane D. enflurane E. Halothane

313 D. (SIMILAR TO PREVIOUS BOARD EXAM CONCEPT/PRINCIPLE) Katzung review 8th ed p 211

what is the drug of choice for restless leg syndrome? A. haloperidol B. aripriprazole C. Citalopram D. Ropinirole E. Propanolol

314 D. the pathologic basis for restless leg syndrome is dopamine blockade. Agonist to dopamine receptors such as pramipexole or ropinirole is very effective in treating this condition. Katzung review 8th ed p 235

Prolonged treatment with a PPAR alpha ligand in the treatment of hyperlipidemia will increase the risk for the following condition? A. Cutaneous flushing B. Hyperuricemia leading to Gout C. Intestinal malabsorption D. Gallstones E. hepatotoxicity

315 D. PPAR alpha ligands is the MOA of fibric acid derivatives ( Gemfibrozil and fenofibrate). It increases the supersaturation of bile leading cholelithiasis. Risk for hepatotoxicity only significantly increase if gemfibrozil is combined with HMG CoA inhibitor. Cutaneous flushing and hyperuricemia are toxicity profile of Niacin. Intestinal malabsorption is a primary side effect of resins ( cholestyramine, colestipol, colsevelam)

oral administration of this antibiotic will yield greater blood levels compared when it is administered intravenously? A. chlorampenicol B. Quinipristin -dalfopristin C. Azithromycin D. Telithromycin E. None of the above

316 A. the pharmacokinetics of chlorampenicol is unusual since blood levels are higher when the drug is given at oral form rather than intravenous form. Katzung 11th ed page 802

what is the mechanism of action of nystatin? A. Formation of artificial pore in the fungal membrane B. Inhibit ergosterol synthesis C. Inhibit microtubular formation D. Inhibits squalene epoxidase E. Inhibit the synthesis of B(1-2) glycan

317 A. nystatin and ampothericin B are both polyene antifungal antibiotics. It binds to ergosterol and creates an artificial pores in the fungal membrane leading to increase membrane permeability. Katzung review 8th ed p 401

the drug of choice for Chagas disease acts through what mechanism of action? A. Inhibit mitochondrial electron transport B. Trypanothione reductase inhibition C. Suicide substrate of ornithine decarboxylase D. Inhibits enzyyme sulfhydryl groups E. none of the above

318 B. the drug of choice for Chagas disease or american trypanosomiasis is nifurtimox. Its anitprotozoan action is due to inhibition of trypanothiane reductase. A- atovaquone MOA ( use for PCP and chloroquine resistant malaria) C- eflornithine ( use in cerebral stage of african trypanosomiasis). D- melarsoprol ( african sleeping sickness). SIMILAR TO PREVIOUS BOARD EXAM CONCEPT/PRINCIPLE Katzung review p 438-439.

what is the primary drug of treatment for ascariasis, pinworm, whipworm and hydatid disease? A. albendazole B. Mebendazole C. Pyrantel pamoate D. Diethylcarbamazine E. All of the above

319 A. B,C,D has no clinical effectivity in hydatid disease.

Which of the following is a potential side effect of clozapine? A. Cholestatic jaundice B. QT prolongation C. Agranulocytosis D. Photosensitivity E. Galactorrhea

32 C. Agranulocytosis occurs more frequently with clozapine than with other agents, requiring routine blood tests. It is the only agent that improves the negative symptoms of schizophrenia. Cholestatic jaundice and photosensitivity are common with chlorpromazine. Galactorrhea is a side effect of older high-potency agents that block dopamine. QT prolongation is a complication of agents such as thioridazine and ziprasidone.

what is the dose limiting toxicity of vincristine ? A. areflexia B. Bone marrow suppression C. Acute tubular necrosis D. Hepatocyte degeneration E. None of the above

320 A. vincristine does not cause serious myelosuppresive effect however it induces neurotoxicity such as areflexia, ileus and peripheral neuritis. Katzung review 8th ed p 455.

Among insulin secretagogues, the risk of hypoglycemia is least in? A. Meglitinides B. 1st generation sulfonylureas C. 2nd generation sulfonylureas D. Biguanides

321 A. Biguanides are not insulin secretagogues.

What is the most important drug in pulmonary tuberculosis? A. INH B. Pyrazinamide C. Rifampin D. Ethambutol

322 A.

Pupillary construction is a characteristic effet of all opioids, except: A. Oxycodone B. Meperidine C. Morphine D. Fentanyl

323 B.

This antiplatelet inhibits aggregation by interfering with GpIIb/IIIa binding to fibrinogen and other ligands: A. Aspirin B. Clopidogrel C. Dipyridamole D. Tirofiban

324 D. Aspirin is a nonselective irreversible COX inhibitor. Clopidogrel irreversibly inhibits platelet ADP receptor. Dipyridamole inhibits adenosine uptake and inhibits phosphodiesterase enzymes that degrade cyclic nucleotides.

Which tetracycline has the broadest spectrum of microbial coverage? A. Tetracyline B. Doxycyline C. Minocycline D. Tigecycline

325 D.

The following drugs' efficacy is directly related to time above the minimal inhibitory concentration and becomes independent of concentration once the MIC has been reached, except? A. Penicillins B. Cephalosporins C. A & B D. none of the above

326 D. What is described is the time-dependent killing property for which penicillins and cephalosporins are known. Aminoglycosides are known for their concentration-dependent killing.

The following are bactericidal, except: A. TMP-SXZ B. Vancomycin C. Metronidazole D. Cefalexin E. none of the above

327 E All drugs stated are bactericidal. TMP and SXZ alone are bacteriostatic but are bactericidal when in combination.

The following anti-tuberculosis drugs are correctly paired with their mechanisms of action, except: A. INH - inhibits mycolic acid synthesis B. Rifampin - inhibits RNA dependent DNA polymerase C. Ethambutol - inhibits arabinosyl transferase D. none of the above

328 B. Rifampicin inhibits DNA dependent RNA polymerase

This drug requires phosphorylation by host cell kinases to be active: A. Efavirenz B. Delavirdine C. Nevirapine D. Tenofovir

329 D. Efavirenz, delaverdine, and nevirapine are non nucleoside reverse transcriptase inhibitors which do not require host cell kinase for activation, unlike tenofovir, a nucleoside reverse transcriptase inhibitor which does.

Which of the following is a good choice to treat newly diagnosed generalized anxiety disorder in a truck driver? A. Alprazolam B. Triazolam C. Buspirone D. Trazodone E. Thiopental

33 C. Buspirone is a partial serotonin 5-HT1A-receptor agonist that has efficacy comparable to that of benzodiazepines for the treatment of anxiety, but is significantly less sedating. Alprazolam is an intermediate-acting benzodiazepine used in the treatment of generalized anxiety disorder (GAD) but still has some sedation, which would be undesirable in this situation. Triazolam is a short-acting benzodiazepine, and trazodone is a heterocyclic antidepressant, both used to induce sleep. Thiopental is a barbiturate sometimes used to induce anesthesia.

This antihistamine causes depolarization-induced paralysis in nematodes: A. Mebendazole B. Ivermectin C. Pyrantel pamoate D. Piperazine

330 C. Mebendazole selectively inhibits microtubule synthesis and glucose uptake. Ivermectin interferes with GABA mediated neurotransmission. Piperazine is an agonist at GABA receptors.

Which of the following is uricosuric? A. Allopurinol B. Colchicine C. Sulfinpyrazone D. Celecoxib

331 C. Allopurinol is a xanthine oxidase inhibitor. Colchicine inhibits microtubule assembly. Celecoxib is a COX-2 inhibitor.

Which drug is most effective in raising HDL? A. Niacin B. Fibrates C. Statins D. Bile acid sequestrants

332 A.

The metabolism of which drug is inhibited by allopurinol and febuxostat? A. Cytarabine B. Methotrexate C. Fluorouracil D. Mercaptopurine

333 D. Allopurinol inhibits the enzymatic inactivation of 6- MP and its derivative, Azathioprine, by xanthine oxidase. Thus, when allopurinol is used concurrently with oral 6-MP/azathioprine, dosage of the antineoplastic agent must be reduced to 1/2 to 1/3 of usual dose.

This regimen is utilized in the treatment of Hodgkin lyphoma: A. PEB regimen B. ABVD regimen C. CHOP regimen D. GnRH agonist and androgen receptor antagonist

334 B. ABVD - adriamycin, bleomycin, vincristine, dacarbazine, prednisone. PEB (cisplatin, etoposide, bleomycin) regimen is used in testicular CA. CHOP (cyclophosphamide, doxorubicin, vincristine, prednisone) regimen in non-Hodgkin lymphoma, and GnRH agonist and androgen receptor antagonist in prostate CA treatment.

This drug is a monoclonal antibody that binds to VEGF and prevents it from interacting with VEGF receptors: A. Bevacuzimab B. Rituximab C. Trastuzumab D. Imatinib

335 A. Rituximab is a monoclonal antibody that binds to surface protein in NHL inducing lysis, cytotoxicity and apoptosis. Trastuzumab is a monoclonal antibody that recognizes protein in breast CA cells overexpressing HER-2neu receptor for EGF. Imatinib inhibits tyrosine kinase activity of the product of bcr-abl oncogene expressed in CML.

This chemotherapeutic agent is known for its association with heart toxicity in cumulative doses: A. 5-FU B. 6-MP C. Methotrexate D. Doxorubicin

336 D.

The following laxative mechanisms are correctly paired with its representative laxative drug, except: A. bulk forming - psyllium B. stool softening - docusate C. osmotic - lactulose D. stimulant - polyethylene glycol

337 D. Polyethylene glycol is an osmotic laxative. Examples of stimulant laxatives include senna, bisacodyl, castor oil.

Ondansetron, highly effective in preventing chemotherapy-induced nausea and vomiting, is an antagonist at which receptor? A. muscarinic B. B2 C. 5-HT3 D. D2

338 C.

This drug stimulates platelet production and decreases the number of platelet transfusions required by patients undergoing bone marrow suppression in treatment for CA: A. Oprevelkin B. Filgrastim C. Erythropoietin D. Folic acid

339 A. Filgrastim stimulates the function and production of neutrophils.

A 17-year-old girl sees her physician for swollen lymph nodes in the supraclavicular region. A core biopsy demonstrates Reed-Sternberg cells and fibrotic bands, a finding characteristic of nodular sclerosis Hodgkin disease. Which of the following combined regimens might be used in this patient? A. R-CHOP B. CMF C. FOLFOX D. BEP E. ABVD

34 E ABVD is a treatment regimen used for Hodgkin disease and includes adriamycin, bleomycin, vinblastine, and dacarbazine. R-CHOP is used for treating non-Hodgkin's disease. CMF, or cyclophosphamide, methotrexate, and fluorouracil, is used for breast cancer. FOLFOX, a regimen that uses 5-fluorouracil, oxaliplatin, and leucovorin, is used in the treatment of colon cancer. BEP (bleomycin, etoposide, and platinum [cisplatin]) is used in the management of metastatic testicular neoplasms.

Serum concentration of lithium in mEq/L favored for acutely manic patients is: A. 0.9-1.1 B. 0.6-0.75 C. 0.5-0.85 D. 0.1-0.5

340 A. The serum concentration favored for long-term use in the prevention of reccurent manic-depressive illness, on the other hand, is 0.6-0.75 mEq/L.

This direct acting cholinomimetic is used in the treatment of postoperative ileum and neurogenic bladder: A. Neostigmine B. Bethanechol C. Pilocarpine D. Physostigmine

341 B.

Use of aspirin as an anti-platelet inhibits primarily which prostanoid? A. Prostacyclin B. Thromboxane C. Prostaglandin D. Epoprosterenol

342 B.

This is not a characteristic of phase I clinical trials in drug development: A. Pharmacokinetic measurements are done B. Determination of drug’s efficacy C. Determination of maximum tolerated dose D. Subjects are healthy volunteers

343 B. Determination of drug efficacy is done during phase II of clinical trials.

The prime target of addictive drugs in the brain: A. Thalamocortical system B. Nigrostriatal dopamine system C. Mesolimbic dopamine system D. Mesocortical dopamine system

344 C. Reference: Katzung. Basic and Clinical Pharmacology, 11th ed. p. 554

A 30 year old female with bipolar disorder delivered a baby with Ebstein anomaly. The drug that the mother was taking during her pregnancy that caused this congenital anomaly is: A. Olanzapine B. Lithium C. Carbamazepine D. Risperidone

345 B.

Ketamine exerts its anaesthetic effect by its inhibitory action on these receptors: A. NMDA B. GABA-A C. Glycine D. Neuronal nictonic acetylcholine receptors

346 A.

A 20 year old male develops tachycardia, hypotension, muscle rigidity, and spiking fever immediately after start of surgery. Which of the following are commonly associated with this anaesthetic complication? A. IV anesthetic and succinylcholine B. IV anesthetic and midazolam C. Inhalational anesthetic and succinylcholine D. Inhalational anesthetic and midazolam

347 C. This is a case of malignant hyperthermia, which is associated with the combination of inhaled anaesthetics and succinylcholine.

Nitrates relieve angina by: A. Decreasing venous return to the heart B. Increasing intracardiac volume C. Increasing end diastolic left ventricular volume D. All of the above

348 A.

The main indication of this agent is hypertension in pregnancy: A. Reserpine B. Prazosin C. Methyldopa D. Clonidine

349 C.

Which of the following statements regarding the pharmacokinetics of theophylline is correct? A. It is primarily metabolized by the kidney B. Its metabolism depends on age C. It is poorly absorbed after oral administration D. It has a wide therapeutic index E. It stimulates phosphodiesterase

35 B. The metabolism of theophylline depends on age; the half-life of the drug in children is much shorter than in adults. The methylxanthines are all well absorbed and are metabolized in the liver. The Pharmacodynamic properties of theophylline include its adenosine-receptor antagonist activity and the inhibition of phosphodiesterase.

This antihypertensive drug is absolutely contraindicated in pregnancy because it can cause renal damage in the fetus: A. Reserpine B. Hydralazine C. Propranolol D. Captopril

350 D.

Which class of antibiotics can be safely given during pregnancy? A. Tetracycline B. Aminoglycoside C. Fluoroquinolones D. Beta-lactams

351 D.

Vancomycin is administered orally for this condition: A. Sepsis B. Endocarditis C. Antibiotic-associated enterocolitis D. Meningitis

352 C.

These anti-neoplastic agents are cell-cycle specific, except: A. 5-fluorouracil B. Cyclophosphamide C. Bleomycin D. Vincristine

353 B.

The preferred anti-thyroid drug for thyroid storm: A. Methimazole B. Carbimazole C. Propylthiouracil D. Propranolol

354 C.

Which of the following is a contraindication to the use of thiazolidinediones? A. Alcohol ingestion B. Heart failure C. Cholelithiasis D. Renal insufficiency

355 B. Reference: Katzung. Basic and Clinical Pharmacology, 11th ed. p. 743

Clomiphene acts to induce ovulation by: A. Diminishing estrogen mediated negative feedback at the pituitary B. Increasing the action of estrogen in the hypothalamus C. Increasing the action of estrogen in the ovary D. Increasing the amount of estrogen receptors

356 A.

Organophosphate toxicity is based on its ability to: A. Compete with brain pyridoxal phosphate B. Inhibit acetylcholinesterase C. Interrupt heme synthesis D. React with sulfhydryl groups binding to protein and other enzymes

357 B.

A drug used for treatment of advanced prostatic carcinoma by producing continuous gonadal suppression: A. Ketoconazole B. Testosterone undecanoate C. Leuprolide D. Cyproterone acetate

358 C. Leuprolide combined with an androgen receptor antagonist such as flutamide is the primary medical therapy for advanced prostate cancer and is as effective as surgical castration. Reference: Katzung. Basic and Clinical Pharmacology, 11th ed. p. 654

The following drug/s is/are used to augment labor: A. Oxytocin B. Misoprostol C. Methyl ergonovine D. All of the above

359 A.

A 74-year-old woman who is undergoing chemotherapy for advanced lung cancer presents to the infusion center for her next treatment. Before each treatment her white count, emoglobin, and platelet counts are checked to make sure she is not experiencing chemotherapy-related cytotoxicity. Her blood sample is run in the analyzer, and her platelet count is reported to be at a dangerously low level. Which medication is her oncologist likely to prescribe in this situation, along with a platelet transfusion? A. Erythropoietin B. Oprelvekin C. Filgrastim D. Sargramostim E. Leucovorin

36 B. Oprelvekin has been shown to reduce the need for platelet transfusions following myelosuppressive chemotherapy. Erythropoietin is used for anemia. Filgrastim and sargramostim are used for neutropenia. Leucovorin is used in patients undergoing treatment with methotrexate, to prevent some of its side effects.

Type II DM with insulin resistance will be best treated with which of the following: A. Sulfonylureas B. Gliptins C. Insulin glargine D. TZDs

360 D.

A 26/M presented with RUQ pain and fever. Imaging showed an abscess and aspiration biopsy was done. The aspirate resembled anchovy paste in color and in consistency. What is/are the possible drug/s that you can give for this patient? A. Metronidazole B. Tinidazole C. Chloroquine D. A and B E. All of the above

361 E For cases of amebic hepatic abscess you can give either Metronidazole, 750 mg 3 times daily (or 500 mg IV every 6 hours) for 10 days or Tinidazole, 2 g daily for 5 days PLUS a luminal agent. Alternatively you can give Dehydroemetine2 or emetine,2 1 mg/kg SC or IM for 8â€"10 days, followed by (liver abscess only) chloroquine, 500 mg twice daily for 2 days, then 500 mg daily for 21 days PLUS a luminal agent Luminal agent (Diloxanide, iodoquinol, paramomycin) Katzung 12th ed pg 928

Which antifungal drug inhibits β-glucan synthase causing a disruption in cell wall synthesis? A. Terbinafine B. Ketoconazole C. Amphotericin B D. Fluconazole E. Caspofungin

362 E Terbinafine - inhibits squalene oxidase; Ketoconazole - inhibits synthesis of ergosterol; Fluconazole - inhibits thymidylate synthase; Ampho B - polyene causes disruption of fungal cell walls Katzung 12th ed pg 857

A 65/M hypertensive, diabetic, smoker presents to you with fever, productive cough with an infected non healing wound in the foot. Labs showed anemia and very elevated creatinine.You schedule the patient for dialysis. As the prescribing physician, which of the following drug/s dose/s would you have to modify? A. Vancomycin B. Meropenem C. Metoprolol D. A and B E. All of the above

363 D. Vancomycin and meropenem need to be adjusted Katzung 12th ed pg 798 Metoprolol is metabolized in the liver and requires no adjustment pg 179

A 34/F was diagnosed to have lepromatous leprosy. She was given the appropriate drugs which she took religiously. One day she came back to you complaining that her skin changed to a red-brownish color. Which of the following drug/s most likely have caused her predicament? A. Dapsone B. Rifampicin C. Clofazimine D. Rifabutin E. Cycloserine

364 C. Clofazimine is a phenazine dye which causes red- brown to nearly black skin discoloration. SE of dapsone: methemoglobinemia; Rifampicin: red orange discoloration of body fluid Katzung 12th ed pg 846

Which of the following drug/s can be used for eliminating meningococcal carriage states? A. Minocycline B. Pen G C. Rifampin D. Demeclocycline E. A and C

365 E Minocycline, 200 mg orally daily for 5 days, can eradicate the meningococcal carrier state, but because of side effects and resis- tance of many meningococcal strains, rifampin is preferred. Katzung 12th ed pg 812

A 3/M was brought to the hospital for decreased sensorium. On further history, the child accidentally ingested a mouthful of antifreeze. You then proceed to give the child which antidote? A. Disulfiram B. Fomepizole C. Famotidine D. Methanol E. There is no antidote

366 B. Antidotes for ethylene glycol or methanol poisoning include ethanol and fomepizole. Although ethanol is effective, it is difficult to achieve safe and effective blood levels that is why fomepizole is preferred. Katzung 12th ed pg 1037

Which antihelminthic drugs rapidly kills adult worms presumably through inhibition of oxidative phosphorylation or stimulation of ATPase activity? A. Mebendazole B. Niclosamide C. Praziquantel D. Pyrantel pamoate E. Ivermectin

367 B. MOA of mebendazole: Mebendazole probably acts by inhibiting microtubule synthesis; Praziquantel: increase the permeability of trematode and cestode cell membranes to calcium, resulting in paralysis, dislodgement, and death; Pyrantel pamoate: neuromuscular blocking agent that causes release of acetylcholine and inhibition of cholinesterase which results in paralysis of worms followed by expulsion; Ivermectin: paralyze nematodes and arthropods by intensifying γ-aminobutyric acid (GABA)â€"mediated transmission of signals in peripheral nerves. Katzung 12th ed pg 941-945

A 60/M was diagnosed to have refractory follicular B-cell lymphoma. He was prescribed with an agent which binds CD20 on both normal and malignant B lymphocytes. Which of the following is this drug? A. Rituximab B. Cetuximab C. Bevacizumab D. Omalizumab E. Natalizumab

368 A. Cetuximab targets EGFR; Bevacizumab targets VEGF; Omalizumab is an anti-IgE useful in asthma therapy; Natalizumab binds to α4-subunit of α4β1 and α4β7 integrins expressed on the surfaces of all leukocytes except neutrophils. Used in MS and Crohn's disease. Katzung 12th ed pg 991-993

A 34/F asthmatic, scheduled to undergo cholelithiasis, was being induced by the anesthesiologist. After starting the volatile anesthetic, the anesthesiologist noted increased peak air pressure, prolonged expiration with the classic "shark fin" capnograph. Which of the following most likely have caused the above findings? A. Halothane B. Sevoflurane C. Desflurane D. Nitrous oxide E. None of the above

369 C. The patient is most likely experiencing bronchospasm and although all volatile anesthetics possess varying degrees of bronchodilating properties airway irritation induced by the pungency of isoflurane and desflurane may induce bronchospasm. These rarely occur with halothane, sevoflurane and nitrous oxide since they are non pungent. Katzung 12th ed pg 436

A 34-year-old carpenter presents to the ER after an accident in which he inadvertently chopped off the tip of his index finger. He is taken to the OR for reattachment of the digit, and after sedation, a local anesthetic is administered around the site of the injury. The local anesthetic used in the procedure did not contain any epinephrine, as it usually does for most surgical procedures. The reason for this is: A. Epinephrine causes increased blood loss during delicate surgery B. Epinephrine causes swelling of the tissues, making surgery more challenging C. Epinephrine is contraindicated in emergency surgery D. Epinephrine causes vasoconstriction, which can lead to vascular ischemia in digits E. Epinephrine can cause hypotension when administered with sedative agents

37 D. Epinephrine is contraindicated as an anesthetic adjuvant for surgeries involving most facial structures, digits, and the penis, because of the risk of vascular compromise. This agent causes decreased blood loss for most other surgeries because of vasoconstriction. Although local anesthetic agents such as lidocaine or xylocaine can cause mild local tissue swelling, epinephrine does not; either way, it is not a contraindication for hand surgery. Epinephrine causes elevated blood pressure when administered systemically; however, it has no systemic side effects when administered locally.

Which of the following regarding anticholesterol drugs is/are TRUE? A. Simvastatin, a reversible inhibitor of HMG-CoA reductase, is most effective in reducing LDL levels B. Niacin stimulates PPAR-α causing an increase in HDL and a decrese in triglycerides C. There is increased risk of gallstone formation when fibrates are combined with bile acid binding resins D. A and C E. All of the above

370 D. Fibrates stimulate PPAR alpha, most effective for decreasing elevated triglyceride levels. Katzung 12th ed pg 626, 628, 631

A 50/M is about to be given 6-mercaptopurine for his ulcerative colitis as an off-label use. The patient has multiple comorbidities which include hypertension, diabetes, asthma and gout and takes losartan, metformin, salmeterol, allopurinol and diclofenac for these conditions. Which of the above drugs would prompt you to rethink the dose of 6-MP that you are about to administer? A. Losartan B. Metformin C. Salmeterol D. Allopurinol E. Diclofenac

371 D. Because allopurinol inhibits xanthine oxidase, simultaneous therapy with allopurinol and 6-MP would result in increased levels of 6-MP, thereby leading to excessive toxicity. In this setting, the dose of mercaptopurine must be reduced by 50â€"75%. Katzung 12th ed pg 961-962

A 18/M previously diagnosed with epilepsy and maintained on phenobarbital seizure free for the past 2 years, presented to you with a 3 month history of cough, recurrent fever and weight loss. AFB was positive on two occasions and CXR show cavitations on the upper lung fields. You conclude that he has PTB and you start him of HRZE. After a few days, you were informed that you patient was in the ER and their working impression is status epilepticus. Which of the following drugs could have caused the breakthrough seizures? A. Isoniazid B. Rifampicin C. Pyrazinamide D. Ethambutol E. None of the above

372 B. Memorize inducers & inhibitors VERY HIGH YIELD

A 50/M previously diagnosed with BPH develops mild fever, cough and rhinorrhea. Thinking that he just has the common cold, he self medicates with over the counter medication. After 12 hours he rushes to the ER and complains of inability to void. Which of the following OTCs may have caused his condition? A. Chlorphenamine B. Phenylephrine C. Acetaminophen D. Ibuprofen E. None of the above

373 B. Sympathomimetic agents may cause or worsen urinary difficulty in patients with prostate enlargement due to smooth muscle contraction in the bladder neck via stimulation of alpha-1 adrenergic receptors. Katzung 12th ed pg 139

Which of the following have been proven to cause pulmonary fibrosis? A. Methothrexate B. Bleomycin C. Busulfan D. Amiodarone E. All of the above

374 E Topnotch

A 21/M presented with recurrent fever, night sweats, enlarged abdomen and generalized lymphadenopathy. Biopsy of a lymph node showed Reed-Sternberg cells. Which of the following chemotherapeutic regimen/s can be given to the patient? A. ABVD B. CHOP C. MOPP D. Rituximab E. A and C

375 A. ABVD (doxorubicin, bleomycin, vinblastine, and dacarbazine) and MOPP (mechlorethamine, vincristine, procarbazine, and prednisone) have been used to treat Hodgkin's lymphoma. ABVD has supplanted MOPP as the regimen of choice since it is more effective and less toxic. CHOP and Rituximab are used for NHL Katzung 12th ed pg 970

Drug X is currently undergoing clinical trials with it being tested on a small number of volunteers with the goal being to find the maximum tolerated dose. It is now on what phase of clinical testing? A. Phase 1 B. Phase 2 C. Phase 3 D. Phase 4 E. Phase 5

376 A. Phase 1- tested on small number of healthy volunteers in order to find the maximum tolerated dose and prevent severe toxicity; Phase 2- studied on a modest number of patients with the target disease to test efficacy; Phase 3- evaluated on a large number of patients to further establish efficacy and safety, usually a RCT with blinding and crossover; Phase 4- post marketing surveillance Katzung 12th ed pg 75

A 65/M was diagnosed to have CAP-HR. You plan to start Piperacillin-Tazobactam with a loading dose of 4.5g IV. Which among the following variables is not involved in the calculation of the loading dose? A. Volume of distribution B. Clearance C. Bioavailability D. Desired concentration E. All are needed to calculate the LD

377 B. Clearance is involved in determining the maintenance dose Topnotch

Which of the following inhibit/s cortisol synthesis? A. Aminoglutethimide B. Ketoconazole C. Mifepristone D. A and B E. All of the above

378 D. Aminoglutethimide blocks the conversion of cholesterol to pregnenolone and causes a reduction in the synthesis of all hormonally active steroids. Ketoconazole is a potent nonselective inhibitor of adrenal and gonadal steroid synthesis through inhibiton of P450 enzymes. Mifepristone is a glucocorticoid receptor antagonist with strong antiprogesterone activity. Katzung 12th ed pg 709- 711

A 34/F came in to the ER complaining of severe eye pain. On PE you notice that the cornea is steamy with accompanying conjunctival injection. You proceed to administer a drug that will cause an increase in the outflow of aqueous humor through the canal of Schlemm resulting in a decrease in IOP. Which drug was administered? A. Timolol B. Apraclonidine C. Latanoprost D. Pilocarpine E. Epinephrine

379 C. All of the choices can decrease IOP and are used in the treatment of glaucoma. Beta blockers, osmotic agents, alpha-2 agonists and carbonic anhydrase inhibitors decrease secretion of aqueous humor from the ciliary epithelium. Cholinomimetics cause contraction of the ciliary muscle causing the trabecular meshwork to open, increasing outflow. Prostaglandins increase outflow through the canal of Schlemm. Nonselective alpha agonists increase outflow via the uveoscleral veins. Topnotch

A 30-year-old woman has suffered from cyclical migraines for many years. She now presents to her physician asking for a medication designated specifically for migraines, not just a general pain reliever. Her physician decided to prescribe sumatriptan as a trial medication. The patient, who is a biochemist, would like to know how this medication works. A. It is a 5-HT1A agonist B. It is a 5-HT1D agonist C. It blocks reuptake of serotonin D. It is a 5-HT3 antagonist E. It is a 5-HT2A antagonist

38 B. Sumatriptan is a 5-HT1D agonist. An example of an agent known as a 5-HT1A agonist would be buspirone, an antianxiety agent. Fluoxetine is an example of a serotonin-reuptake inhibitor. Ondansetron, an antinausea medication, is a 5-HT3 antagonist. The antipsychotic medication Risperdal is an example of a 5-HT2A antagonist.

Minoxidil causes vasodilation through: A. Hyperpolarization of smooth muscle membrane through opening of potassium channels B. Blockade of voltage gated calcium channels causing a reduction in calcium influx C. Release of nitric oxide from the drug itself D. Activation of dopamine receptors E. Blockade of beta receptors

380 A. Choice B - calcium channel blockers; Choice C - Nitroprusside, nitrates; Choice D - fenoldepam

Which of the following drugs is classified as Category X during pregnancy? A. Valproate B. Statins C. Aspirin D. Captopril E. Phenytoin

381 B. The rest of the choices are category D drugs.

Which of the following is a balanced vasodilator--meaning, both an arterial and a venous vasodilator--used to treat both congestive heart failure and coronary artery vasospasm? A. Verapamil B. Captopril C. Nifedipine D. Propranolol E. Hydralazine

382 B. Balanced vasodilators include ACEIs, ARBs, alpha- adrenergic blockers, alpha-central agonists, nitroprusside and nesiritide.

Which of the following diuretics do not act on the luminal side of the renal tubules? A. Mannitol B. Furosemide C. Spironolactone D. Acetazolamide E. Hydrochlorothiazide

383 C. SIMILAR TO PREVIOUS BOARD EXAM CONCEPT/PRINCIPLE. Spironolactone is an aldosterone antagonist that blocks its binding to mineralocorticoid receptors in the basolateral membrane of the cortical collecting tubules. The rest of the choices act on the luminal side. Mannitol is an osmotic diuretic. Furosemide inhibits Na/K/Cl transporters in the luminal side of the loop of Henle. Acetazolamide inhibits carbonic anhydrase activity at the luminal side of the PCT. Hydrochlorothiazide inhibits Na/Cl transporters in the luminal side of the DCT.

Which of the following vasodilators has also been used for the treatment of male pattern baldness due to its side effect of inducing hair growth A. Verapamil B. Fenoldopam C. Nitroprusside D. Minoxidil E. Hydralazine

384 D.

Which of the following is the drug of choice for tic disorders? A. Carbamazepine B. Lamotrigine C. Valproic acid D. Topiramate E. Phenytoin

385 A. Carbamazepine is also the drug of choice for trigeminal neuralgia.

Which of the following drugs used in the treatment of peptic ulcer disease works by forming a protective coating over ulcer beds and has known adverse effects of melena and darkening of the tongue? A. Sucralfate B. Bismuth salicylate C. Psyllium D. Senna E. Kaolin

386 B.

Which of the following analgesics would you avoid in a patient with history of epilepsy as it is known to lower the seizure threshold? A. Ketorolac B. Aspirin C. Tramadol D. Celecoxib E. Indomethacin

387 C.

In which of the following drugs used for the treatment of gout is cataract an important side effect? A. Colchicine B. Probenicid C. Allopurinol D. Febuxostate E. Indomethacin

388 C. SIMILAR TO PREVIOUS BOARD EXAM CONCEPT/PRINCIPLE.

Which of the following adrenergic receptors in the ciliary body of the eye causes a decrease in aqueous humor production when activated? A. alpha-1 B. alpha-2 C. beta-1 D. beta-2 E. beta-3

389 B. Activation of alpha-2 receptors decreases aqueous humor production, while activation of either beta-1 or beta-2 receptors increases aqeous humor production. Alpha-1 and beta-3 receptors have no known function in relation to aqueous humor production.

A 12-year-old boy presents with a rash on the palms and soles of his feet as well as fever and headache. He was camping last weekend and admits to being bitten by a tick. His Weil-Felix test result is positive, suggesting Rocky Mountain spotted fever. What antibiotic should be given? A. Streptomycin B. Bacitracin C. Ciprofloxacin D. Doxycycline E. Erythromycin

39 D. Doxycycline, a tetracycline, is the antibiotic of choice to treat Rocky Mountain spotted fever, a rickettsial disease. Streptomycin can be used to treat plague and brucellosis. Bacitracin is only used topically. Ciprofloxacin can be used to treat anthrax. Erythromycin is the most effective drug for the treatment of Legionnaires disease.

Tocainide is an antiarrhythmic agent classified under which group? A. Class IA B. Class IB C. Class IC D. Class II E. Class III

390 B.

Ondansetron is an antagonist of which serotonin receptor? A. 5HT1D B. 5HT1B C. 5HT2 D. 5HT3 E. 5HT4

391 D. 5HT3 receptor is also the only ligand-gated ion channel among the serotonin receptors.

Clomiphene is a selective estrogen-receptor modulator commonly used to induce ovulation in patients undergoing assisted reproduction therapy. Which of the following best describes its mechanism of action? A. It potentiates the action of FSH and LH on the ovaries. B. It acts as a partial agonist in the pituitary to block negative feedback from estradiol. C. It acts as a GnRH analogue that increases the release of FSH and LH from the pituitary. D. It inhibits the release of inhibin from ovarian follicles, thus facilitating FSH and LH secretion. E. It is an LH analogue and mimics the physiologic LH surge when given just before ovulation.

392 B. C refers to leuprolide when administered in pulsatile fashion.

Which of the following anti-lipid drugs would you avoid giving to a diabetic patient due to its adverse effect of inducing hyperglycemia? A. Cholestyramine B. Egetimibe C. Niacin D. Gemfibrozil E. Colestipol

393 C.

Which of the following traditional antipsychotic agents has now been more widely used for antiemetic treatment of nausea and vertigo? A. Prochlorperazine B. Promethazine C. Haloperidol D. Chlorpromazine E. Metoclopramide

394 A. Promethazine is histamine H1-antagonist. Haloperidol and chlorpromazine are typical antipsychotics mainly used for psychotic disorders, not so much as antiemesis. Metoclopramide is a D2- antagonist used as GI prokinetic agent.

Which of the following is not used for the treatment of alcohol dependence? A. Chlordiazepoxide B. Disulfiram C. Naltrexone D. Acamprosate E. None of the above

395 A. Chlordiazepoxide is used for the treatment of alcohol withdrawal, not dependence. Diazepam may also be used for alcohol withdrawal.

Which of the following antitumor antibiotics is cell-cycle specific? A. Mitomycin B. Doxorubicin C. Bleomycin D. Actinomycin E. None of the above

396 C. Bleomycin is specific for G2 phase of the cell cycle. The rest of the choices are non-cell-cycle specific.

Which of the following antimicrobial agent acts as a dihydrofolate reductase inhibitor? A. Trimethoprim B. Sulfamethoxazole C. Metronidazole D. Nitrofurantoin E. Rifampicin

397 A.

In which of the following penicillins is interstitial nephritis a known adverse effect, partly leading to a decline in its popularity? A. Nafcillin B. Methicillin C. Carbenicillin D. Bacitracin E. Metampicillin

398 B. SIMILAR TO PREVIOUS BOARD EXAM CONCEPT/PRINCIPLE.

Which of the following antibiotics work by binding to the D-Ala-D-Ala terminus of bacterial cell wall precursors, thus preventing peptidoglycan polymerization? A. Imipenem B. Cycloserine C. Bacitracin D. Vancomycin E. Piperacillin

399 D. SIMILAR TO PREVIOUS BOARD EXAM CONCEPT/PRINCIPLE. Piperacillin: inhibits transpeptidase and binds to penicillin-binding proteins in the cell membrane. Cycloserine: blocks incorporation of D-Ala into the pentapeptide side chain of peptidoglycan. Bacitracin: interferes with a late stage in cell wall synthesis.

Which of the following antibiotic agents is bacteriostatic? A. Piperacillin B. Vancomycin C. Gentamycin D. Doxycycline E. Co-trimoxazole

4 D. Generally, cell wall inhibitors are -cidal, drugs which interfere with DNA are -cidal except sulfonamides which are only static unless they are given with trimethoprim / pyrimethamine (in this case synergism results to cidal activity) Protein synthesis inhibitors are -static except aminoglycosides because they cause frameshift mutations by misreading of the genetic code. Source: Topnotch handout on Pharmacology

A 55-year-old woman is admitted to the surgical intensive care unit after having a coronary artery bypass grafting of four of her coronary vessels. Overnight she develops hypotension, and her cardiac output, as measured by the Swan-Ganz catheter, is significantly lower than it had been postsurgery. You decide to give her a dose of milrinone. This results in an increase in her cardiac output. How does milrinone work? A. It is a cholinergic agonist B. It reduces left ventricular filling pressure C. It potentiates cardiac phosphodiesterase type 3 D. It decreases cyclic AMP E. It decreases intracellular calcium

40 B. Milrinone reduces left ventricular filling pressure and thus enhances cardiac output. It is related to the anticholinergic agent biperiden. Milrinone inhibits cardiac phosphodiesterase type 3. It increases cAMP, and therefore intracellular calcium.

Which of the following ophthalmic agents decreases intraocular pressure by increasing outflow of aqueous humor through the uveoscleral or unconventional pathway? A. Homatropine B. Bimatoprost C. Apraclonidine D. Mannitol E. Timolol

400 B. Bimatoprost is a prostaglandin analogue that increase outflow via the uveoscleral pathway. Apraclonidine is a alpha-2 receptor agonist that decreases aqueous humor production. Mannitol works via osmosis. Timolol is a beta-receptor antagonist that also decreases aqueous humor production. Homatropine has no effect on IOP.

Which of the following chemotherapeutic drugs may be used for Lentigo Maligna? A. Actinomycin B. Bleomycin C. Doxorubicin D. Vinblastine E. Cisplatin

401 A. SIMILAR TO PREVIOUS BOARD EXAM CONCEPT/PRINCIPLE

Which of the following chemotherapeutic drugs may be used for Lung Cancer? A. Procarbazine B. Cytarabine C. Topotecan D. Cisplatin E. Cyclophosphamide

402 D. SIMILAR TO PREVIOUS BOARD EXAM CONCEPT/PRINCIPLE

Which of the following drugs is an effective prophylactic drug against migraine headache attacks? A. Ondansetron B. Sumatriptan C. Ergotamine D. Methylergonovine E. Propranolol

403 E SIMILAR TO PREVIOUS BOARD EXAM CONCEPT/PRINCIPLE

A newly formulated drug with a half life of 6 hours is eliminated via 1st order kinetics, how many hours will the drug be 100% eliminated from the body of a normal person? A. 6 hours B. 12 hours C. 18 hours D. 24 hours E. 36 hours

404 D. 100% elimination will be reached in 4 half lives. (6 x 4=24)

In calculating the loading dose of a drug, which of the following parameters are not necessary? A. Volume of distribution B. Desired Plasma Concentration C. Bioavailability D. Clearance E. None of the Above

405 D. LD = (Vd x DPC) / Bioavailability

Which of the following is a reversible inhibitor of Xanthine Oxidase used in Chronic Gout? A. Allopurinol B. Colchicine C. Febuxostat D. Mycophenolate Mofetil E. Probenecid

406 C. Allopurinol is an IRREVERSIBLE Xanthine Oxidase inhibitor, Febuxostat is a REVERSIBLE Xanthine Oxidase inhibitor,

A patient will be undergoing neurosurgery, the neurosurgeon would like to maintain the patient's consciousness while the procedure is on going. What would be the preferred anesthetic of choice? A. Ketamine B. Etomidate C. Midazolam D. Fentanyl E. Propofol

407 A. Ketamine is capable of Dissociative Anesthesia ( Analgesia, Amnesia and Catatonia with retained consciousness)

A 14 year old child was diagnosed with Schizophrenia, which of the following is the only antipsychotic approved for pediatric patients? A. Clozapine B. Risperidone C. Quetiapine D. Olanzapine E. Thioridazine

408 B.

The following statements are true of Aminoglycosides, EXCEPT: A. Act by Time-Dependent Killing Action B. As the plasma level is increased above the MIC an increasing proportion of bacteria are killed and at a more rapid rate C. Has a post-antibiotic effect D. Requires Oxygen for uptake E. Amikacin has the widest coverage

409 A.

Which of the following is true A. Azithromycin has lesser bioavailability when taken with food B. Metronidazole has the same oral and parenteral bioavailability. C. The oral bioavailabity of chloramphenicol is higher than parenteral form. D. Both B and C E. AOTA

41 E Choramphenicol is one of the few antibiotics with higher oral bioavailability than parenteral form.

Among the following Anti-arrhythmics, which of the following has a highest potential for Torsade de pointes? A. Lidocaine B. Propranolol C. Sotalol D. Procainamide E. Verapamil

410 C. K channel blockers like Sotalol can increase the QT interval thereby increasing the propensity to develop Torsade de Pointes.

A patient with CHF was rushed to the ED where you are on-duty, ABG was done revealing Metabolic Acidosis. His wife told you that he had a history of severe allergic reaction to Sulfa drugs, but you need to give the patient a powerful diuretic, Unfortunately the following drugs are the only ones available in your Pharmacy, Among which of the following will you give? A. Furosemide B. Spironolactone C. Torsemide D. Ethacrynic Acid E. Acetazolamide

411 D. Spironolactone and Acetazolamide can aggravate the ongoing acidosis of the patient, the only ones left are loop diuretics, but then Furosemide and Torsemide are both Sulfa containing hence cannot be given to the patient. Ethacrynic acid is most suitable to this patient.

To improve the quality of life of patients with on-going treatment for Prostate Adenocarcinoma with Leuprolide, which of the following medications should be added? A. Anastrazole B. Examestane C. Flutamide D. Prednisone E. Morphine

412 C. Flutamide an Androgen Antagonist is given in adjunct to Leuprolide to lessen the side effects of Leuprolide like Gynecomastia, decreased libido, Apoplexy and hot flushes.(SIMILAR TO PREVIOUS BOARD EXAM CONCEPT/PRINCIPLE)

A 9-year old Filipino child was brought to you in the clinics by her mother due to an observed blank-stare episodes noted by her mother which lasts from a few seconds to a few minutes. Which of the following medications is the most suitable for this patient? A. Valproate B. Phenytoin C. Phenobarbital D. Carbamazepine E. Lorazepam

413 A. Valproate is the DOC for Absence Seizures, most especially in our setting here in the Philippines, Ethosuximide is NOT commercially available.

Which of the following Beta Blockers has an Intrinsic Sympathomimetic Activity? A. Carvedilol B. Pindolol C. Nadolol D. Metoprolol E. Timolol

414 B. Pindolol has ISA, which makes it a B blocker with a partial Alpha agonist action.

Early morning hypoglycemia is a challenge among physicians in treating diabetic patients. When a diabetic patient monitors his Glucose at 3 AM revealing Hypoglycemia with a Hyperglycemic 7 AM CBG, This effect is termed as: A. Somogyi B. Waning C. Dawn D. Dawn and Waning E. Somogyi and Waning

415 A.

In a patient with Grave's Disease this drug is given to decrease peripheral conversion of T4 to T3: A. Propranolol B. I 131 C. Methimazole D. PTU E. Lugol's Iodine

416 A.

In a Diabetic patient with impaired renal function which of the following drugs is contraindicated? A. Nateglinide B. Glipizide C. Metformin D. Pioglitazone E. Acarbose

417 B. Before prescribing 2nd generation Sulfonylureases liver and hepatic function must be checked first because they are contraindicated.

Which of the following Cephalosporins is synergistic with Aminoglycosides? A. Cefazolin B. Cefaclor C. Cefixime D. Cefipime E. Loracarbef

418 C. 3rd gen Cephalosporins have syndergistic effects with Aminoglycosides, 1st and 2nd gen cephalosporines can increase nephrotoxic effects of aminoflycosides.

What class of anti-hypertensive drugs are also used in BPH? A. Alpha 1A Agonists B. Alpha 1B Agonists C. Alpha 1A Antagonists D. Alpha 1B Antagonists E. Alpha 2 Agonists

419 C. Alpha 1-A receptor is specifically seen in the Urinary tract most especially the Prostate.Alpha 1A antagonists/Quinazolines include Prazosin Terazosin, and Tamsulosin

The following has increased bioavailabity when taken with food, except A. Cefuroxime B. Fusic acid C. Nitrofuratoin D. Griseofulvin E. NOTA

42 E All of the choices has increased bioavailability when taken with food.

Cocaine has which of the following mechanism of action? A. Promotes Adrenergic Synthesis B. Inhibits Adrenergic Storage C. Promotes Adrenergice Release D. Inhibits Adrenergic Metabolism/Degradation E. Inhibits Adrenergic Reuptake

420 E

A condition where there is depletion of needed substrates as a result of continuous activation by a drug and hence decrease in responsiveness, such as in chronic nitroglycerin use, is known as: A. Tachyphylaxis B. Idiosyncratic response C. Tolerance D. Resistance

421 C. Idiosyncratic means unusual drug reponse, infrequently observed in most patients; tolerance on the other hand, refers to decrease in the intensity of response to drug as a consequence of continued administration.

Teratogenesis is one of the dreaded complications of improper use of drugs during pregnancy. Of note, lithium, a drug primarily used to treat bipolar disorders, can cause: A. Ebstein's anomaly B. Vaginal clear cell adenocarcinoma C. Phocomelia D. Mobius sequence

422 A.

Pralidoxime is an antidote to organophosphate poisoning. Which of the following is not true about this drug? A. It is a cholinesterase regenerator. B. It requires to be given 6-8 hours before organophosphate-cholinesterase binding occurs. C. It binds with the phosphorus of organophosphates. D. All of the above E. None of the above

423 E

Methyldopa is a drug of choice in treating pre-eclampsia. Its ability to lower blood pressure is based on its action of: A. Activating alpha-1 adrenergic receptors B. Inhibiting alpha-1 adrenergic receptors C. Activating alpha-2 adrenergic receptors D. Inhibiting alpha-2 adrenergic receptors

424 C. Activation of alpha-2 adrenergic receptors by methyldopa metabolites in the brainstem appears to inhibit sympathetic nervous system output and lower blood pressure. This is also the mechanism of action of clonidine. Activation of alpha-1 adrenergic receptors meanwhile, stimulates vasoconstriction and hence increase in BP.

Hydralazine lowers BP by causing arteriolar smooth muscle relaxation. This effect is achieved by its ability to alter metabolism of which ion? A. Sodium B. Potassium C. Magnesium D. Calcium

425 D.

Throbbing headaches in patients taking nitrates are mainly attributed to: A. Chemical toxicity B. Meningeal artery vasodilation C. Reflex tachycardia D. Abrupt lowering of BP

426 B.

Which diuretic is found to have long-term benefits and can reduce mortality in chronic heart failure? A. Thiazide B. Furosemide C. Spironolactone D. Ethacrynic acid

427 C.

Aside from its lipid-lowering action, statins are used in coronary artery disease because it can also: A. Vasodilate stenotic segments of the coronary B. Stabilize atherosclerotic plaques C. Inhibits platelet aggregation D. Decrease myocardial oxygen demand

428 B.

Which of the following is considered as a reliever of asthma rather than a controller? A. Montelukast B. Cromolyn C. Omalizumab D. Terbutaline

429 D. Terbutaline is a beta-2 receptor agonist used as a fast-acting bronchodilator for asthma and is also used in preventing premature labor (tocolysis).

The following drugs have concentration dependent killing, except A. Aminoglycosides B. Fluoroquinolones C. Penicillin D. Metronidazole E. NOTA

43 C. Concentration dependent or dose-dependent killing means that the higher the concentration, the greater the bactericidal effect. Examples are aminoglycosides, fluoroquinolines, daptomycin, metronidazole, streptogramins. Penicillin employs time-dependent killing.

Oral chelation treatment in hemochromatosis can be done by giving: A. Deferoxamine B. Deferasirox C. EDTA D. Penicillamine

430 B. Therapeutic phlebotomy is the main line of treatment of hemochromatosis, especially the primary type. Should chelation becomes warranted, it can be done via IV (deferoxamine) or oral (Deferasirox).

Aspirin toxicity results in increased respiratory drive that leads to hyperventilation and respiratory alkalosis. Which of these statements is not true about aspirin intoxication? A. It is achieved when given at a dose of 150 mg/kg. B. It can manifest as a pure high anion gap metabolic acidosis in children. C. It is a known inhibitor of the electron transport chain. D. All of the abbove E. None of the above

431 C. Aspirin is not an inhibitor of electron-transport chain (ETC), but an uncoupler. Uncouplers of oxidative phosphorylation in mitochondria inhibit the coupling between the electron transport and phosphorylation reactions and thus inhibit ATP synthesis without affecting the respiratory chain and ATP synthase (H(+)-ATPase), thus heat is just produced without producing the needed ATPs.

Tranexamic acid is used to prevent and treat acute bleeding episodes in patients who are at high risk, especially those who have hemophilia and intracranial aneurysms. The mechanism of action of this drug involves: A. Inhibition of plasminogen activators B. Inhibition of plasmin-degrading factors C. Potentiation of factor VIII activity D. Delivery of Vitamin K to the clotting factor-synthesizing cells

432 A. Plasminogen are converted to plasmin whose function is to lyse the formed fibrin. When it is inhibited, hemostasis is promoted.

Treatment for inflammatory bowel disease: A. Sulfadiazine B. Sulfasalazine C. Sulfamethoxazole D. Sulfatide

433 B. Sulfasalazine is poorly absorbed in the GIT and therefore it is believed that it has its main action in lumen. It is used to treat IBDs by reducing the synthesis of inflammatory mediators known as eicosanoids and inflammatory cytokines. However, unlike glucocorticoids, sulfasalazine is also a mild immunosuppressant.

Remarkable to these drugs is their ability to inhibit cytochrome P450 causing significant interactions with other drugs, except: A. Isoniazid B. Cimetidine C. Erythromycin D. Barbiturates

434 D.

Which aminoglycoside has the narrowest therapeutic window but with the widest coverage? A. Gentamicin B. Streptomycin C. Amikacin D. Kanamycin

435 C.

Antifolates sulfamethoxazole and trimethoprim are bactericidal by virtue of their synergistic actions. Nonetheless, resistence can develop via plasmid-mediated: A. Decrease in sensitivity of dihydropteroate synthase to sulfonamides B. Increase in PABA synthesis by the organism C. Poor intracellular accumulation of the drugs D. All of the above E. None of the above

436 D.

Isoniazid is the most important drug used in TB. It inhibits mycolic acid synthesis and is therefore bactericidal. Possible adverse effects include neuritis, insomnia and seizures. To prevent neurotoxicities, patient should be supplemented with: A. Folic acid B. Pyridoxine C. Vitamin A D. Riboflavin

437 B.

What is the drug of choice for the cyst carrier state of Entamoeba histolytica? A. Diloxanide furoate B. Metronidazole C. Tinidazole D. Paramomycin

438 A.Diloxanide furoate is a luminal amoebicide which is hydrolysed in the gut, thus releasing the free diloxanide which acts as an amoebicide. It is given alone in asymptomatic cyst passers. For patients with active amoebic infections, it can be administered along with metronidazole.

A vinca alkaloid acting primarily in the M phase of cancer cell cycle that is useful in the treatment of acute leukemias, lymphomas and neuroblastoma: A. Paclitaxel B. Vincristine C. Doxorubicin D. Cytarabine

439 B.

Strongest inhibitor of CYP3A4 amongst the protease inhibitor A. Ritonavir B. Saquinavir C. Lopinavir D. Amprenavir E. Atazanavir

44 A. Ritonavir - Strongest

This is a mucosal protective drug used as an adjunct treatment of PUD. It binds to injured tissues and forms a protective covering over ulcer beds, resulting in accelerated healing: A. Misoprostol B. Bismuth salicylate C. Calcium carbonate D. Sucralfate

440 D.

Factor/s that influence/s passage of drugs through cell membranes: A. Presence of pores in the membrane B. Amount of protein binding C. Lipid solubility D. pH E. All of the above

441 E All are faCtors that influenCe passage of drugs through Cell membranes. Lipid solubility is the most important limiting factor for permeation of drugs because cells are covered by lipid membranes.

In this clinical phase of drug development, the drug is studied in patients with the target disease to determine its efficacy: A. Phase 0 B. Phase 1 C. Phase 2 D. Phase 3 E. Phase 4

442 C. Phase 1 trial involves evaluation of dose-response relationship and pharmacokinetics in normal volunteer patients with target disease. Phase 2 trial involves determination whether the agent has the desired efficacy at doses that are tolerated by sick patients. Phase 3 trial is a large design including placebo and positive controls in a double-blind crossover design. Phase 4 trial is a postmarketing surveillance.

The most important pharmacologic action of digitalis in the treatment of heart failure is its ability to: A. Increase heart rate B. Reduce venous pressure C. Reduce arterial pressure D. Produce diuresis in edematous patients E. Increase myocardial contractile force

443 E The most important pharmacologic action of digitalis in the treatment of heart failure is its ability to increase myocardial contractile force by inhibiting Na/K ATPase and increaseing intracellular calcium.

Reserpine blocks this step of adrenergic transmission: A. Synthesis of the neurotransmitter B. Transport of neurotransmitter to vesicles C. Release of the neurotransmitter D. Reuptake into the presynaptic neuron E. Diffusion in the synaptic cleft

444 B. Reserpine blocks vesicular transport in adrenergic transmission.

A 25 year-old farmer was brought to the emergency room after having allegedly drank a poison. PE revealed that he had constricted pupils, excessive salivation and sweating, wheezing and had several bouts of vomiting and diarrhea. He was given antidotes Atropine and Pralidoxime. Pralidoxime exert its effects by: A. Destruction of the enzyme B. Competitive antagonism C. Noncompetitive antagonism D. Physiologic antagonism E. Chemical antagonism

445 E Pralidoxime is a cholinesterase regenerator, an antidote for organophosphate poisoning. Pralidoxime is an example of chemical antagonism. Chemical antagonism is when an antagonist directly interacts with the drug being antagonized to remove it or prevent it from reaching its target.

It is the major second messenger of beta receptor activation that participates in signal transduction: A. Inositol triphosphate B. cAMP C. cGMP D. Calcium E. Adenylyl cyclase

446 B. cAMP is the major second messenger of beta receptor activation that participates in signal transduction.

A neonate was given IV antibiotics for 3 days. However, the baby developed hypothermia associated with diarrhea and grayish color of the skin. What is the mechanism of action of the drug that was given to the baby? A. Inhibits transpeptidation in bacterial cell walls B. Inhibits protein synthesis by binding to 30s subunit C. Inhibits transpeptidation at 50s subunit D. Inhibits dihydropteroate synthase E. Inhibits DNA replication by binding to DNA gyrase

447 C. Chloramphenicol is the drug that was given to neonate causing gray baby syndrome. It inhibits transpeptidation at 50s subunit and it is bacteriostatic. Penicillin inhibits transpeptidation in bacterial cell walls. Aminoglycoside inhibits protein synthesis by binding to 30s subunit. Co-trimoxazole inhibits dihydropteroate synthase. Fluoroquinolone inhibits DNA replication by binding to DNA gyrase.

It is a drug that is used almost solely as an alternative treatment for gonorrhea in patients whose gonococci are resistant to other drugs is: A. Docycycline B. Spectinomycin C. Ofloxacin D. Azithromycin E. Tetracycline

448 B. Spectinomycin is an aminoglycoside that inhibits protein synthesis by binding to 30s subunit. It is the drug of choice of drug-resistant gonorrhea and for gonorrhea in penicillin-allergic patients.

Drugs like quinidine, procainamide and disopyramide are very useful in treatment of: A. Congestive heart failure B. Hypertension C. Thyrotoxicosis D. Status asthmaticus E. Arrhythmia

449 E Drugs like quinidine, procainamide and disopyramide are very useful in treatment of arrhythmia.

High dose statins are given during ACS, the MOA of which in these cases is: A. Inhibit HMG-CoA reductase B. Stabilize plaque C. Reduce TAG synthesis D. Increase HDL E. AOTA

45 B.

It is an anticonvulsant that is effective for neuropathic pain: A. Valproic acid B. Gabapentin C. Phenytoin D. Duloxetine E. Levetiracetam

450 B. Gabapentin is an anticonvulsant drug, a GABA derivative which blocks calcium channels, increases GABA release, very useful in neuropathic pain also in postherpetic neuralgia.

A 43 year-old male was diagnosed with Pulmonary Tuberculosis. He was given Anti-TB medications. However, he developed hyperuricemia. Which among these drugs inhibits the excretion of urate causing the above findings: A. Ethionamide B. Streptomycin C. Ethambutol D. Pyrazinamide E. Rifampicin

451 D. Pyrazinamide provoke avute gouty arthritis by inhibiting urate excretion.

A 28 year-old woman wanted to get pregnant. A substance that enhances probability of ovulation by blocking the inhibitory effect of estrogen and thus stimulating the release of gonadotrophin from the pituitary is: A. Tamoxifen B. Danazol C. Clomiphene D. Progesterone E. Oxytocin

452 C. Clomiphene citrate enhances probability of ovulation by blocking the inhibitory effect of estrogen and thus stimulating the release of gonadotrophin from the pituitary.

Upon taking Penicillin, a 20 year-old male suddenly experienced respiratory distress. A drug would be most helpful to treat the respiratory symptoms was given in this patient. In comparison to norepinephrine, epinephrine has more effects on: A. alpha 1 receptors B. alpha 2 receptors C. beta 1 receptors D. beta 2 receptors E. dopamine receptors

453 D. Epinehrine is a sympathomimetic, nonselective and direct-acting. It is helpful in patients with anaphylactic shock. Respiratory distress is treated by Epinephrine's action on beta 2 receptors resulting to bronchodilation.

Warfarin is prescribed to a 40 year-old male. The excessive anticoagulant effect results to bleeding in this patient can be reversed by: A. Administration of vitamin K B. Administration of fresh frozen plasma C. Discontinuance of the drug D. A and C E. All of the above

454 E Bleeding with warfarin can be reversed by administration of Vitamin K, FFP and by discontinuance of the drug.

An anticancer agent that is prescribed for melanoma: A. Carmustine B. Cytarabine C. Vincristine D. Vinblastine E. Cisplatin

455 A. Carmustine is an alkylating agent, cell cycle non- specific that is given for melanoma.

An antihelminthic agent for strongyloidiasis: A. Nifurtimox B. Praziquantel C. Ivermectin D. Diethylcarbamazine E. Niclosamide

456 C. Ivermectin is the drug of choice for strongyloidiasis. It intensifies GABA-mediated neurotransmission in nematodes and immobilizes parasites.

A 60 year-old female patient was diagnosed with breast cancer. She was given an alkylating agent, Cyclophosphamide. Later in the treatment, she developed hemorrhagic cystitis due to this toxic metabolite: A. Amifostine B. Acrolein C. Mercaptopurine D. Lomustine E. Mesna

457 B. Acrolein is the metabolite of Cyclophosphamide that causes hemorrhagic cystitis.

Chlorpromazine may be used not only in treating schzophrenia but is also effective in: A. Reducing nausea and vomiting B. Allergies C. Hypertension D. Treating bipolar disorders E. Sleep disorders

458 A. Chlorpromazine may be used not only in treating schzophrenia but is also effective in reducing nausea and vomiting.

The best treatment option for an obese patient with Type 2 Diabetes Mellitus is: A. Glibenclamide B. Metformin C. Repaglinide D. Acarbose E. Insulin

459 B. Metformin is a biguanide which is the first-line treatment for Type 2 DM, also a drug of choice for obese diabetics.

Single dose drug used in eradication of meningococcal carier state A. Rifampicin B. Ciprofloxacin C. TMP/SMX D. Chloramphenicol E. NOTA

46 B. Drugs used to eradicate meningococcal carrier state: 1. rifampin 2. ciprofloxacin (single dose) 3. ceftriaxone (single dose)

A 10 year-old child with asthma was prescribed with a drug which inhibits binding of leukotriene D4 to its receptor on target tissues. The drug prescribed is: A. Zileuton B. Ipratropium C. Montelukast D. Omalizumab E. Theophylline

460 C. Montelukast blocks leukotriene receptor for leukotrienes C4, D4, E4 preventing airway inflammation and bronchoconstriction in asthmatic patients.

The most dangerous hallucinogenic agent. A. LSD B. phencyclidine C. MDMA D. cocaine E. Marijuana

461 B.

A 50 year old smoker, hypertensive patient complained of 2 month cough and dyspnea. He was on Carvedilol for his hypertension. What is the side effect of this drug that aggravates the condition. A. bronchospasm B. bronchoconstriction C. tachycardia D. tremor E. all of the choices

462 A.

Mr. X was maintaining an unrecalled drug for the past few months. He is now complaining of visual changes or "yellow halo vision". What is the drug that is known to cause such effect? A. amiodarone B. ethambutol C. digoxin D. furosemide E. Procainamide

463 C.

A 45 year old male complained of chest pain upon exertion but relieved by rest. The doctor prescribed him a beta blocker. What is the effect of beta blocker? A. Increase oxygen supply B. Decrease oxygen demand C. Increase oxygen demand D. decrease oxygen supply E. A and B

464 B. drugs used in angina pectoris are the following: those that increase the oxygen supply are: nitrates and calcium blockers; those that decrease the oxygen demand are: beta blockers and still calcium blockers

Monday disease is caused by occupational exposure to: A. lead B. dust C. arsenic D. nitrates E. Gun powder

465 D.

What is the predominant form of a weakly acidic drug with pKa of 5 at physiologic pH? A. Unprotonated charged B. Protonated charged C. Unprotonated uncharged D. Protonated uncharged E. None of the choices

466 A. The physiologic pH is at 7.35-7.45 which is above the pKa of this drug 5. At pH above pKa, weakly acidic drug is predominantly unprotonated charged, if it is below pKa, the predominant form is protonated uncharged.

It is the flow or transfer of a drug to the bloodstream. A. distribution B. bioequivalence C. absorption D. bioavailability E. Elimination

467 C. Absorption is the transfer of a drug to the bloodstream. Distribution is the entry of a drug to the target organ. Elimination is the termination of drug action. Bioavailability is the fraction of a drug that reaches the systemic circulation. Bioequivalence is the comparability of 2 related drugsnto achieve peak blood concetration.

What type of muscarinic receptors is used by the heart? A. M1 B. M2 C. M3 D. N1 E. N2

468 B. M1 - nerve endings, M2-heart, M3 - smooth muscle, glands, endothelium

Rivastigmine is used for: A. Motion sickness B. Myasthenia gravis C. Schizophrenia D. Parkinson's disease E. Alzheimer's disease

469 E Rivastigmine is a cholinomimetic drug used for Alzheimer's disease.

Single dose oral antibacterial agent used to eradicate uncomplicated gonorrhoea A. Ceftriaxone B. Cefixime C. Azithromycin D. Both B and C E. AOTA

47 D. Ceftriaxone - single dose IM

The drug that is used for cardiogenic shock as a last resort. A. epinephrine B. dopamine C. dobutamine D. norepinephrine E. Vasopressin

470 D.

A 58 year old male hypertensive was diagnosed with benign prostatic hyperplasia. What is the antihypertensive drug that is the most selective for prostatic smooth muscle? A. prazosin B. tamsulosin C. losartan D. metoprolol E. Terazosin

471 B. Alpha1 blockers are antihypertensive that is also used for BPH. Prazosin, tamsulosin, and terazosin are alpha1 blockers, but the most selective for prostatic smooth muscle is tamsulosin.

A 75 year old male diabetic, hypertensive and a diagnosed case of CHF came in due to edema and dyspnea. If the patient had hypokalemia, a potassium sparing diuretic is indicated. This potassium sparing diuretic inhibits ENaC - epithelial sodium channel in collecting duct. A. Amiloride B. triamterene C. spironolactone D. all of the above E. A and B only

472 E Spironolactone, triamterene, and amiloride are all potassium sparing diuretic. Triamterene and amiloride inhibit the ENaC of collecting duct, while spironolactone inhibits aldosterone receptor in collecting ducts.

A 100kg male patient had elevated LDL and VLDL. What vitamin is also used as antihyperlipidemic drug? A. Vitamin A B. pyridoxine C. niacin D. resin E. Cholestyramine

473 C.

What is the drug of choice for paroxysmal supraventricular tachycardia? A. Esmolol B. adenosine C. verapamil D. Sotalol E. Any of the choices

474 B. Esmolol, sotalol, verapamil are all used for the treatment of supraventricular tachycardia. For paroxysmal supraventricular tachycardia, the drug of choice is adenosine.

A 10 year old child, known asthmatic had episodes of nocturnal asthma attacks approximately 2-3x/week. In order to prevent subsequent attack, what drug is used for prophylaxis against nocturnal attack? A. Salbutamol + Ipratropium B. Salbutamol C. salmeterol D. cromolyn E. Aminophylline

475 E Salbutamol is the druig of choice for acute asthma attack, while aminophylline is used for prophylaxis against nocturnal attacks.

A 4 year old child, febrile at 38deg. C was given Paracetamol at 15mkd. What is the mechanism of action of this drug? A. COX-2 selective inhibitor B. IL-1 and IL-6 inhibitors C. COX-3 selective inhibitor D. Irreversible COX-1 and COX-2 inhibitor E. TNF - α inhibitor

476 C. Paracetamol or acetaminophen selectively inhibits COX-3. COX-2 selective inhibitors are coxib such as celecoxib. Irreversible COX-1 and 2 inhibitor is aspirin. TNF - α inhibitors are infliximab, adalimumab and etanercept.

One of the choices does not belong to the third generation cephalosporins. A. cefotaxime B. ceftazidime C. ceftriaxone D. cefoxitin E. Ceftizoxime

477 D. All of the choices except cefoxitin are all third generation cephalosporin. Cefoxitin is a second generation.

A 32 year old G2P1 mother delivered a live term baby Boy with an upturned nose, mild midfacial hypoplasia, long upper lip, and lower distal digital hypoplasia. Upon review of maternal history, the mother was taking an anti-epileptic drug. The drug that is most likely used was: A. phenytoin B. valproic acid C. diazepam D. carbamazepine E. lamotrigine

478 A. The physical description of the neonate is consistent with fetal hydantoin syndrome which is the side effect of phenytoin

A 50 year old obese patient was diagnosed to have DM type 2. There were no other comorbidities. What hypoglycemic drug should you prescribed with this patient? A. Insulin B. Gliclazide C. Metformin D. Pioglitazone E. Sitagliptin

479 C. Metformin is the first line treatment for type 2 DM and is the drug of choice for obese patients. It is contraindicated in patients with renal disease, hepatic disease, and alcoholism

Disruption of cell membrane is not seen with A. Amphotericin B B. Fluconazole C. Nystatin D. Polymxin B E. Griseofulvin

48 B. Fluconazole inhibits ergosterol synthesis

Penicillin is the drug of choice for syphilis. It acts through: A. Inhibition of transpeptidase B. Inhibition of topoisomerase II C. Inhibition of glucan synthase D. Inhibition of D-ala-D-ala terminus E. None of the choices

480 A. Penicillin inhibits transpeptidase enzyme needed for the bacterial cell wall synthesis. Vancomycin inhibits D-ala D ala synthase. Caspofungin inhibits glucan synthase. Flouroquinolone inhibits topoisomerase II and IV.

Marc, an asthmatic, is on theophylline. He has a number of other medical conditions. Which of the following medications may be safely administered with theophylline? A. Penicillin B. Erythromycin C. Phenytoin D. Rifampicin

481 A. Theophylline has a narrow therapeutic window and drug interactions can lead to severe morbidity. Erythromycin decreases the clearance of theophylline and increases its serum levels. Phenytoin and rifampicin have the oppositve effect. Advice: Memorize the inducers and inhibitors of the CYP450 system!

What is the maximum dose of Vincristine? A. 100 micrograms B. 200 micrograms C. 1 milligram D. 2 milligrams

482 D. SIMILAR TO PREVIOUS BOARD EXAM CONCEPT/PRINCIPLE

A pregnant patient complains of multiple external genital warts. Which of the following agents can be used in this patient? A. Podofilox solution B. Imiquimod cream C. Trichloroacetic acid D. Sinecatechins ointment

483 C. The most common treatments for genital warts in pregnancy are cryotherapy, laser removal, and trichloroacetic acid. The rest of the choices are under Pregnancy Category C.

The drug of choice for leptospirosis is: A. Doxycycline B. Penicillin C. Ceftriaxone D. Erythromycin

484 B. SIMILAR TO PREVIOUS BOARD EXAM CONCEPT/PRINCIPLE The rest of the choices are alternate drug choices for leptospirosis.

A 23-year-old presents with abdominal pain, tenesmus, and mucoid diarrhea with frank blood. Which is the best empiric treatment? A. Ciprofloxacin B. Metronidazole C. Amoxicillin D. Cotrimoxazole

485 A. Ciprofloxacin is the recommended empiric treatment for patients with acute dysenteriae. SIMILAR TO PREVIOUS BOARD EXAM CONCEPT/PRINCIPLE

The most powerful hallucinogen known to man is: A. LSD B. marijuana C. methamphetamine D. phencyclidine

486 A. SIMILAR TO PREVIOUS BOARD EXAM CONCEPT/PRINCIPLE

It is the only licensed chemotherapy drug for advanced melanoma: A. adriamycin B. bleomycin C. cisplatin D. Dacarbazine

487 D. SIMILAR TO PREVIOUS BOARD EXAM CONCEPT/PRINCIPLE There was quite a number of questions on oncology drugs.

Plasma magnesium levels should be maintained at this therapeutic range to prevent eclamptic convulsions: A. 2-4 meq/L B. 4-7 meq/L C. 8-10 meq/L D. 10-12 meq/L

488 B.

This is the only ligand-gated serotonin receptor: A. 5HT1 B. 5HT2 C. 5HT3 D. 5HT4

489 C.

Highly leukogenic anti-neoplastic drugs A. Topoisomerase II inhibitor B. Alkylating agents C. Anthracycline antibiotics D. Both A and B E. AOTA

49 D. Toposiomerase II inhibitors and alkylating agents are the most leukogenic anti-neoplastic agents.

This autonomic drug should be given with caution in patients with depressive disorders: A. guanethidine B. metyrosine C. vesamicol D. Reserpine

490 D. Reserpine which crosses the blood brain barrier is notorious for causing depression and increasing suicidal tendencies.

If a patient has COPD, which of the following beta-blockers would be more preferrable to use, theoretically? A. propranolol B. carvedilol C. labetalol D. atenolol

491 D. Atenolol, metoprolol, esmolol, betaxolol, and acebutolol (A-BEAM) are more beta1-selective hence are advantageous in patients with comorbid pulmonary disease.

An 8-year-old being treated with a combination of chemotherapeutic agents had difficulty eating and drinking due to red, inflamed sores in her mouth and esophagus. Which of the following antineoplastic agents is the most likely etiology? A. Methotrexate B. Vinblastine C. Doxorubicin D. Prednisone

492 A. Major toxicities of methotrexate include gastrointestinal mucositis, bone marrow suppression, skin erythema and hepatic dysfunction. SIMILAR TO PREVIOUS BOARD EXAM CONCEPT/PRINCIPLE

A young boy with G6PD deficiency would note tea-colored urine after the intake of which drug/s: A. primaquine B. cotrimoxazole C. nitrofurantoin D. All of the above

493 D.

This drug activates peroxisome proliferator-activated receptors, a group of nuclear receptors, with greatest specificity to PPAR-γ receptors: A. pioglitazone B. gemfibrozil C. sitagliptin D. Exenatide

494 A. Do not confuse with the mechanism of action of fibrates which is activation of the nuclear transcription factor PPAR-α.

Constipation is one of the major side effects of this drug: A. Magnesium hydroxide B. Aluminum hydroxide C. Omeprazole D. Colchicine

495 B. Aluminum hydroxide causes constipation (ALang tae). Magnesium hydroxide causes diarrhea (MaGtatae). Colchicine also causes diarrhea if taken in excess.

A patient presents with petit mal seizures. The drug of choice for this type of seizure is: A. phenytoin B. topiramate C. ethosuximide D. lamotrigine

496 C. Petit mal seizures, also known as absence seizures, involves a brief, sudden lapse of consciousness. The DOC is ethosuximide.

A psychotic patient on haloperidol presents with rigidity, hyperpyrexia and autonomic instability. This condition is known as: A. Serotonin syndrome B. Neuroleptic malignant syndrome C. Tardive dyskinesia D. Malignant hyperthermia

497 B. Tardive dyskinesia is also one of the toxicities of antipsychotics aside from NMS. It is a condition where the patient presents with stereotypic oral- facial movements.

This diuretic can also be used in the treatment of glaucoma: A. acetazolamide B. hydrochlorothiazide C. mannitol D. amiloride

498 A. Technically, mannitol can also be used but acetazolamide is a better answer. SIMILAR TO PREVIOUS BOARD EXAM CONCEPT/PRINCIPLE

It is a monoclonal anti-IgE antibody used in the treatment of asthma resistant to inhaled steroids and long-acting B2-agonists: A. trastuzumab B. rituximab C. infliximab D. omalizumab

499 D. Trastuzumab aka herceptin is a monoclonal antibody that interferes with the HER2/neu receptor, rituximab is against the protein CD20 on B cells, and infliximab is against TNF-alpha.

Which of the following is not an indication for SSRIs? A. Major depressive disorder B. OCD C. Anxiety and panic attacks D. Phobias E. None of the above

5 E SSRIs are the first line drugs for MDD. OCD - SSRI! Source: Topnotch handout on Pharmacology

The prokinetic MOA of metoclopramide A. Inhibition of dopamine receptors in the gut B. Inhibition of motilin receptors in the gut C. Inhibition of serotonin receptors in the gut D. AOTA E. Both A and B

50 A. Metoclopramide inhibits the D2 receptors in the gut; Erythromycin stimulates the motilin receptors in the gut.

This drug inhibits platelet aggregation by irreversibly blocking ADP receptors: A. cilostazol B. tirofiban C. alteplase D. Ticagrelor

500 D. Cilostazol is a phosphodiesterase III inhibitor, tirofiban binds to GP IIb/IIIa receptor, and alteplase is a thrombolytic (not an anti-platelet) which converts plasminogen to plasmin.

These drugs act by preventing the binding of the agonist molecule to the receptor but do not activate generation of a signal: A. Pharmacologic antagonists B. Inverse agonists C. Competitive agonists D. Partial agonists E. Orphan drug

501 A. Katzung 10th ed, p. 11 - Pharmacologic anatgonists bind to the receptor preventing agonist binding thereby blocking the agonist's biologic actions.

This route of drug administration is the most convenient, however bioavailability may be less than 100% because of incomplete absorption and first-pass elimination. A. Intramuscular B. Subcutaneous C. Oral D. Transdermal E. All of the above

502 C. Katzung 10th ed, p. 41

Loca is a 23 year old G4P4 (4004) who has been on oral contraceptives for 8 months now. What is the expected drug-drug interaction If she was to start on an anti-tuberculosis drug that acts by inhibitting RNA synthesis and which imparts orange color to urine and sweat? A. Enhanced efficacy of the oral contraceptive she is taking B. Enhanced potency of the oral contraceptive she is taking C. reduced activity of the cytochrome P450 isoforms D. Increased elimination of the anti-tuberculosis drug E. Significantly lower serum level of the oral contraceptive

503 E Rifampin revs up the cytochrome P450 enzyme system thereby enhancing the metabolism and hence the elimination of the oral contraceptive the patient is taking resulting to its significantly lower serum level. Katzung 10th ed, p.774.

It is during this phase of drug development and testing that the effects of the drug as a function of dosage are established in a small number of healthy volunteers. A. Phase 1 B. Phase 2 C. Phase 3 D. Phase 4 E. Phase 5

504 A. Phase 1 trials determine the probable limits of the safe clinical dosage range in a small number of healthy volunteers. In Phase 2, the drug is studied in patients with the target disease to determine its efficacy. In phase 3, the drug is evaluated in larger numbers of patients with the target disease to further establish safety and efficacy. Phase 4 constitutes monitoring the safety of the new drug under actual conditions of use in large number of patients. Katzung 10th ed., 70-71.

Which of the following drugs lower blood pressure by preventing normal physiologic release of norepinephrine from postganglionic sympathetic neurons? A. Metyrosine B. Cocaine C. Reserpine D. Botulinum toxin E. Guanethidine

505 E Metyrosine inhibits tyrosine hydroxylase which is responsible for the conversion of Tyrosine to Dopa, the rate-limiting step in catecholamine synthesis. Cocaine blocks re-uptake of NE from the synaptic cleft thereby prolonging its effect. Reserpine blocks dopamine transport into the vesicle. Botulinum blocks Acetylcholine release from the nerve terminal into the junctional cleft. Study figures 6-3 and 6-4 of Katzung 10th ed, p. 79 and 81.

A tricyclic antidepresssant drug long used to reduce incontinence in institutionalized elderly patients and in children with bedwetting problems because of its strong anti-muscarinic actions: A. Ipratropium B. Imipramine C. Scopolamine D. Oxybutinin E. Fluoxetine

506 B. Katzung 115: Ipratropium-is an anti-muscarinic drug used as an inhalational drug for asthma. Scopolamine- is an antimuscarinic drug used to remedy seasickness. Oxybutinin- is an antimuscarinic selective for M3 receptors used to relieve bladder spasm after urologic surgery and is also valuable in reducing involuntary voiding in patients with neurologic disease. Fluoxetine is an SSRI.

Which of the following statements regarding diuretics is not true? A. Diuretics lower blood pressure primarily by depleting body sodium stores. B. The most common adverse effect of thiazide diuretics is potassium depletion. C. The intercalated cells in the collecting tubule are +the primary sites of H secretion. D. The distal convoluted tubule is the final site of NaCl reabsorption and is responsible for the final sodium concentration of the urine. E. The thick ascending limb of the loop of Henle is the "diluting segment".

507 D. The collecting tubule and not the distal tubule is the final site NaCl reabsorption. It is responsible for tight regulation of body fluid volume and for determining the final sodium concentration of the urine. Katzung 10th ed., 239.

Which of the following drug acts by selectively inhibitting stimulation of B1 adrenoceptor and may be advantageous in treating hypertensive patients who also suffer from asthma, diabetes, or peripheral vascular disease? A. Nadolol B. Metoprolol C. Propranolol D. Labetalol E. Carvedilol

508 B. Katzung 10th ed., 169. Metoprolol, Esmolol, Atenolol, Acebutolol, Betaxolol, Bisoprolol are selective B1-blockers. Propranolol and Nadolol are non-selective B-blockers. Labetalol and Carvedilol have combined alpha and b-blocking activity.

It is a very efficacious orally active arteriolar vasodilator that is also available in topical preparations used to stimulate hair growth for correction of baldness. A. Minoxidil B. Sodium Nitroprusside C. Hydralazine D. Diazoxide E. Fenoldopam

509 A. Katzung 10th ed, 173.

The following anti-convulsant is not sedating A. Phenobarbital B. Phenytoin C. Clonazepam D. Gabapentin E. NOTA

51 B. Phenytoin is the oldest non-sedating anti-seizure drug

A Class I-B anti-arrhythmic drug which has a high degree of effectiveness in arrhythmias associated with acute myocardial infarction. A. Quinidine B. Flecainide C. Propranolol D. Lidocaine E. Amiodarone

510 D. Katzung 10th ed., 225 Quinidine- Class IA, associated with cinchonism; Flecainide- Class IC, very effective in suppressing premature vebtricular contractions; Propranolol-Class 2, Amiodarone-broad spectrum of actions

A potent antiandrogen that is usually coadministered with leuprolide that has been used in the treatment of prostatic carcinoma that frequently causes gynecomastia and occassionally mild reversible hepatic toxicity: A. Goserelin B. Spironolactone C. Cyproterone D. Ketoconazole E. Flutamide

511 E Katzung 10th ed., 679. this is SIMILAR TO PREVIOUS BOARD EXAM CONCEPT/PRINCIPLE

Disulfiram causes extreme discomfort in patients who drink alcoholic beverages by inhibitting which enzyme resulting to accumulation of acetaldehyde causing flushing, throbbing headache, nausea, vomiting, sweating, hypotension and confusion within few hours of drinking alcohol: A. alcohol dehydrogenase B. aldehyde dehydrogenase C. alcohol decarboxylase D. aldehyde decarboxylase E. none of the above

512 B. Katzung 10th ed., 371. this was SIMILAR TO PREVIOUS BOARD EXAM CONCEPT/PRINCIPLE

The following helminths are matched with their respective drug of choice except: A. Fasciola hepatica - Bithionol B. Echinococcus granulosus - Albendazole C. Stongyloides stercoralis - Thiabendazole D. Schistosoma mansoni - Praziquantel E. Dracunculus medinensis - Metronidazole

513 C. The drug of choice in strongylodiasis and onchocerciasis that acts by intensifying GABA- mediated transmission of signals in peripheral nerves resulting to paralysis of the worms is IvermeCtin. Thiabendazole is an alternative drug only. Katzung 10th ed., 870; See also p.868 for the table of drug of choice for other helminthic infections

What is the drug of choice for the eradication of dormant liver forms of Plasmodium vivax and Plasmodium ovale? A. Quinidine B. Primaquine C. Mefloqione D. Chloroquine E. Lumefantrine

514 B. Primaquine is the only available agent active against the dormant hypnozoite stages of p. vivax and p. ovale. (radical cure) Katzung 10th ed., 852

This halogen in a 1:20,000 solution is bactericidal in 1 minute and kills spores in 15 minutes. It is the most active antiseptic for intact skin: A. Phenol B. Hypochlorus acid C. Sodium hypochlorite D. Iodine E. Hexachlorophene

515 D. Katzung 10th ed., 822. Phenolic disinfectants are the oldest of the surgical antiseptics. They are used for hard surface decontamination in hospitals and labs. They are no longer used as a disinfectant because of its corrosive effect on tissues, its toxicity when absorbed (hyperbilirubinemia in newborns) and its carcinogenic effect. Sodium hypochlorite (household bleach) in 1:10 dilution which provides 5000 ppm of available chlorine is recommended by CDC for disinfection of blood spills and to kill spores.

This antiviral agent against Influenza A acts by blocking the M2 proton ion channel of the virus particle and by inhibitting uncoating of the viral RNA within infected host cells, thus preventing its replication. A. Amantadine B. Ribavirin C. Oseltamivir D. Entecavir E. None of the abpve

516 A. katzung 10th ed., 815.

This drug is a derivative of rifamycin which is indicated in place of rifampin for treatment of tuberculosis in HIV-infected patients who are receiving concurrent protease inhibitors: A. Rifapentine B. Rifabutin C. Ritonavir D. Rifadin E. Rimactane

517 B. Katzung 10th ed., 777. Rifabutin is a less potent inducer of the cytochrome P450 enzyme (compared to Rifampin) resulting to slower elimination and longer halflife of protease inhibitors used in treating HIV patients is cases of HIV with concurrent tuberculosis. Rifapentine like Rifampin is a potent inducer of cytochrome p450 enzymes and should not be used to treat HIV-infected patients because of an unaccepptably high relapse rate with rifampin- resistant organisms. Rtonavir is a protease inhibitor. Rifadin and Rimactane are brand names of Rifampin.

A 35 year old woman came to your clinic because of dysuria. She has been experiencing burning on urination with increased frequency and urgency to urinate for 5 days now. Urinalysis revealed WBC of 20-30/hpf, bacteria=moderate, nitrite=positive. You may prescribe the following drugs to her except for: A. Ciprofloxacin B. Levofloxacin C. Ofloxacin D. Moxifloxacin E. None of the above

518 D. Quinolones act by blocking bacterial DNA synthesis by inhibitting bacterial topoisomerase II and IV. Moxifloxacin is the only non-renally cleared fluoroqionolone and thus achieves low urinary level making it ineffective for cases of UTI. Katzung 10th ed., 768.

The following are true of sulfonamides except: A. Sulfonamides are structural analogs of p-aminobenzoic acid and act by inhibitting dihydropteroate synthase and folate production. B. Sulfonamides are very effective against rickettsiae. C. Combination of a sulfonamide with an inhibitor of dihydrofolate reductase provides synergistic activity because of sequential inhibition of folate synthesis. D. trimethoprim-sulfamethoxazole is the drug of choice for infections such as Pneumocystis jiroveci pneumonia. E. Sulfadiazine with pyrimethamine is first-line therapy for treatment of acute toxoplasmosis.

519 B. It is interesting that rickettsiae are not inhibitted by sulfonamides but are actually stimulated in their growth. Katzung 10th ed., 763.

IOP can be decreased by increasing aqueous outflow with the following drugs A. Timolol B. Latanoprost C. Mannitol D. Dorzolamide E. NOTA

52 B. PG analogues decrease IOP by increasing aqueous outflow. CA inhibitors and Beta-blocker decrease aqueous production. Alpha2 agonist increases outflow and decreases production.

A 45 year old soldier has been maintaining on fluoxetine for his major depressive disorder. Which of the following drugs can precipitate a potentially fatal syndrome of hypertension, hyperreflexia, tremor, clonus, hyperhtermia, diarrhea, mydriasis and agitation within hours when taken consurrently with fluoxetine? A. Amytriptyline B. Imipramine C. Sertraline D. Citalopram E. Phenelzine

520 E This is a case of serotonin syndrome which is a condition precipitated when MAO inhibitors are given with serotonin agaonists, especially antidepressants of the SSRI class. Amytriptyline and Imipramine are TCAs. Fluoxetine, Sertraline and Citalopram are SSRIs. Phenelzine is a MAO inhibitor. Katzung 10th ed., 267.

A 37 year old man comes to the physician because of a 1 week history of pain with swallowing. He received the diagnosis of AIDS 4 years ago, but has not been able to tolerate highly active antiretroviral therapy. He currently takes TMP-SMX. Vital signs are within normal limits. Examination shows a few white plaques over the pharynx. CD4+ count is 50/mm3 and plasma HIV viral load is 50000 copies/mL. Which of the following is the most appropriate pharmacotherapy? A) Acyclovir B) Amphotericin B C) Fluconazole D) Foscarnet

521 C.

A 72 year old man is brought to the physician by his daughter because of a 7 month history of difficulty with memory. He frequently loses his keys and misses appointments and regular family dinners. He lives alone, but recently family members had to drive him on errands and help him with daily tasks at home. He has not had difficulty sleeping and does not use illicit drugs. Mental status examination shows an irritable mood and labile affect. There is no evidence of depressed mood, anxiety, or hallucinations. Which of the following is the most appropriate pharmacotherapy for this patient? A) Alprazolam B) Donepezil C) Dextroamphetamine D) imipramine

522 B.

A 47 year old woman comes to the physician for a routine health maintenance examination. She has a 2 year history of venous insufficiency and noted daily swelling of her ankles that worsens in the evening. She takes no medications. Her pulse is 80 bpm, and blood pressure is 160/100 mmHg. Cardiopulmonary examination shows no abnormalities. Examination of the lower extremities shows 2+ edema and increased pigmentation .Which of the following antihypertensive medications would most likely exacerbate this patient’s swelling? A) Atenolol B) Lisinopril C) Losartan D) Nifedipine

523 D.

A 55-year-old man has had crushing substernal chest pain on exertion over the past 6 weeks. He had a myocardial infarction 2 months ago. He takes nitroglycerin as needed and one aspirin daily. He has smoked two packs of cigarettes daily for 30 years. Examination shows normal heart sounds and no carotid or femoral bruits. Treatment with a β-adrenergic blocking agent is most likely to improve his symptoms due to which of the following mechanisms? A) Decreasing diastolic relaxation B) Decreasing myocardial contractility C) Dilating the coronary arteries D) Peripheral vasodilation

524 B.

A 72-year-old man comes for a routine follow-up examination. He has chronic obstructive pulmonary disease treated with β-adrenergic agonists and ipratropium by metered-dose inhaler and mild arterial insufficiency of the lower extremities treated with aspirin. His blood pressure is 160/60 mm Hg, pulse is 70/min, and respirations are 12/min. Funduscopic examination shows arteriovenous nicking. Pedal pulses are decreased bilaterally. Which of the following antihypertensive drugs is most likely to cause adverse effects in this patient? A) α2-Adrenergic agonist B) α-Adrenergic blocking agent C) β-Adrenergic blocking agent D) Angiotensin-converting enzyme (ACE) inhibitor

525 C. COPD and Peripheral arterial disease >> contraindications for Beta-Blockers

A 17-year-old boy is brought to the emergency department by his parents because of bizarre behavior for 6 hours. Last night he was out with friends, and since returning, he has been confused and has "trashed" his room. His blood pressure is 165/95 mm Hg. He is hypervigilant, has little spontaneous speech, and is disoriented to place and time. He appears catatonic but abruptly becomes assaultive two times and needs to be restrained. Which of the following is the most likely substance taken? A) Cocaine B) Ecstasy C) LSD D) PCP

526 D.

A 72-year-old man comes to the physician because of a 2-month history of urination twice nightly and occasional urinary frequency and urgency. He has a 15-year history of type 2 diabetes mellitus now moderately well controlled with glyburide. His father was diagnosed with prostate cancer at the age of 70 years, and his sister died of complications from systemic lupus erythematosus. His blood pressure is 135/86 mm Hg. Cardiopulmonary examination shows no abnormalities. Abdominal examination shows no suprapubic fullness or tenderness. There is mild enlargement of the prostate with no palpable nodules. His postvoid residual volume is 10 mL. Serum studies show a urea nitrogen (BUN) level of 45 mg/dL and creatinine level of 3.8 mg/dL. Urine dipstick shows 3+ protein. Which of the following is most likely to have prevented progression of this patient's renal disease? A) Oral finasteride therapy B) Oral terazosin therapy C) Oral cyclophosphamide and prednisone therapy D) Oral enalapril therapy

527D.

A 47-year-old man is admitted to the hospital after threatening to harm a radio announcer he believed was broadcasting his thoughts. Over the past 20 years, he has had multiple psychiatric hospitalizations for threatening people who he believed were plotting against him, trying to control his mind, or causing him to hear voices by implanting devices in his head. Past symptoms improved with neuroleptic therapy; after discharge, he discontinued the medication and his symptoms worsened. Which of the following is the most appropriate pharmacotherapy to decrease this patient's risk for future hospitalization? A) Clozapine B) Fluphenazine hydrochloride C) Haloperidol decanoate D) Risperidone

528 C.

A 47-year-old man is admitted to the hospital after threatening to harm a radio announcer he believed was broadcasting his thoughts. Over the past 20 years, he has had multiple psychiatric hospitalizations for threatening people who he believed were plotting against him, trying to control his mind, or causing him to hear voices by implanting devices in his head. Past symptoms improved with neuroleptic therapy; after discharge, he discontinued the medication and his symptoms worsened. Which of the following is the most appropriate pharmacotherapy to decrease this patient's risk for future hospitalization? A) Clozapine B) Fluphenazine hydrochloride C) Haloperidol decanoate D) Risperidone

529 C.

Ampicillin is eliminated by first-order kinetics. Which of the following statements best describes the process by which the plasma concentration of this drug declines? A. There is only 1 metabolic path for drug elimination B. The half-life is the same regardless of the plasma concentration C. The drug is largely metabolized in the liver after oral administration and has low bioavailability D. The rate of elimination is proportional to the rate of administration at all times E. The drug is distributed to only 1 compartment outside the vascular system

53 B. "First-order" means that the elimination rate is proportional to the concentration perfusing the organ of elimination. The half-life is a constant. The rate of elimination is proportional to the rate of administration only at steady state. The order of elimination is independent of the number of compartments into which a drug distributes.

A 32-year-old woman comes to the physician because of a 3-month history of increasing pain and stiffness in her wrists, hands, and ankles. During this period, she also has had progressive fatigue and morning stiffness lasting 2 hours. She has a 1-year history of rheumatoid arthritis treated with naproxen. Examination shows redness, swelling, and warmth over the wrist, hand, and ankle joints bilaterally. There are nontender subcutaneous nodules over the extensor surfaces of both elbows. X-ray films of the hands show diffuse osteopenia and erosions over several of the distal metacarpal bones. Which of the following is the most appropriate pharmacotherapy? A) Add oral cyclophosphamide B) Add oral gold C.) Add oral Methotrexate D.) Add Oral Penicillamine

530 C.

An otherwise healthy 19-year-old woman comes to the physician because of a 3-year history of intermittent facial blemishes. She drinks wine occasionally on weekends. She takes no medications. Examination shows multiple 1- to 2-mm red and white papules and on the forehead and cheeks. Which of the following is the most appropriate initial pharmacotherapy? A) Topical benzoyl peroxide B) Systemic corticosteroids C) Oral isotretinoin D) Topical corticosteroids

531 A. Oral isotretinoin is never the first option for acne, only used when topical treatments have failed

A 57-year-old man has been hospitalized for 2 days for treatment of unstable angina pectoris. He is currently receiving intravenous heparin and undergoing evaluation for coronary artery bypass grafting. His blood pressure is 160/90 mm Hg, pulse is 88/min, and respirations are 16/min. Laboratory studies show: Platelet count 90,000/mm3 Prothrombin time 12 sec (INR=1.1) Partial thromboplastin time 35 sec Which of the following is the most likely cause of these findings? A) Excessive platelet destruction B) Factor VIII deficiency C) Inadequate platelet production D) Uncontrolled activation of coagulation and fibrinolytic cascades

532 D. Heparin-induced thrombocytopenia >>> the Heparin-Platelet4 complex are attacked by IgG in the human bodies that essentially activate the platelet and thus form clots. These clots are the reason free platelets are low.

A 42-year-old woman, gravida 2, para 2, comes to the physician because of a 3-month history of swelling of her legs and mild abdominal pain and bloating. Abdominal examination shows no abnormalities. Rectovaginal examination shows fullness in the right adnexa. Transvaginal ultrasonography shows an irregular mass in the right ovary with some solid components to a predominantly cystic lesion. Her serum CA 125 level is 120 U/mL (N<35). Treatment with which of the following is most likely to have prevented this patient's symptoms? A) Antiestrogens B) Antiprogestationals C) Medroxyprogesterone D) Oral contraceptives

533 A.

A 67-year-old man is hospitalized for treatment of renal insufficiency. Three days after admission, his pulse is 40/min. An ECG shows tall, tented T waves. Serum studies show a sodium level of 134 mEq/L, potassium level of 6.9 mEq/L, and glucose level of 85 mg/dL. The most appropriate next step in management is intravenous administration of which of the following? A) Calcium, furosemide, and 3% saline B) Calcium, insulin, and digitalis C) Calcium, insulin, and glucose D) Glucose, furosemide, and phosphate

534 C. insulin and glucose >> fastest way to lower down serum K

A 37 year old man comes to the physician for a follow-up examination. He was diagnosed with HIV infection 3 weeks ago. He is asymptomatic and takes no edications. His temperature is 37.2 C, pulse is 100 bpm, and blood presure is 100/60 mmHg. Examination shows no abnormalities except for erythematous scaling at the hairline and nasolabial folds. His CD4+ T-lymphocyte count is 160/mm3 and plasma HIV viral load is 25,000 copies/mL. Which of the following is the most appropriate pharmacotherapy? A) Pneumocystis jiroveci prophylaxis only B) Three-drug antiretroviral therapy only C) Three-drug antiretroviral therapy and P. jiroveci prophylaxis D.) Two Drug Antiretroviral therapy and P. iroveci Prophylaxis

535 C. Start HAART with CD4 below 350 and Pneumocystis prophylaxis with CD4 below 200

A 55-year-old woman was recently diagnosed with an adrenal mass due to symptoms of sweating, elevated blood pressure, and severe headaches. Which of the following nonselective, irreversible alpha blocker is the drug of choice as a preoperative agent used in this disease? A) Phentolamine B) Phenoxybenzamine C) Prazosin D) Yohimbine

536 B. Phenoxybenzamine >> nonselective, IRREVERSIBLE alpha blocker..Phentolamine>> reversible alpha blocker.

A 72-year-old man was diagnosed to have glaucoma. Which of the following drugs induces ciliary muscle contraction thereby opening the trabecular meshwork and increasing the outflow? A) Timolol B) Mannitol C) Epinephrine D) Pilocarpine

537 D. Pilocarpine >> ciliary muscle contraction >> opening trabecular meshwork and increasing the outflow

A 58-year-old woman with newly diagnosed type 2 diabetes mellitus presents to the emergency department complaining of vomiting, severe headache, dizziness, blurry vision, and dyspnea. She says that she had been at a party when the symptoms began. Her skin is notably flushed on physical examination. Which of the following medications is responsible for this reaction? A) Tolbutamide B) Glipizide C) Glyburide D) Metformin

538 A. tolbutamide >> disulfuram reaction

A 25-year-old woman with AIDS comes to the physician because of a 10-day history of fever, shortness of breath, night sweats, fatigue, and a non-productive cough. She had been treated with antiretroviral agents for 2 years but stopped taking her medications 6 weeks ago. She has no known allergies. Her temperature is 38.6 C, pulse is 110/min, respiration are 20/min, and blood pressure is 110/60 mm Hg. Examination shows no cyanosis. Diffuse fine crackles are heard bilaterally. An x-ray of the chest shows bilateral interstitial infiltrates. A silver stain of sputum is positive for cysts and organisms. Which of the following is the most appropriate pharmacotherapy? A) Ceftriaxone B) Erythromycin C) Cotrimazole D) Co-trimoxazole

539 D.

Which helminthic infection does not respond to treatment with praziquantel? A. Hydatid disease B. Opisthorchiasis C. Paragonimiasis D. Pork tapeworm infection E. Schistosomiasis

54 A. In hydatid disease, praziquantel has marginal efficacy because it does not affect the inner germinal membrane of Echinococcus granulosus present in hydatid cysts.

A 27-year-old nulligravid woman comes to the physician for preconceptional counseling. She has a mechanical mitral heart valve and chronic rheumatoid arthritis. Her cardiac status is New York Heart Association Class II. She feels well. Current daily medications include warfarin, prednisone, and acetaminophen with codeine. Examination shows no abnormalities except for audible clicking from the heart valve. Which of the following is the most appropriate advice for this patient? A) Chemical dependency counseling before pregnancy B) Discontinuation of anticoagulant therapy during pregnancy C) Discontinuation of prednisone during pregnancy D) Switching from warfarin to heparin before pregnancy

540 C.

What is the half-life of alcohol? A. 2 hours B. 4 hours C. 8 hours D. 12 hours E. None of the above

541 E Ethanol, over most of its plasma concentration range, exhibits zero-order elimination. Thus, there is no constant half life of elimination as in drugs exhibiting first order elimination. (Katzung Board Review 9e, p.6)

A 48/M farmer with a history of alcohol abuse, is rushed to the emergency room after the patient has deliberately ingested 300 mL of an unknown pesticide while drunk. On examination, you note that the patient has pinpoint pupils and frothy secretions at the mouth. After you determine that the patient has ingested malathion; you then immediately start treatment with pralidoxime. Pralidoxime and malathion are examples of: A. Competitive pharmacologic antagonists B. Irreversible pharmacologic antagonists C. Physiologic antagonists D. Structural antagonists E. Chemical antagonists

542 E A chemical antagonist interacts directly with the drug being antagonized to remove it or prevent it from binding to its target. Pralidoxime which combines avidly with the phosphorus in organosphosphate cholinesterase inhibitors is an example. (KBR9e, p. 14)

A 48-year old Caucasian male is referred to you for chronic cough, night sweats and weight loss. Sputum AFB was positive on three occasions; findings of chest X ray done was also consistent with pulmonary tuberculosis. Before you start drug therapy, you remember that a much larger percentage of Caucasians are slow acetylators of one of the anti-TB drugs. The dosage of which of the following may need to be modified. A. Isoniazid B. Rifampicin C. Streptomycin D. Pyrazinamide E. Ethambutol

543 A. 50% of white and African american persons are slow acetylators of INH, hydralazine and procainamide. KBR9e, p33

A recently-diagnosed hypertensive was started on a BP lowering agent. Due to financial constraints, the patient abruptly stopped taking the prescribed medications after stocks were consumed. A day later, the patient was brought to the clinic for light-headedness; on admission, patient's BP was 180/100. Which of the following anti-hypertensives, known for rebound hypertension, was most likely taken by the patient? A. Enalapril B. Furosemide C. Methyldopa D. Clonidine E. Metoprolol

544 D.

A 24/G3P2(2002) who came in with a BP of 200/140 was given Hydralazine. What is the mechanism of action of hydralazine? A. Reduction of Ca influx via L-type channels B. Antagonism of alpha-adrenergic receptors in blood vessels C. Release of NO D. Hyperpolarization of smooth muscles due to opening of K channels E. Activation of dopamine D1 receptors

545 C. A describes the MOA of verapamil; D describes minoxidil; E describes fenoldopam. All are direct vasodilators.

Which of the following diuretics is incorrectly paired with its associated toxicity? A. Thiazide - renal stone B. Acetazolamide - hepatic encephalopathy C. Bumetanide - hypokalemic metabolic alkalosis D. Eplerenone - hyperkalemic metabolic acidosis E. Mannitol - pulmonary edema

546 A. Thiazides increase reabsorption of Ca and prevent Ca stone formation. Hepatic encephalopathy can ensue with acetazolamide because it causes alkalinization of urine due to decreased bicarbonate reabsorption. This prevents ammonia from turning to ammonium. Ammonia is more easily reabsorbed.

Leukotrienes are among the most potent bronchoconstrictors implicated in the pathogenesis of bronchial asthma. Which of the following is a leukotriene receptor inhibitor that has found clinical application in asthma prophylaxis? A. Zileuton B. Zafirlukast C. Ipratropium D. Budesonide E. Bosentan

547 B. Zileuton is a lipooxygenase inhibitor.

A 28/M car mechanic was brought to the emergency room after ingesting automobile antifreeze. ABG reveals high anion gap metabolic acidosis; while urinalysis reveals calcium oxalate crystals. You suspect ethylene glycol poisoning. Which of the following drugs is recommended? A. Naltrexone B. Diazepam C. Disulfiram D. Acamprosate E. Fomepizole

548 E Ethylene glycol and methanol poisoning may be treated with fomepizole, an alcohol dehydrogenase inhibitor, which prvents conversion of methanol/ehtylene glycol to toxic metabolites.

A patient with neuropathic pain was prescribed gabapentin. What is the primary mechanism of action of gabapentin? A. Direct activation of GABA reeptors B. Inhibition of T type Ca2+ channels C. Blockade of voltage-gated Na channels D. Inhibition of GABA transporter (GAT-1) prolonging the action of the neurotransmitter E. Enhancement of K+ channel permeability causing neuronal hyperpolarization

549 B. Although gabapentin is a structural analog of GABA, it does NOT activate GABA receptors directly.

This drug that produces a permanent reduction in thyroid activity is A. 131-I B. Amiodarone C. Propranolol D. Propylthiouracil E. Triiodothyronine

55 A. Radioactive iodine is the only medical therapy that produces a permanent reduction of thyroid activity.

A 6/F is brought to the clinic due to frequent episodes of sudden activity arrest with staring and minimal eyelid flutter occurring for 10 - 20 seconds, 5 - 10 times per day. During such episodes, patient is unresponsive to voice or tactile stimulation. She is diagnosed with absence seizure. Which of the following antiseizure drugs is most appropriate? A. Carbamazepine B. Valproic acid C. Phenytoin D. Phenobarbital E. Levetiracetam

550 B. Ethosuzimide and valproic acid are the preferred drugs for absence seizures because they cause minimal sedation.

Administration of most inhaled anesthetics cause a decrease in the following except: A. Arterial blood pressure B. Brain metabolic rate C. Myocardial function D. Tidal volume E. None of the above

551 E

A 53/M, admitted at the Medical ICU, after having a myocardial infarction, develops ventricular tachycardia. Which of the following anti-arrhythmics is most appropriate? A. Quinidine B. Mexiletine C. Adenosine D. Lidocaine E. Flecainide

552 D. Lidocaine and other Class IB drugs act primarily in ischemic tissues and are best for arrhythmias following MI.

Flumazenil may be effective in patients who overdosed on the following sedative-hypnotics except: A. Alprazolam B. Zolpidem C. Thiopental D. Midazolam E. Lorazepam

553 C. Most S-H drugs facilitate the actions of GABA, a major inhibitory transmitter. Benzodiazepines potentiate GABA by increasing frequency of Cl ion channel opening; its action is blocked by flumazenil, a BZ receptor antagonist. Flumazenil, on the other hand, does NOT block the effects of barbiturates.

Which of the following IV anesthetics is the only one that causes CV stimulation? Its use in neurosurgical operations is limited due to its propensity to increase ICP. A. Midazolam B. Propofol C. Fentanyl D. Ketamine E. Thiopental

554 D.

Sisa, 28/F, recently-diagnosed schizophrenic was admited to the Psychiatry ward and started on Risperidone. Three days later, the patient developed fever (39.9C) and rigidity. Patient was notably confused. Vital signs were as follows: BP 170/120, HR 118, RR 22. What could explain Sisa's symptoms? A. Extrapyramidal symptoms B. Tardive dyskinesia C. Serotonin syndrome D. Neuroleptic malignant syndrome E. Malignant hyperthermia

555 D. Neuroleptic malignant syndrome is characterized by FEVER, RIGIDITY, and AUTONOMIC INSTABILITY. NMS is linked to intake with several atypical antipsychotics.

Formation of methyltransferases that alter drug binding sites on the 50S ribosomal subunit is the primary mechanism of resistance to which antibiotic: A. Streptomycin B. Tetracycline C. Amikacin D. Azithromycin E. Levofloxacin

556 D. Question could be rephrased into: which acts on the 50s subunit? Macrolides such as azithromycin do.

A 41/M, presenting with high fever and chills, is admitted at the hospital. He reports having had serious allergic reactions to amoxicillin one year ago. Blood GS/CS reveals growth of Gram negative bacteremia. Which of the following antibiotics is MOST appropriatE? A. Cefazolin B. Ampicillin-sulbactam C. Piperacillin-tazobactam D. Aztreonam E. Vancomycin

557 D. Aztreonam is active only against Gram-(-) organisms and has NO cross-reactivity with penicillins.

Ceftriaxone is commonly given as an empiric antibiotic in the treatment of bacterial meningitis. However, it must be noted that this antibiotic will NOT cover the following etiologic agent in meningitis: A. Neisseria meningitidis B. Streptococcus pneumoniae C. Listeria monocytogenes D. Group B streptococcus E. C and D

558 C. Organisms NOT covered by cephalosporins are L.A.M.E.: Listeria monocytogenes, Atypicals (chlamydia, mycoplasma), MRSA, and Enterococci.

A 10/F, diagnosed case of HSV encephalitis, was given IV acyclovir. Acyclovir is an efffective antiviral because it inhibits: A. DNA polymerase B. RNA polymerase C. Aspartate protease D. Neuraminidase E. Hemagluttinin

559 A. Acyclovir, foscarnet and ganciclovir are DNA polymerase inhibitors.

Tolerance is seen in which of the following drugs: A. Nitrates B. Dopamine C. Epinephine D. Aspirin E. AOTA

56 A. Tolerance develop with nitrates when used for more than 8 hours without interruption

A 55/M comes in due to sudden headache described as the "worst headache of his life." He is a diagnosed hypertensive, with poor compliance to medications. PE reveals nuchal tenderness (positive Kernig's sign), and a left cranial nerve palsy. Brain CT reveals diffuse subarachnoid hemorrhage in the basal cisterns. Which of the following drugs has a proven role in the management of his condition? A. Nimodipine B. Propanolol C. Captopril D. Nifedipine E. Aspirin

560 A. Nimodipine given for 21 days should be started on admission for vasospasm prophylaxis.

The drug of choice for clostridium tetani infection because of the drug's excellent antimicrobial activity and the absence of the GABA-antagonistic activity a. Penicillin G b. Metrobidazole c. Vancomycin d. Chloramphenicol

561 B. The use of penicillin (10â€"12 million units IV, given daily for 10 days) has been recommended, but metronidazole (500 mg every 6 h or 1 g every 12 h) is preferred by some experts on the basis of this drug's excellent antimicrobial activity and the absence of the GABA-antagonistic activity seen with penicillin.

a second generation Cephalosporin with intrinsic activity against anaerobes: a. cefoxitin b. cefipime c. cefaclor d. ceftriaxone 0

562 A. Cefoxitin has anaerobe coverage apart from its activity against gram positive and some gram neg infections

this oral iron compound has the highest elemental iron content: a. ferrous sulfate b. ferrous gluconate c. ferrous fumarate d. iron dextran

563 C. Ferrous sulfate (FEOSOL, others) is the hydrated salt, FeSO4×7H2O, which contains 20% iron. Dried ferrous sulfate (32% elemental iron) is also available. Ferrous fumarate (FEOSTAT, others) contains 33% iron and is moderately soluble in water, stable, and almost tasteless. Ferrous gluconate (FERGON, others) also has been successfully used in the therapy of iron-deficiency anemia. The gluconate contains 12% iron. Polysaccharide-iron complex (NIFEREX, others), a compound of ferrihydrite and carbohydrate, is another preparation with comparable absorption. The effective dose of all of these preparations is based on iron content.

which anong the following drugs should be avoided in patients with porphyria? a. phenytoin b. secobarbital c. midazolam d. sodium valproate

564 B. Phenobarbitals should be avoided in patients diagbosed with porphyria

Xenobiotic metabolizing enzymes have historically been grouped into the phase 1 reactions and the phase 2 reactions. Which is NOT TRUE regarding the former? a. enzymes carry out oxidation, reduction, or hydrolytic reactions b. enzymes lead to the introduction of what are called functional groups resulting in a modification of the drug c. the reaction process adds an -OH, -COOH, -SH, -O- or NH2 group. d. the addition of functional groups significantly increase the water solubility of the drug

565 D. Phase 1 reaction does little to the water solubility of a drug metabolized

NOT True of Phase 2 enzymes in Xenobiotic metabolism: a. they facilitate the elimination of drugs b. inactivation of electrophilic and potentially toxic metabolites produced by oxidation. c. phase 2 reactions produce a metabolite with improved water solubility d. products have decreased molecular weight

566 D. Because of conjugation of these substances, they have a higher molecular weight

majority of phase2 reactions involving Glucoronic acid conjugation are found in what cell compartment? a. ER b. cytosol c. Mitochondrial matrix d. Golgi bodies

567 A. conjugating enzymes, notably the UGTs, are all located in the endoplasmic reticulum of the cell

Notable to these drugs are its capability of inducing CP450 enzyme system in the liver except: a. Phenytoin b. Cimetidine c. Carbamazepine d. Griseofulvin

568 B. All except Cimetidine induce CYP450 enzyme complex

Flushing is a unique side effect of Nicotinamide used in hyperlipidemia. The drug that may be used to counter this effect is: a. Acetaminophen b. Aspirin c. Chlorphenamine d. Arginine

569 B. The cutaneous effects include flushing and pruritus of the face and upper trunk, skin rashes, and acanthosis nigricans. Flushing and associated pruritus are prostaglandin-mediated. Taking an aspirin each day alleviates the flushing in many patients.

The only beta-lactam antibiotcs that does not cross react with penicillins A. Ceftriaxone B. Aztreonam C. Meropenem D. Aminoglycosides E. Vancomycin

57 B. The monobactams have activity against aerobic Gram negative bacilli and dose not cross react with penicillin.

Local anesthetics block nerve conduction through inhibition of what electrolyte? a. Potassium b. Sodium c. Calcium d. All of the above

570 B. Local anesthetics block conduction by decreasing or preventing the large transient increase in the permeability of excitable membranes to Na+ that normally is produced by a slight depolarization of the membrane

a dreadful side effect, although not common, of this atypical antipsychotic is agranulocytosis. a. Quetiapine b. Clozapine c. Risperidone d. Aripiprazole

571 B. Clozapine is notorious for its side effect of agranulocytosis, although uncommon

True of Acetaminophen: a. It covalently binds to COX inhibiting prostaglandin synthesis b. very weak antiinflammatory agent at the typical daily dose of 1000 mg c. incidence of Gastrointestinal adverse effects comparable to Ibuprofen d. its inhibition of thromboxane A2 synthesis discourages its use in dengue cases

572 B. Observational studies suggest that acetaminophen, which is a , very weak antiinflammatory agent at the typical daily dose of 1000 mg is associated with a reduced incidence of gastrointestinal adverse effects compared to tNSAIDs

A condition characterized by the acute onset of encephalopathy, liver dysfunction, and fatty infiltration of the liver and other viscera known as Reye's syndrome is associated with what drug? a. Aminosalicylate b. Long chain fatty acids c. Indomethacin d. nitric oxide

573 A. Due to the association with Reye's syndrome, aspirin and other salicylates are contraindicated in children and young adults less than 20 years old with fever associated with viral illness. Reye's syndrome is characterized by the acute onset of encephalopathy, liver dysfunction, and fatty infiltration of the liver and other viscera

incidence of Reye syndrome among patients less than 20 who took Aspirin concurrent with influenza A/B infection: a. 0.1% b. 1% c. 5% d. 10%

574 A. Less than 0.1% of children who took aspirin developed Reye syndrome, but more than 80% of patients diagnosed with Reye syndrome had taken aspirin in the past 3 weeks

. A good drug combination along with thiazides by virtue of its effect on potassium retention: a. losartan b. amlodipine c. verapamil d. metoprolol

575 A. Urinary K+ loss can be a problem with thiazides. Angiotensin converting enzyme (ACE) inhibitors and angiotensin receptor antagonists will attenuate diuretic-induced loss of potassium to some degree, and this is a consideration if a second drug is required to achieve further blood pressure reduction beyond that attained with the diuretic alone.

a highly lipophilic derivative of obiquinone that has activity against plasmodium species and Pneumocystis pneumonia among patients intolerant to Co-trimoxazole: a. Atorvaquone b. Quinine c. Proguanil d. artemisinin

576 A. Atorvaquone is promising synthetic derivative with potent activity against Plasmodium species and the opportunistic pathogens Pneumocystis carinii and Toxoplasma gondii. After limited clinical trials, the FDA approved this compound in 1992 for treatment of mild-to-moderate P. carinii pneumonia in patients intolerant to trimethoprim-sulfamethoxazol

The least nephrotoxic among the aminoglycosides is: a. Tobramycin b. Kanamycin c. Amikacin d. Streptomycin

577 D. The nephrotoxic potential varies among individual aminoglycosides. The relative toxicity correlates with the concentration of drug found in the renal cortex in experimental animals. Neomycin, which concentrates to the greatest degree, is highly nephrotoxic in human beings and should not be administered systemically. Streptomycin does not concentrate in the renal cortex and is the least nephrotoxic

A fusion inhibitor used in the treatment of HIV infection a. Efavirenz b. Lamivudine c. Enfuvirtide d. Atazanavir

578 C.

In dermatologic pharmacology, approximately how many grams of topical preparation is needed to cover the body surface? a. 10g b. 20g c. 30g d. 40g

579 C. An amount of topical medication sufficient to cover affected body surfaces in repeated applications must be dispensed to the patient. A general rule is that approximately 30 g is required to cover the body surface.

Which of the following can't be used topically? A. Clindamycin B. Erythromycin C. Amoxicllin D. Fusidic acid E. Mupirocin

58 C. Topical clindamycin and erythromycin are used topically against acne. Topical fusidic acid and mupirocin are used topically against other superficial bacterial infection.

A topical preparation that are concentric spherical shells of phospholipids in an aqueous medium that may enhance percutaneous absorption. a. Liposome b. Microgrel c. Transfersome d. Micelle

580 A. Newer vehicles include liposomes and microgel formulations. Liposomes are concentric spherical shells of phospholipids in an aqueous medium that may enhance percutaneous absorption. Variations in size, charge, and lipid content can influence liposome function substantially. Liposomes penetrate compromised epidermal barriers more efficiently (Korting et al., 1991). Microgels are polymers that may enhance solubilization of certain drugs, thereby enhancing penetration and diminishing irritancy.

Theophylline has a pKa of 8.7, what is the predominant form at pH 11.3? A. Ionized and protonated B. Non-ionized and protonated C. Charged and unprotonated D. Uncharged and unprotonated E. None of the above

581 D. Above pKa, the unprotonated and uncharged form of a weak base predominates.

Some drugs like magnesium sulfate needs a loading dose in order to reach steady state at a reduced amount of time. The following are used to compute for loading dose EXCEPT? A. Clearance B. Volume of Distribution C. Minimum effective dose D. Bioavailabiloity E. No exception

582 A. Loadign dose = (Vd) (desired plasma concentration)/(bioavailabitlity) the minimum effective dose is usually the desired plasma concentration. Clearance is used to calculate the maintenance dose

A 5 year old known case of seizure disorder maintained on phenytoin was rushed to the ER due to an asthma attack. IV theophylline was given to control the asthma using usual dose computations. What are the consequences of the drug-drug interactions? A. The patient will be more prone to the toxic side-effects of theophylline. B. The patient may be at risk for a seizure attack. C. The patient is not being treated adequately for his asthma. D. There will be an increased in plasma levels of phenytoin. E. No drug interactions exist beween theophylline and phenytoin

583 C. Phenytoin is an inducer, which will lower theophylline levels. Asthma may not be adequately treated with this monotherapy.

Which of the following substances will increase the available adrenergic neurotransmitter in the synaptic cleft by inibiting reuptake? A. Botulinum B. MAOIs C. Cocaine D. Guanethidine E. Reserpine

584 C. Cocaine and TCA prevents reuptake of norepinephrine. Review the inhibitors for each step in the process of neurotransmitter release for both the sympathetic and parasympathetic system.

Which of the following drugs for glaucoma increased aqueous humor outflow through the non-conventional pathway without decreaing the secretion of aqueous humor by the ciliary body? A. β-blockers B. Prostaglandin analogues C. α-adrenergic agonist D. Carbonic anhydrase inhibitors E. Mannitol

585 B. Beta blockers, CAIs and osmotic agents (mannitol) decreases aqueous humor production. Alpha adrenergic agonist increases outflow and also decreases the secretion of the ciliary body. Prostaglandins increased outflow but has no effect on production.

Which of the following beta-blockers demonstrates the relative selectivity in the manner described? A. Betaxolol: relatively selective for β2-receptors B. Timolol: relatively selective for β1-receptors C. Levobunolol: relatively selective for β2-receptors D. Betaxolol: relatively selective for β1-receptors E. Levobunolol: relatively selective for β1-receptors

586 D. Because of its relatively β1 selectivity, betaxolol has fewer side effects. Timolol and levobunolol are non- selective β-blockers

What hypertensive drug commonly used for the outpatient treatment of preeclampsia is prominent for its side-effect of hemolytic anemia with a positive Coomb's test? A. α1 adrenergic agonist B. α2 adrenergic antagonist C. α1 adrenergic agonist D. α2 adrenergic antagonist E. non-selective α antagonist

587 C. Methyldopa is a selective alpha 2 agonist.

Which of the following antiarrhythmics primarily exerts its action on phase 4 of the cardiac action potential, prolonging PR interval? A. Procainamide B. Lidocaine C. Propanolol D. Amiodarone E. Verapamil

588 C. Class 2 anti arrhythmics or B blockers exert its effort on phase 4 and prolongs the PR interval

Which of the following loop diuretics is not prone to induce an allergic response? A. Furosemide B. Bumetanide C. Brinzolamide D. Torsemide E. Ethacrynic acid

589 E Ethacrynic acid does not contain a sulfa group.

Fifith Generation cephalosporin with activity against Pseudomonas aeruginosa A. Ceftaroline B. Ceftobiprole C. Ceftazidime D. Cefepime E. Cefixime

59 B. Both A and B are fifth generation but only B has activity against Pseudomonas.

Which of the following is a major toxicity associalted with gemfibrozil therapy? A. Bloating and constipation B. Cholelithiasis C. Hyperuricemia D. Myopathy E. Severe cardiac arryhtmia

590 B. Gemfibrozil is associated with cholelithiasis. Myopathy is only expected in combination with statins.

Which of these anesthetics is known to cause acute porphyria? A. Nitrous oxide B. Halothane C. Ketamine D. Thiopental E. Propofol

591 D. Barbiturates such as thiopental can cause acute porphyria.

Which of the following is the drug of choice for partial seizures? A. Valproic acid B. Phenobarbital C. Lamotrigine D. Ethosuximide E. Topiramate

592 C. Carbamazepine, phenytoin and lamotrigine are drug of choices for partial seizures

Which drugs reduces the risk of suicide? A. Clozapine B. Clonidine C. Clonazepam D. Clofazimine E. Clofibrate

593 A. Clozapine is the anit-psychotic that reduces the risk of suicide.

In the acute settings the following drugs are useful for the emergent management of thyroid storm EXCEPT? A. Propanolol B. KI C. Propylthiouracil D. Methimazole E. Hydrocortisone

594 D. Propylthiouracil, iodides and propanolol are the first line drugs for thyroid storm. Hydrocortisone may be added for realtive adrenal insufficiency in the acute attack. Methimazole may be used, but is not usually given in the acute therapy of thyroid storm and PTU is preferred.

Which of the following insulin preparations mimics the secretion pattern of insulin by the normal pancreas in the fasting state? A. Lispro B. Regular C. NPH D. Glulisine E. Detemir

595 E The long acting insulin mimics the normal secretion of the pancreas in the non-fed state.

Which of the following antibiotics does NOT follow time-dependent killing action? A. Amoxicillin B. Aztreonam C. Metronidazole D. Bacitracin E. Vancomycin

596 C. The broad class of Beta lactams including the class of monobactams and aztreonams exhibit time- dependent killing action. Bacitracin and vancomycin that also act on the cell wall exhibit time dependent killing. Metronidazole, FQ and AG are concentration- dependent.

Which of the following anti-malarial is contraindicated for pregnant women? A. Chloroquine B. Quinine C. Quinidine D. Primaquine E. Mefloquine

597 D. The fetus is G6PD deficient, so primaquine cannot be used. To achieve radical cure for pregnant women, primaquine is given after delivery of the baby.

Which of the following anti-neoplastic drug is highly lipophilic, use for the treatment of cancers that are not readily permeable through vascular channels? A. Bleomycin B. Busulfan C. Carmustine D. Cyclophosphamide E. Carboplatin

598 C. Carmustine and Lomustine are highly lipophilic, used to treat brain cancer.

Which novel and innovative drug is used to treat rheumatoid arthritis through its action as an antibody against the protein CD20, found in the surface of B-cells? A. Abciximab B. Rituximab C. Omalizumab D. Infliximab E. Trastuzumab

599 B. Abciximab - Glycoprotein Iib/IIIa (anti-platelet) Omalizumab - IgE (asthma) Infliximab - TNF (inflammatory bowel disease) Trastuzumba - Her2neu (Breast cancer)

Propylthiouracil and Methimazole are both used in the treatment of hyperthyroidism. They are thioamides which inhibits the enzyme thyroid peroxidase necessary for the synthesis of thyroid hormone. Which of the following is false about propylthiouracil? A. PTU is more highly protein bound than methimazole B. PTU has a longer halflife and requires less frequent dosing than methimazole C. PTU is less potent than methimazole. D. PTU has the added activity of preventing peripheral conversion of T4 to T3 E. None of the Above

6 B. The halflife of PTU is 1.5hrs and methimazole is 6- 8hrs. PTU is generally given every 6-8hrs while methimazole is given once a day. Thus methimazole is more convenient for patients. Methimazole is also 10 times more potent than PTU. Source: Katzung Basic and Clinical Pharmacology 11th ed p671

Blocks the final common pathway of platelet aggregation A. ASA B. Abciximab C. Clopidogrel D. Both A and B E. AOTA

60 B. ASA - inhibits COX Abciximab - GPIIBIIIA inhibitor Clopidogrel - ADP receptor inhibitor

Which of the following substances can be adsorbed by activated charcoal, effectively decontaminating the gastrointestinal tract? A. Barbiturates B. HCl C. Lithium D. Cyanide E. Lye

600 A. Remember that activated charcoal is effective only to organic substances, so if it is inorganic like ions or acids/bases, it is not effective. Alcohol, though organic does not enhance its elimination with activated charcoal.

Salmeterol has a slower onset of action as compared with formoterol in terms of producing bronchodilation. This is because of: A. Increased lipophilicity of salmeterol B. Faster degradation of salmeterol C. More efficient absorption of formoterol D. Higher affinity of formoterol to beta adrenergic receptors E. A and D

601 A. The quickier onset of action of formoterol can be explained by diffusion microkinetic model. This model suggests that formoterol and salmeterol are both retained in the lipid bilayer adjacent to the beta2-adrenergic receptor. Because of salmeterol's lipophilicity, it associates more with the lipid bilayer than with the receptor, accounting for its slower onset of action.

If the plasma concentration of a drug declines with “first order kinetics,†this means that: A. The halflife is constant regardless of plasma concentration B. The drug is largely metabolized in the liver after oral administration and has low bioavailability C. The rate of elimination is proportionate to the rate of administration at all times D. The drug has a high volume of distribution E. The drug is eliminated with a constant concentration regardlesss of the plasma concentration

602 A. First Order Kinetics : elimination of the Drug is directly proportional to its plasma concerntration. its dependent on its half life, which remains constant. First order implies that no matter how much concerntration of the Drug u give it will be eliminated 50% by its first half- life

A 35 year old female underwent treatment for endometriosis for 12 months. She then developed male-pattern hair growth, deepening of the voice and acne. The drug given is most likely: A. GnRH agonist B. Danazol C. Combined oral contraceptive pills D. Medroxyprogesterone acetate E. GnRH antagonist

603 B. Danazol is a derivative of the synthetic steroid ethisterone, a modified testosterone, also known as 17-alpha-ethinyl testosterone. Danazol inhibits ovarian steroidogenesis resulting in decreased secretion of estradiol and may increase androgens. This drug causes androgenic side effects, such as hirsutism, deepening of the voice, acne and oily skin.

In the case above, which of the following patient's manifestations may be IRREVERSIBLE? A. Acne B. Male-pattern hair growth C. Deepening of the voice D. All of the above E. B and C

604 C. All the other side effects of danazol is reversible, except for deepening of the voice.

Which of the following anti-hypertensives has been recalled from the market due to its side effect of producing rebound suicide? A. Guanethidine B. Phenoxybenzamine C. Reserpine D. Trimetaphan

605 C. Reserpine is an indole alkaloid antipsychotic and antihypertensive drug that has been used for the control of high blood pressure and for the relief of psychotic symptoms. Reserpine mediated depletion of monoamine neurotransmitters in the synapses is often cited as evidence to the theory that depletion of the neurotransmitters causes subsequent depression in humans

Organs involved in xenobiotic metabolism are as follows, EXCEPT: A. Liver B. Small intestines C. Lung D. Nasal mucosa E. None of the above

606 E Xenobiotic metabolizing enzymes are found in most tissues in the body with the highest levels located in the tissues of the gastrointestinal tract (liver, small and large intestines). Other organs that contain significant xenobiotic-metabolizing enzymes include the tissues of the nasal mucosa and lung, which play important roles in the first-pass metabolism of drugs that are administered through aerosol sprays.

A pure opioid antagonist with a greater affinity for μ receptors and used for acute opioid overdose: A. Morphine B. Naloxone C. Codeine D. Dextrometorphan E. Diphenoxylate

607 B. Naloxone is a pure opioid anatagonist and is used to counter the effects of opiate overdose, for example heroin or morphine overdose. The other drugs are opioid agonists.

Given a hypothetical species of bacteria that is resistant to antibacterial agents that prevent translocation during translation, which of the following can be used effectively for this bacterial infection? A. Chloramphenicol B. Clindamycin C. Erythromycin D. A and B E. B and C

608 A. Chloramphenicol inhibits peptidyl transferase. Clindamycin and erythromycin inhibits translocation.

The drug of choice for the treatment of bladder and bowel atony is: A. Neostigmine B. Metoclopramide C. Pilocarpine D. Bethanecol E. None of the above

609 D. Bethanechol is a muscarinic agonist that is administered orally to promote GI and bladder motility, usually used in post-op ileus and diabetic neurogenic bladder.

Which of the ff statements regarding drug elimination is correct? A. Elimination of parent molecules terminates the drug's action for drugs with active metabolites B. First order elimination occurs when the concentration of the drug decreases exponentially over time C. Zero order kinetics exhibits the characteristic half life of most drugs D. First order kinetics occurs when drugs have saturated their elimination mechanisms E. Phenobarbital is an example of a drug that exhibit zero order kinetics

61 B. Elimination of parent molecules does not terminate the drug's action for drugs with active metabolites. First order kinetics exhibits the characteristic half life of most drugs. Zero order kinetics occurs when drugs have saturated their elimination mechanisms. Phenobarbital does not display zero order kinetics

Beta-lactamase inhibitors are usually given with beta-lactam containing antibiotics, such as penicillins. This is done to produce a favorable drug interaction, specifically: A. Synergism B. Potentiation C. Additive D. Antagonism E. None of the above

610 B. Synergism is giving two active drugs and the combined effect is greater than the sum of their effects (1+1=3). Potentiation is the addition of an inactive drug to an active drug, leading to increased effect of the active drug (1+0=3).

A 67-year old male, on warfarin maintenance after undergoing valve replacement surgery, was brought to the ER due to unresolving epistaxis and easy bruising noted for the past week. In asking the history, the following substances are also being taken by the patient. Which of them contributed to the patient's condition? A. Cimetidine B. Digoxin C. Griseofulvin D. Procainamide

611 A. Cimetidine inhibits metabolism of warfarin, thereby prolonging its effect.

Clopidogrel exerts its antithrombotic effect through which of the following mechanisms: A. Irreversible inhibition of ADP receptor B. Inhibition of thromboxane synthesis C. Reversible blockade of glycoprotein IIb/IIIa D. Conversion of plasminogen to plasmin E. Posttranslational modification of vitamin K-dependent clotting factors

612 A. Clopidogrel works by irreversibly inhibiting a receptor called P2Y12, an adenosine diphosphate (ADP) chemoreceptor on platelet cell membranes.

Sympathomimetic drug that causes mydriasis without cycloplegia: A. Atropine B. Tropicamide C. Phenyephrine D. Pilocarpine E. Timolol

613 C. Atropine and tropicamide cause mydriasis and cycloplegia. Timolol has no mydriatic effect.

A 33-year old farmer, after 1 hour of spraying insecticides, started to note generalized body weakness, drowsiness, shortness of breath, numbness and severe epigastric pain. After 10 hours, he was brought to the ER due to loss of consciousness. He was soaking in sweat, drooling with saliva and diarrheic. What should be administered to this patient? A. Atropine B. Pralidoxime C. Neostigmine D. A and B E. B and C

614 A. The patient had an insecticide poisoning. Insecticides usually contain organophosphates or carbamates. Before 6 hours, pralidoxime and atropine should be both administered. In this patient, 10 hours has already elapsed, hence pralidoxime can no longer be effective.

Lactulose is the most frequently administered laxative in post-MI patients. It is classified as: A. Bulk-forming laxative B. Osmotic laxative C. Stool softener D. Emollient E. None of the above

615 B. Lactulose is an osmotic agent. Bulk forming agents include insoluble fibers (psyllium). Stool softeners include docusate Na. Emollients include mineral oil.

A hypertensive patient was found to have elevated triglycerides, normal LDL and decreased HDL levels. Which of the following is most efficacious for this patient? A. Simvastatin B. Ezetimibe C. Colestipol D. Fenofibrate E. Orlistat

616 D.Fibrates are indicated for hypertriglyceridemia. Fibrates typically lower triglycerides by 20% to 50%. Level of the good cholesterol HDL is also increased. Fibrates may decrease LDL, though generally to a lesser degree than statins.

Which of the following cephalosporins is highly effective against pseudomonas? A. Cefazolin B. Cefuroxime C. Ceftazidime D. Cefaclor E. Ceftriaxone

617 C. Ceftazidime, 3rd gen cephalosporin, is considered to be most efficacious agaianst pseudo monas among cephalosporins.

Which of the following antivirals is a reverse transcriptase inhibitor that is used in the treatment of Hepatitis B infection? A. Amantadine B. Ganciclovir C. Lamivudine D. Interferon alpha E. Acyclovir

618 C. Lamivudine is used in hepatitis B as well as in HIV infection.

Which of the following is NOT true about insulin action? A. It stimulates glycogen synthesis in muscle fiber B. It inhibits lipolysis in the adipocyte C. It stimulates fatty acid synthesis in the hepatocytes D. It binds to G receptor to increase intracelluar cAMP

619 D. The insulin receptor is a transmembrane receptor that is activated by insulin, IGF-I, IGF-II and belongs to the large class of tyrosine kinase receptors.

27-year old female presents to the ER approximately one hour after swallowing 70 propranolol tablets with suicidal intent. At the time of assessment she is drowsy (GCS 13) with a heart rate of 46 bpm and BP 100/60. What drug/class can be given by virtue of physiologic antagonism? A. Beta agonists B. Alpha agonists C. Glucagon D. Diazepam E. Atropine

62 C. The physiological antagonism is related to the fact that glucagon activates cAMP through non-adrenergic pathways. Because the adrenergic pathways are blocked by propanol, glucagon essentially uses a back door to enhance myocardial activity.

Disseminated intravascular coagulation (DIC) is a condition characterized by widespread stimulation of thrombosis. Which of the following is NOT consistent with the clinical findings in DIC? A. Increased PTT B. Increased PT C. Decreased fibrinogen D. Increased fibrin split products E. None of the above

620 E DIC clinical findings: â†" Fibrinogen, â†' fibrin split products, â†" platelets, â†' PT, â†' PTT, fragmented red blood cells on peripheral smear

A 53 year old woman with newly diagnosed type 2 diabetes presents to mergency department complaining of vomiting, severe headache, dizziness, blurry vision, and DOB. She says that she had been at a cocktail party when the symptoms began. her skin is notably flushed on physical examination. Which of the following medications is responsible for this reaction? A. Acarbose B. Glipizide C. Glyburide D. Metformin E. Tolbutamide

621 E This is a disulfiram like reaction after drinking alcohol at a cocktail party. TOLBUTAMIDE is associated with this kind of reaction

A 65 year old man comes to the physician because he awakens to urintae several times per night and has developed problems starting and stopping his stream of urine. A biopsy of the prostate shows enlargement and dilation of the prostate glands but no dysplasia. Which of the following is the most appropriate pharmacological treatment for this patient? A. Finasteride B. Flutamide C. Ketoconazole D. Spirinolactone E. Yohimbine

622 A. BPH. Finasteride is a 5 alpha reductase inhibitor that inhibits conversion of testosterone to DHT.

A 45 year old man who has received long treatment for schizophrenia has recently been noted to display involuntary movements that include lateral deviations of the jaw and "fly catching" motions in his tongue. Which of the following agents is the most likely cause of his involuntary movements? A. Clozapine B. Fluphenazine C. Lithium D. Risperidone E. selegiline

623 B. this is TARDIVE DYSKINESIA, which is a complication of long term antipsychotic therapy with the older agents like, FLUPHENAZINE.

Which of the following drugs can be used to reduce intraocular pressure in the treatment of glaucoma? A. Pilocarpine B. Acetazolamide C. Neostigmine D. Timolol E. All of the above

624 E

Dantrolene is the drug of choice to treat malignant hyperthermia caused by succinylcholine because? A. It blocks Ca release from sarcoplasmic reticulum B. It induces contraction of skeletal muscle C. It increases the rate of succinylcholine metabolism D. Succinylcholine binding to nicotinic receptors is antagonized by dantrolene E. Dantrolene acts centrally to reduce fever.

625 A.

In the kidney, which one of the following is inhibited by thiazide diuretics? A. Sodium chloride reabsorption in the early part of the distal convuluted tubule B. Water removal from intracellular space by osmosis C. Reabsorption of Calcium D. Aldosterone action on the nephron E. Excretion of chloride

626 A. Thiazide diuretics inhibit sodium chloride reabsorption in DCT.

Which of the following agents will increase myocardial contractility with the LEAST effect on total peripheral resistance? A. Epinephrine B. Phenylephrine C. Terbutaline D. Dobutamine E. Carbachol

627 D. Dobutamine increases myocardial contractility, but it has little effect on TPR. Beta 1 selective.

Which of the following antihypertive agents commonly used in hypertension in pregnancy is contraindicated to patients having myocardial ischemia? A. Methyldopa B. Methergine C. Clonidine D. Hydralazine E. Magnesium sulfate

628 D. Hydralazine is CI in ptients with myocardial ischemia.

Which of the following beta blocker drugs has/have intrinsic sympathomimetic activity? A. Timolol B. Acebutolol C. Pindolol D. Betaxolol E. B and C

629 E Pindolol and Acebutolol have intrinsic sympathomimetic activity.

A patient was put on standrard dose of warfarin therapy, aftter 1 week he suddenly experienced hematochezia. Which of the ff drugs he is also currently taking could have caused this? A. Valproic acid B. Ethanol C. Carbamazepine D. Isoniazid E. Griseofulvin

63 A D. INH is also an inhibitor. Rifampicin is an inducer. The patient probably is taking a CYP450 inhibitor such as valproic acid that decreased the metabolism of warfarin leading to its toxic side effects. All the rest of the drugs are inducers.

Which of the following nonselective , irreversible alpha blocker is the drug of choice as a preoperative agent used in Pheochromocytoma? A. Phentolamine B. Phenoxybenzamine C. Prazosin D. Yohimbine E. Clonidine

630 B. DOC for pheochromocytoma is Phenoxybenzamine, it is a nonselective, IRREVERSIBLE alpha blocker.. Phentolamine is a reversible alpha blocker.

Succor Mendiola was diagnosed to have glaucoma. Which of the following drugs induces ciliary muscle contraction thereby opening the trabecular meshwork and increasing the outflow? A. Timolol B. Mannitol C. Epinephrine D. Latanoprost E. Pilocarpine

631 E Pilocarpine induces ciliary muscle contraction thereby opening the trabecular meshwork and increasing the outflow

Which of the following vasodilators has the common side effect of headache, hypotension and CYANIDE toxicity? A. Fenoldopam B. Nitroprusside C. Nifedipine D. Dopamine E. Minoxidil

632 B. Nitroprusside is a parenteral vasodilator known to cause CYANIDE toxicity.

Which of the following drug/s is/are notorius in causing gingival hyperplasia? A. Nifedipine B. Cyclosporine C. Phenytoin D. Verapamil E. All of the above

633 E

Which of the following statements concerning barbiturates is true? A. Barbiturates can increase bleeding time when administered to patients taking anticoagulants B. Barbiturates are contraindicated in patients with acute intermittent porphyria C. Patients tolerant to the therapeutic actions of barbiturates are also tolerant to analgesic effect of morphine D. Barbiturates are used to prevent withdrawal symptoms associated with heroin dependence E. All of the above

634 B.

Which of the following has the lowest incidence of extrapyramidal symptoms but is known to cause agranulocytosis? A. Clozapine B. Fluphanazine C. Chlorpromazine D. Pimozide E. Haloperidol

635 A. Clozapine is a new drug with least extrapyramidal effect but is known to cause agranulocytosis.

Which of the following statements about the mechanism of action of ipratropium? A. It acts cebtrally to decrease vagal Ach release B. It inhibits pulmonary Ach receptors C. It decreases mast cell release of histamine D. It blocks the action of histamine at H1 receptors E. None of the above

636 B.

Which of the following anti TB drugs is known to cause hyperuricemia and is noted to be the most hepatotoxic? A. Rifampicin B. Ethambutol C. Pyrazinamide D. Streptomycin E. Isoniazid

637 C.

A 5 year old boy with Trisomy 21 was diagnosed to have acute lymphoblastic leukemia. What is the most important drug in the therapeutic regimen of this disease entity? A. Rituximab B. Cytarabine C. Idarubucin D. Asparaginase E. Prednisone

638 D. Asparaginase, depletes serum asparagine. It is the main and the most impt drug in the management od ALL.

Which of the following ergot alkaloids is known to cause retroperitoneal fibrosis? A. Methylsergide B. Dihydroergotamine C. Methylergonovine D. Sumatriptan E. None of the above

639 A.

Which of the ff teratogenic drugs causes atrialization of the right ventricle? A. Misoprostol B. Lithium C. Ethanol D. Penicillamine E. Phenytoin

64 B. Ebstein's anomaly from lithium is a congenital heart defect in which the septal and posterior leaflets of the tricuspid valve are displaced towards the apex of the right ventricle of the heart. There is subsequent 'atrialization' of a portion of the morphologic right ventricle

Which of the following medication-rescue therapy pair is INCORRECT? A. Methotrexate : Leucovorin B. Cyclophosphamide : MESNA C. Doxorubicin : Dexrazoxane D. Cisplatin : Amifostine E. None of the above

640 E

Allen Dimo, is a 38 y/o G8P8(6208), known hypertensive and diabetic, turned out to be pregnant for 5 weeks already. What anti-hypertensive medication should you AVOID to give to the patient? A. Propranolol B. Captopril C. Hydralazine D. Methyldopa E. None of the above

641 B. Captopril is an ACE inhibitor and its use during gestation may result in IUGR, acute fetal renal failure, PDA, and even fetal death.

Assuming that the patient above is NOT pregnant. What is the best medication for her? A. Propranolol B. Captopril C. Hydralazine D. Methyldopa E. None of the above

642 B. ACE inhibitors block the formation of Angiotensin II which is a potent vasoconstrictor of the afferent and efferent Renal blood vessels. When administered to the patient, it prevents diabetic nephropathy by increasing blood flow to the kidneys.

Excessive use of Nitroprusside may result in Methemoglobinemia. The antidote for this condition is? A. Methylene blue B. Silver Nitrate C. 100% oxygen D. Dialysis E. Blood transfusion

643 A. Methylene blue

This is the anti-Asthma drug which is only of worth if given as prophylaxis. A. Salbutamol B. Budesonide C. Salmeterol D. Cromolyn E. Hydrocortisone

644 D.

One of the following drugs causes an increase in metabolism of other drugs when administered simultaneously. A. Valproic Acid B. Rifampicin C. Venlafaxine D. Ritonavir E. Ketoconazole

645 B. Rifampicin is an inducer. All other choices are inhibitors.

Before intubation, prior to surgery, a 66 year-old male Asthmatic was injected with Succinylcholine. He was noted to remain apneic and paralyzed for an extended period of time. The patient most likely A. Received an excessive dose of the drug B. has been recieving aminoglycosides C. has impaired renal function D. had an allergic reaction E. has pseudocholinesterase deficiency

646 E Succinylcholine is being metabolized by the enzyme pseudocholinesterase, hence deficiency or lack of this enzyme will lead to extended drug effect of Succinyhlcholine.

What is the drug of choice for trigeminal neuralgia? A. Lamotrigine B. Gabapentin C. Pregabalin D. Carbamazepine E. B or C

647 D. SIMILAR TO PREVIOUS BOARD EXAM CONCEPT/PRINCIPLE

Which among the lipid-lowering medicatins causes flushing? A. Statins B. Niacin C. Cholestyramine D. A and B E. B and C

648 B.

Which anti-parasitic drug exerts its actions on the microtubules of helminths? A. Mebendazole B. Stibogluconate C. Ivermectin D. Diloxanide Furoate E. Metronidazole

649 A. SIMILAR TO PREVIOUS BOARD EXAM CONCEPT/PRINCIPLE

A patient came to the ER with diarrhea, tremors and excessive sweating. HR is 50, BP is 120/70, RR is 22. Pupils are 1-2 mm. What drug can be given to the patient? A. Naloxone B. Neostigmine C. Diazepam D. Atropine E. EDTA

65 D. The patient is exhibiting symptoms of organophosphate poisoning with increased secretion of all body fluids plus bradycardia, miosis and skeletal muscle excitation. Antidote is Atropine and Pralidoxime

Zidovudine is a very remarkable drug for HIV. Howeve,r precaution for its use should be observed because of this side effect. A. pancreatitis B. diarrhea C. myelosuppression D. Pulmonary fibrosis E. Optic neuritis

650 C. SIMILAR TO PREVIOUS BOARD EXAM CONCEPT/PRINCIPLE

A 50 year old female consulted due to palpitations. This was accompanied by frequent bowel movements and heat intolerance. You gave her Radioactive iodine and you know that its most common side effect is? A. Papillary Thyroid CA B. Medullary Thyroid CA C. Rebound hyperthyroidism D. Permanent hypothyroidism E. Rebound tachycardia

651 D. RAI may induce production of cancer cells but ALL patients administered with RAI will suffer from permanent hypothyroidism and will require exogenous thyroxine as supplement

Which among the following diuretics can also be used for mountain sickness? A. Acetazolamide B. Furosemide C. Thiazide D. Mannitol E. None

652 A. Carbonic anhydrase inhibitors

Cimetidine is used as anti-PUD drug and it acts as a/an? A. mucuprotectant B. H1 blocker C. pH neutralizer D. A and B E. None

653 E Cimetidine is a H2 blocker

Which among the following is/are opioid agonist? A. Hyoscine B. Metoclopramide C. Loperamide D. Attapulgite E. AlMg(OH)

654 C. Loperamide is an opioid that acts on Kappa receptors to increase gastric emptying time.

After the oath-taking, you decided to establish a clinic on your province. A 63 year-old farmer was brought to you, soaked in sweat and salivating. When you asked his son about the history, you were told that he was found in that condition at the middle of the field, which is an 8-hour travel from your clinic, with a bottle of Malathion on his side. Assuming you have all the following medications in hand, what is the most appropriate to give? A. Pralidoxime B. Physostigmine C. Atropine D. Scopolamine E. Atracurium

655 C. The antidote for organophosphate poisoning is Pralidoxime. BUT the golden period of 6 hours had already passed. And the best medication to give in this case is ATROPINE.

On your clinic, an 18 month old infant was brought for consult due to "blank stares" with a duration of 5-10 seconds. The best medication to be given for this case is A. Phenobarbital B. Valproic acid C. Ethosuximide D. Levepiracetam E. Carbamazepine

656 C. This is absence seizure and DOC is Ethosuximide

You encountered a 2 year old patient suffering from an acute asthmatic attack. You immediately nebulized him with Salbutamol. Knowing its mode of action, you would expect that the patient may have A. Seizures B. Dry mouth C. Tachycardia D. Somnolence E. Excessive salivation

657 C. Salbutamol is a short-acting beta agonist and may stimulate beta 1 receptors of the heart, leading to transient tachycardia.

Furosemide is a loop diuretic that is known for its ototoxicity. Which among the following drugs should you AVOID administering it simultaneously? A. Penicillin B. Ceftriaxone C. Clindamycin D. Kanamycin E. Erythromycin

658 D. Kanamycin is an aminoglycoside is very ototoxic

Which among the following drugs is commonly used in controlling the manic phase of a bipolar patient? A. Lithium B. Selegiline C. Reserpine D. Haloperidol E. Biperiden

659 A.

On a patient with signs of shock, At a dose of 5 mcg/kg/min, dopamine will predominantly have what effect? A. Increased peripheral vascular resistance B. Activation of the RAAS C. Inotropic effect D. Renal vasodilation E. Arryhthmias

66 C. Dopamine has different dose dependent actions. At 1-2 mcg/kg/min, it preferentially activates D1 receptor. At 2-10 mcg, it activates B1 receptor At >10 mcg, it activates A1 receptors

Being an IM resident, you are assigned to watch over a patient who is undergoing chemotherapy. Which drug is best to be given to prevent post-chemo vomiting? A. Metoclopramide B. Hyoscine N-Butyl-bromide C. Ondansteron D. Methylergonovine E. Methyrgine

660 C. Ondansetron is a 5TH-3 anatagonist and is the drug of choice for post-chemo vomiting.

This is time of drug solubility and absorption and clearance? A. Ionized drugs are better absorbed in the GIT B. Polar drugs have decreased clearance C. Non-ionized drugs are better absorbed in the GIT D. Non-polar drugs have increased clearance E. None of the above

661 C. polar, ionized - less soluble in lipids, excreted; non - polar, unionized - more soluble, absorbed

Route of administration with instantaneous absorption, bypasses first pass effect with I00% bioavailabilty? A. IV B. SL C. Oral D. Rectal E. None of the above

662 A. Compared with other routes of administration, the intravenous route is the fastest way to deliver fluids and medications throughout the body.

Drugs which undergo zero order kinetics, except? A. Warfarin B. Heparin C. Hydralazine D. Aspirin E. None of the above

663 C. WHATPET - Warfarin, Heparin, ASA, Tolbutamide, Phenytoin, Ethanol, Theophylline

In this phase of clinical trials, the drig is evaluated in a moderate number of target patients to test dose efficacy of the drug? A. I B. II C. III D. IV E. None of the above

664 B. I - normal volunteers, pharmacokinteics and dose sensitive acute effects II - moderate number of PATIENTS, dose efficacy III - large # of patients, toxicities IV - post - marketing, idiosyncratic reactions

Which of the following drugs is a direct acting cholinoninetic used for treatment of glaucoma, sjoqren and sicca syndrome? A. Betanechol B. Pilocarpine C. Physostigmine D. Neostigmine E. None of the above

665 B. Bethanecol for bowel and bladder atony

Muscarinic antagonists used for the treatment of parkinsonism, except? A. Trinexyphenidyl B. Triamterene C. Benztropine D. Biperiden E. None of the above

666 B. Triamterene - potassium sparing diuretic

Drug of choice for uncomplicated essential hypertension? A. Furosemide B. Metoprolol C. HCTZ D. Captopril E. None of the above

667 C. Thiazide diuretics are first line of treatment for uncomplicated hypertension.

B - Blocker with intrinsic sympathomimetic activity? A. Acebutolol B. Atenolol C. esmolol D. metroprolol E. None of the above

668 A. Acebutolol and Pindolo has ISA.

What is the MOST-SLECTIVE alpha 1 selective antagonist used in the treatment of BPH? A. Prazosin B. tamsulosin C. Terazosin D. Doxazosin E. all of the above

669 B. However, all of the mentioned drugs are alpha-1 selective antagonists used in the treatment of BPH (Prazosin, Tamsulosin, Terazosin, Doxazosin). Tamsulosin is the most selective alpha-1 selective antagonist. . Tamsulosin is a selective α1 receptor antagonist that has preferential selectivity for the α1A receptor in the prostate versus the α1B receptor in the blood vessels. When alpha 1 receptors in the bladder neck and the prostate are blocked, this causes a relaxation in smooth muscle and therefore less resistance to urinary flow. Due to this the pain associated with BPH can be reduced.

Which of the ff drugs acts to improve aqueous humor outflow in patients with glaucoma? A. Pilocarpine B. Mannitol C. Apraclonidine D. Timolol E. Acetazolamide

67 A. Pilocarpine is a cholinomimetic agent that contracts the ciliary muscle opening the trabecular meshwork. All of the other drugs decreases secretion of aqueous humor

Mechanism of action of cyanide poisoning? A. Inhibits complex III of ETC B. Uncoupler C. Inhibits oxidative bursts D. Inhibits complex IV of ETC E. None of the above

670 D. Complex I - amytal, rotenone; Complex II - malonate; Complex III - antimycin A, Dimercaprol; Complex IV - CN, CO, Na azide; Complex V - Oligomycin Uncoupler - ASA, 2,4 - DNP

An anticoagulant which has a more selective effect on factor Xa? A. hepanin B. Lepirodin C. enoxaparin D. warfarin E. None of the above

671 C. It is an anticoagulant used to prevent and treat deep vein thrombosis or pulmonary embolism, and is given as a subcutaneous injection (by a health care provider or the patient).

Antihyperlipidemic agent which is contraindicated in patients prone to GB stone formation? A. Genifibrozil B. Niacin C. Ezetimibe D. Simvastatin E. None of the above

672 A. Fibrates and cholestipol increases risk of forming gallstones.

Mechanism of action of chloroquine except:? A. Suppression of B-Lymphocyte response to mitogens B. Free-Radical Trapping C. Lysosomal enzyme stabilization D. Decreased leukocyte chemotaxis E. None of the above

673 A. Chloroquine suppress T - lymphocyte response

Mechanism of action of diazepam? A. Increases duration of Cl- channel opening B. Increases frequency of Cl- channel opening C. Agonist at 5HT1a and D2 receptors D. Blocks voltage gated Na+ channels E. None of the above

674 B. Increases frequency of channel opening

Drug of choice for partial seizures? A. Diazepam B. Phenobarbital C. Valproic Acid D. Carbamazepine E. None of the above

675 D. Diazepam - s. epilepticus Phenobarbital - s. epilepticus Valproic Acid - absence, myoclonic Carbamazepine - partial seizures

Causes analgesia, amnesia and catatonia with retained conciousness? A. Midazolam B. Ketamine C. Thiopental D. Fentanyl E. None of the above

676 B. Dissociative anesthesia

Early morning hyperglycemia characterized by development of hypoglycemia around 3AM and pre-breakfast hyperglycemia:? A. Waning of insulin dose B. Dawn Phenomenon C. Somogyi Effect D. Combined Waning + Dawn E. None of the above

677 C. Waning of insulin dose - low dose insulin Dawn Phenomenon - wrong type of insulin Somogyi Effect - high dose of insulin

Antibiotic which inhibits transpeptidation by binding to 50s subunit, used in the treatment of meningitis secondary to strep, H.inf, Neisseria neningitidis, causes aplastic anemia as its most common idiosyncratic effect? A. Chloramphenicol B. Erythromycin C. Linezolid D. Clindamycin E. None of the above

678 A. Chloramphenicol is a bacteriostatic drug that stops bacterial growth by inhibiting protein synthesis. Chloramphenicol prevents protein chain elongation by inhibiting the peptidyl transferase activity of the bacterial ribosome.

Mechanism of action of mebendazole? A. Inhibits microtubule assembly B. Inhibits glucose uptake in rematudes C. Causes dipolarization induced paralysis D. Increase permeability of membrane to Ca2+ causing muscle paralysis E. None of the above

679 A. Mebendazole causes degenerative alterations in the tegument and intestinal cells of the worm by binding to the colchicine-sensitive site of tubulin, thus inhibiting its polymerization or assembly into microtubules. The loss of the cytoplasmic microtubules leads to impaired uptake of glucose by the larval and adult stages of the susceptible parasites, and depletes their glycogen stores.

All fo the ff are side effects of lithium therapy EXCEPT? A. Tremors B. Hyperthyroidism C. Nephrogenic diabetic insipidus D. Edema E. Sinus node depression

68 B. Lithium decreases thyroid function in most patients exposed to the drug and some show symptoms of hypothyroidism. Tremors is the most common adverse effect and occurs even on therapeutic doses.

Drug of choice for prostatic carcinoma? A. gemcitabine B. Paditaxel C. Cisplatin D. Leuprolide E. None of the above

680 D. Leuprolide acts as an agonist at pituitary GnRH receptors. By interrupting the normal pulsatile stimulation of, and thus desensitizing, the GnRH receptors, it indirectly down regulates the secretion of gonadotropins luteinizing hormone (LH) and follicle-stimulating hormone (FSH), leading to hypogonadism and thus a dramatic reduction in estradiol and testosterone levels in both sexes.

Doxylamine is an ethanolamine H1 antagonist drug previously utilized as a possible treatment of nausea and vomiting of pregnancy. Now, this drug is still available over-the-counter as sleep aid. What B-vitamin is a component of Doxylamine? A. Vitamin B2 B. Vitamin B3 C. Vitamin B6 D. Vitamin B12

681 C. Answer: C. Vitamin B6 (pp. 389 [.pdf file], Katzung Pharmacology, 9th edition) Notes: Several H1 antagonist drugs have been studied for possible use in treating "morning sickness." The piperazine derivatives were withdrawn from such use when it was demonstrated that they have teratogenic effects in rodents. Doxylamine, an ethanolamine H1 antagonist, was promoted for this application as a component of Bendectin, a prescription medication that also contained pyridoxine. * SIMILAR TO PREVIOUS BOARD EXAM CONCEPT/PRINCIPLE

A few substances that are almost completely inert in the chemical sense nevertheless have significant pharmacologic effects. An example of these substances is Simethicone, which is included in many antacid preparations in order to act as A. Antispasmodic B. Antiflatulent C. Antiemetic D. Antiminth

682 B. Answer: B. Antiflatulent (pp. 1520 [.pdf file], Katzung Pharmacology, 9th edition) Notes: Some antacid preparations include simethicone, an antiflatulent to relieve symptoms of bloating and pressure. *SIMILAR TO PREVIOUS BOARD EXAM CONCEPT/PRINCIPLE

3. Future clinicians may screen every patient for a variety of individual differences in drug response. Differences in the rate of acetylation of isoniazid among groups of individuals exemplifies A. Pharmacodynamics B. Posology C. Pharmacogenetics D. Biochemorphology

683 C. Answer: C. PharmaCogenetiCs (pp. 3 and 82 [.pdf file] Katzung Pharmacology, 9th edition)

A college friend consults you regarding the suitability of the medication given by his doctor for severe hypertension. He complains of postural and exercise hypotension (“dizzinessâ€), some diarrhea, and problems with ejaculation during sexual activity. Given those adverse effects stated, which of the following mechanisms of action is most likely related to the drug described by your friend? A. A drug which inhibits the release of norepinephrine from sympathetic nerve endings. B. A drug which decreases blood pressure as a result of a decrease in cardiac output due to its nonselective β-blockade. C. An oral vasodilator which relaxes smooth muscles of arterioles, thereby decreasing the systemic vascular resistance. D. A drug which inhibits the converting enzyme peptidyl dipeptidase that hydrolyzes angiotensin I to angiotensin II and also inactivates bradykinin.

684 A. Answer: A. A drug whiCh inhibits the release of norepinephrine from sympathetiC nerve endings. The adverse effects described pertains to Guanethidine (pp. 237, 241, 244 and 251 [.pdf file], Katzung Pharmacology, 9th edition). Notes: • A drug which decreases blood pressure as a result of a decrease in cardiac output due to its nonselective β-blockade â€" Propranolol • An oral vasodilator which relaxes smooth muscles of arterioles, thereby decreasing the systemic vascular resistance â€" Hydralazine • A drug which inhibits the converting enzyme peptidyl dipeptidase that hydrolyzes angiotensin I to angiotensin II and also inactivates bradykinin â€" Captopril

The Board of Medicine decided to change the program of examination for the August 2013 Physician Licensure Examination (PLE) changing the first subject to Pharmacology. Because of this, a non-prepared Non-Topnotch examinee suddenly got an attack of diarrhea while on his way to the testing site. If he will stop at a nearby drugstore to buy an over-the-counter opioid with an antidiarrheal action, he will be asking for A. Codeine B. Dextromethorphan C. Loperamide D. Diphenoxylate

685 C. Answer: C. Loperamide (pp. 713 [.pdf file], Katzung Pharmacology, 9th edition) â€" must know, MPL 1.0

Sources of drugs can either be natural or synthetic. These might be derived from plant sources, animal or marine products, minerals, microorganisms and others from laboratory sources. Many antibiotics were derived from Genus Streptomyces, Penicillium, Micromonospora, etc. Which of the following drugs is a polyene antibiotic derived from Streptomyces nodosus? A. Netilmicin B. Amikacin C. Streptomycin D. Amphotericin B

686 D. Answer: D. AmphoteriCin B (pp. 1104 [.pdf file], Katzung Pharmacology, 9th edition.) Notes: incomplete list Streptomycin à Strept. griseus Neomycin à Strept. fradiae Paromomycin à Strept. rimosus Kanamycin (der. Amikacin) à Strept.kanamyceticus Tobramycin à Strept. tenebrarius Ivermectin à Strept. avermitilis Mitomycin à Strept. caespitosus Erythromycin à Strept. erythreus Lincomycin (der. Clindamycin) à Strept. lincolnensis Rifamycin (der. Rifampicin) à Strept. mediterranei Gentamicin à Micromonospora purpurea Sisomicin (der. Netilmicin) à Micromonospora inyoensis

Mrs. T. Anga, 25 yo, G1P1 was misdiagnosed to have preeclampsia because of the inaccuracy of her history and was given an anti-hypertensive to control her blood pressure. Fortunately, patient had no postpartum complications and BP was controlled. She continued taking the previously given antihypertensive but was lost to follow-up. After maintaining the drug for more than 12 months, patient developed intermittent episodes of headache, myalgias, jaundice and passage of dark brown urine. She finally went back to her physician for consult and laboratory testing revealed a positive Antiglobulin Test. If the condition was drug-induced, what is the most likely offending agent? A. Methyldopa B. Hydralazine C. Clonidine D. Magnesium Sulfate

687 A. Answer: A. Methyldopa (pp. 235 [.pdf file], Katzung Pharmacology, 9th edition). Notes: The symptoms experienced by the patient are consistent with Autoimmune Hemolytic Anemia which is a rare adverse effect of methyldopa. Methyldopa is a drug which lowers blood pressure chiefly by reducing the peripheral vascular resistance and variably reduces heart rate and cardiac output. Ten to twenty percent of patients undergoing therapy for longer than 12 months from methyldopa will have positive Coomb’s Test (or Antiglobulin Test).

Baby B. Ulate, suffers from infestation of the “unholy trinity†of roundworms, namely Ascaris lumbricoides, Trichuris trichiura and Ancylostoma duodenale. What is the mechanism of action of the drug suitable for the patient’s condition? A. Increases permeability to calcium causing muscle paralysis, vacuolization and death. B. Selectively inhibits microtubule synthesis and glucose uptake in nematodes. C. Immobilizes the parasite and alters their surface structure displacing them from the tissues and making them more susceptible to destruction by host defense mechanisms. D. Intensifies GABA-mediated neurotransmission in nematodes immobilizing the parasite.

688 B. Answer: B. SeleCtively inhibits miCrotubule synthesis and gluCose uptake in nematodes is the meChanism of aCtion of Mebendazole, the drug of ChoiCe for AsCariasis (pp. 40 of part2, Pharmacology, Topnotch Handouts). Notes: • Increases permeability to calcium causing muscle paralysis, vacuolization and death â€" Praziquantel (DOC for Trematodes and Cestodes) • Immobilizes the parasite and alters their surface structure displacing them and making them more susceptible to destruction by host defense mechanisms â€" Diethylcarbamazine (DOC for Filariasis) • Intensifies GABA-mediated neurotransmission in nematodes immobilizing the parasite â€" Ivermectin (DOC for Strongyloidiasis) • “Unholy trinity†of roundworms â€" Ascaris, Whipworm and Hookworms

The following drugs are included in the ABVD regimen for Hodgkin’s Lymphoma, except: A. Vinblastine B. Doxorubicin C. Dacarbazine D. Oncovin E. Bleomycin

689 A. Answer: A. Vinblastine (pp 41 [part 1], Pharmacology, Topnotch Handouts). Notes: A â€" Adriamycin (Doxorubicin) B â€" Bleomycin V â€" Vincristine (Oncovin) D â€" Dacarbazine

This beta blocker has a unique mechanism of action owing to its it has a nitric oxide (NO)-potentiating vasodilatory effect in addition to beta blockade. A. Carvedilol B. Pindolol C. Nadolol D. Esmolol E. Nebivolol

69 E Nebivolol is a beta1 receptor blocker with nitric oxide-potentiating vasodilatory effect used in treatment of hypertension

The following drugs accelerate substrate metabolism causing a decrease in the pharmacologic action of the co-administered drug, except; A. Griseofulvin B. Rifampicin C. Ethanol D. Cimetidine

690 D. Answer: D. Cimetidine (pp. 8 [part 1], Pharmacology, Topnotch Handouts; pp. 77 [.pdf file], Katzung Pharmacology, 9th edition). The question defines a CYP450 Inducer. The only CYP450 inhibitor among the choices is Cimetidine.

Mr. A. Lipin, a farmer from a distant hacienda was brought to the ED and admitted due to unconsciousness, profuse salivation and shallow breathing. It took 7 hours of travel time from the hacienda to the nearest Provincial Hospital. BP 140/90 mmHg, PR 52 bpm, temp 30OC and with constricted pupils. As an ED physician, what medication will you initially give that will most likely benefit the patient? A. Pralidoxime B. Atropine C. Pralidoxime + Atropine D. Physostigmine

691 B. Answer: B. Atropine (pp. 141, 149 and 162, Katzung Pharmacology, 9th edition) Notes: Atropine is the prototype antimuscarinic drug to be given initially in this patient suffering from organophosphate poisoning. It is best to give both Atropine and Pralidoxime if the golden period of 6 hours was not violated. The likelihood of aging of the phosphate enzyme complex occurs after 6 hours; however, recent reports suggest that administration of multiple doses of pralidoxime over several days may be useful in severe poisoning.

A patient in PACU is in no apparent distress. The V/S are stable except for HR of 128 bpm that is irregular with no p wave. Which of the following treatment option would not be appropriate as initial therapy? A. Metoprolol B. Diltiazem C. Digoxin D. Adenosine

692 D. Answer: D Patient has atrial fibrillation w/ rapid ventricular response. B-blocker, Ca Channel blocker or Digoxin may be used. Supraventricular tachycardia, not a.fib responds to adenosine.

This factor indicates the spread of local anesthetic in CSF? A. Addition of narcotic to local anesthetic. B. Density of local anesthetic C. Patient’s body surface area D. Dose administered

693 B. Answer: B Density of local anesthetic in relation to density of CSF at normal body temp.is termed as baricity. Degree of spread is determnined primarily by density of anesthetic & patient’s position.

A young, male patient presents with mental deterioration and tremors. He has brown pigmentation in a ring surrounding the periphery of his cornea and altered liver function tests. Which treatment modality should he receive? A. Dimercaprol B. Penicillamine C. EDTA D. Deferoxamine

694 B. Answer: B Wilson’s disease: excess copper deposition (cornea = Kayser-Fleischer ring)

In the treatment of CHF, this drug is characterized as having a selective beta-1 antagonistic activity and devoid of dopaminergic activity: A. Fenoldopam B. Dobutamine C. Dopamine D. Isoproterenol

695 B. Answer: B Dobutamine is a relatively b-1 selective adrenoreceptor agonist that causes peripheral vasodilation in some vascular beds useful for many cases of acute heart failure.

A 7 year-old male child with nephritic syndrome on steroid therapy was exposed to a sister with measles. You would administer: A. Measles vaccine B. Human immunoglobulin C. Antisera D. Vitamin A

696 B. Answer: B After susceptible person is exposed to measles, either measles vaccine given within 72 hrs or immune globulin given within 6 days can prevent or modify disease. However, px has nephritic syndrome & is immunocompromised so an active measles vaccine may even be harmful.

At the E.R., you saw a 24-yr old woman complaining of severe, acute, generalized headache. PE revealed T 400C with (+) nuchal rigidity. Her CSF showed gram-neg diplococci & 200 leukocytes/mm3 (98% PMNs). Administration of which of these is the next important management: A. IV sulphonamides B. Intrathecal penicillin C. IV penicillin D. Intratechal chloramphenicol

697 C. Answer: C Lab picture: N.meningitides. Tx = Pen G 4 million units IV q4h

An asthmatic patient whose symptoms occur less than once a week, lasts a few hours to a few days and has nighttime symptoms twice a month is best managed with an: A. Inhaled long-acting B2 agonist at H.S. B. Inlahed short-acting B2 agonist PRN C. Inhaled steroid OD D. Oral B2 agonist TID

698 B. Answer: B Px’s asthma is classified as intermittent. *Review GINA guidelines

The absorption of most orally administered drug, which occur in the small intestine, may be enhanced by A. Taking agents that accelerate gastric emptying time B. The presence of food C. Stimulation of hepatic microsomal CYP3A4 enzymes D. Increasing intestinal motility

699 A. Answer: A Increasing gastric emptying time allow drugs to reach the small intestine sooner, hence increasing the rate of drug absorption.

Which of the following has the most potent mineralocorticosteroid activity? A. Hydrocortisone B. Triamcinolone C. Betamethasone D. Fludrocortisone E. Dexamethasone

7 D. Contraindication to thrombolysis - History of CVS hemorrhage at ANY time - History of CVS infarct within the past year - Marked hypertension (>180/110) - Suspicion of aortic dissection - Active internal bleeding (excluding menstruation) Source: Topnotch handout on Pharmacology

What is the anti-hypertensive drug of choice for patients with diabetic nephropathy? A. Nifedipine B. Clonidine C. Enalapril D. Atenolol E. Thiazides

70 C. ACE inhibitors are renal efferent arteriole dilators decreasing albumin excretion and slowing progression of CKD.

Adverse events associated with the use of quinolones include all of the following except: A. Tendinitis and possible tendon rupture B. Seizures C. Arthropathy in children D. Clostridium difficile colitis E. Narrowing of QT interval

700 E Answer: E Quinolones inhibit topoisomerase II (DNA gyrase) & IV impairing DNA synthesis. Adverse effects include GI intolerance, antibiotic-asso.colitis, cutaneous rxns, hepatotoxicity, prolongation of QT interval and Achilles tendon rupture.

Which of the ff electrolyte abnormality will increase digitalis toxicity? A. Hyponatremia B. Hypokalemia C. Hypocalcemia D. Hypermagnesemia E. Hyperphosphatemia

71 B. Digitalis toxicity is increased in patients with hypokalemia, hypomagnesemia and hypercalcemia. Potassium and digitalis interact by inhibiting each other's binding to Na+/K+-ATPase; therefore, hyperkalemia reduces the enzyme-inhibiting actions of cardiac glycosides, whereas hypokalemia facilitates these actions.

What is the drug of choice for the rapid termination of paroxysmal supraventricular tachycardia? A. Adenosine B. Magnesium C. Lidocaine D. Amiodarone E. Verapamil

72 A. Adenosine is the drug of choice for paroxysmal SVT. Magnesium is the drug of choice for Torsades de pointes. Lidocaine is the drug of choice for post MI arrythmias and digoxin induced arrythmias.

A child was brought to you with severe itching due to multiple insect bites. The mother requested for a non sedating anti histaminic agent. What drug can you give? A. Chlorpheniramine B. Montelukast C. Meclizine D. Fexofenadine E. Hydroxyzine

73 D. Among the choices only fexofenadine is a 2nd generation antihistamine. Montelukast is a leukotriene receptor antagonist. All the others are 1st gen antihistamines

A 15 year old post partum girl in Fabella hospital was bleeding profusely after giving birth despite oxytocin administration, what class of drug can you give as second line treatment? A. 5-HT1 agonist B. 5-HT2 antagonist C. 5-HT3 antagonist D. 5-HT4 antagonist E. Alpha 1 agonist

74 B. Methylergonovine can be given which is a 5-HT2 receptor antagonist selective to the uterus.

This drug class used in asthma exerts its effect by contraction of engorged vessels in the bronchial mucosa and reduces bronchial hyperreactivity? A. Beta 2 agonists B. Inhaled corticosteroids C. Methylxanthines D. Mast cell stabilizers E. Antimuscarinic agent

75 B. Corticosteroids do not relax airway smooth muscle directly but reduce bronchial reactivity and reduce the frequency of asthma exacerbations if taken regularly. Their effect on airway obstruction may be due in part to their contraction of engorged vessels in the bronchial mucosa and their potentiation of the effects of beta-receptor agonists, but their most important action is inhibition of the infiltration of asthmatic airways by lymphocytes, eosinophils, and mast cells.

Which of the ff anticonvulsant agents do not exerts its effect through Na channel blockade? A. Phenytoin B. Carbamazepine C. Topiramate D. Valproic acid E. Ethosuximide

76 E Ethosuximide blocks the T type calcium current in the thalamus and is the drug of choice for absence seizures. All the other drugs block Na channels.

Which of the inhaled anesthetics forms a toxic metabolite, Compound A, that is nephrotoxic at high doses? A. Halothane B. Isoflurane C. Enflurane D. Sevoflurane E. Desflurane

77 D. Well known side effects: Halothane - Hepatitis Isoflurane - Coronary steal syndrome Enflurane - Seizures Sevoflurane - Renal insufficiency Desflurane - Pulmonary irritant

MRSA, a multi drug resistant organism is kept at bay through administration of vancomycin. Currently, reports of MRSA resistant to vancomycin (VRSA) are being reported. What is the mechanism of resistance to vancomycin? A. Structural change in target PBP B. Formation of extended spectrum beta lactamases C. D ala-ala is modified to D ala-lactate D. Changes in porin structure of outer cell wall E. Extrusion of the drug through P-glycoprotein efflux pump

78 C. Structural change in target PBP is mechanism of resistance of MRSA not VRSA. Resistance to vancomycin in vancomycin-resistant S aureus strains is due to modification of the D-Ala-D-Ala binding site of the peptidoglycan building block in which the terminal D-Ala is replaced by D-lactate. This results in the loss of a critical hydrogen bond that facilitates high- affinity binding of vancomycin to its target and loss of activity.

This antidiabetic drug because of its unique mechanism of action should be taken just before ingesting the first portion of each meal? A. Exenatide B. Miglitol C. Pramlintide D. Pioglitazone E. Metformin

79 B. Acarbose and miglitol are competitive inhibitors of the intestinal alpha-glucosidases and reduce postmeal glucose excursions by delaying the digestion and absorption of starch and disaccharide. Both acarbose and miglitol are taken in doses of 25-100 mg just before ingesting the first portion of each meal

Which of the following is not true regarding Noncompetitive antagonism? A. No effect on Emax B. No effect on ED50 C. No effect on potency D. A and B E. All of the above

8 A. Competitive antagonism: ED50 increases, potency decreases, No effect of maximum efficacy Noncompetitive antagonism: No effect ED50, No effect on potency, decreased Emax Source: Topnotch handout on Pharmacology

What anti-TB medication readily penetrates into phagocytic cells and can kill organisms that are poorly accessible to many other drugs, such as intracellular organisms and those sequestered in abscesses and lung cavities? A. Isoniazid B. Rifampicin C. Ethambutol D. Pyrazinamide E. Streptomycin

80 B. As stated verbatim in Katzung, rifampicin readily penetrates most tissues and penetrates into phagocytic cells. It can kill organisms that are poorly accessible to many other drugs, such as intracellular organisms and those sequestered in abscesses and lung cavities.

Which of the following medications is NOT contraindicated in a patient taking RANOLAZINE? A. Losartan B. Itraconazole C. Clarithromycin D. Metronidazole E. Prednisone

81 A. Medscape Expect questions on never before heard drugs.

Phoebe Marie, G2 P1 (1001), recently traveled to Palawan and is taking Quinine for malaria. Which of the following medications can she take? A. Clindamycin B. Doxycycline C. Primaquine D. Halofantrine E. All of the above

82 A. Halofantrine is not used for chemoprohylaxis because of its potential for quinidine like cardiotoxicity (QT prolongation) and embryotoxicity. Likewise, doxycycline and primaquine are contraindicated during pregnancy. Katzung Pharmacology Examination and Board Review 10th edition p.453

Which of the following drugs is a reversible acetylcholinesterase inhibitor that causes an increase in concentrations of acetylcholine and is metabolized by cholinesterase and excreted mainly by the kidneys and used in Alzheimer's disease? A. Memantine B. Galantamine C. Rivastigmine D. Donepezil E. none of the above

83 C. Memantine - low to moderate affinity uncompetitive NMDA receptor antagonist blocking receptor only under conditions of excessive stimulation without effect of normal neurotransmission Donepezil - acetylcholinesterase inhibitor but metabolized by hepatic P - 450 enzymes Galantamine - increases acetylcholine from surviving presynaptic nerve terminals by modulating nicotinic Ach receptor Medscape

Which of the following is the mechanism why Cefuroxime is not used for meningitis? A. poor penetration to CSF B. levels achieved in CSF not adequate to sterilize it C. potential for delayed sterilization and therapeutic failures D. presence of efflux pumps E. none of the above

84 C. http://pedsinreview.aappublications.org/content/2 9/8/264.full

Which of the following is the drug topically used for acne? A. allylamine B. clindamycin C. mupirocin D. bacitracin E. none of the above

85 B. Goodman Gilman 12th edition. Dermatology section

Which of the following anti folate drugs is a blood schizonticide? A. Atovaquone B. Artesunate C. Halofantrine D. Doxycycline E. Pyrimethamine

86E Katzung Pharmacology Examination and Board Review 10th edition p.453

Which of the following is a long acting insulin? A. Lantus B. Lente C. Glargine D. Detemir E. Glulisine

87 C. D and E - short acting A and B - intermediate acting Katzung Pharmacology Examination and Board Review 10th edition p.363

Which of the following is NOT true of Gentamicin? A. killing action continues when plasma levels decline below measurable levels B. lesser efficacy when administered as a single large dose C. in vivo efficacy not directly related to time above MIC D. as plasma level is increased above MIC, it kills an increasing proportion of bacteria at a more rapid rate E. toxicity depends on both critical plasma concentration and the time that such level is exceeded

88 B. Has greater efficacy when administered as a single large dose than when given as multiple smaller doses Katzung Pharmacology Examination and Board Review 10th edition p.399

Which of the following chemotherapeutic drugs is used for bladder cancer (intravesical administration) and control of malignant pericardial/ pleural/ peritoneal effusions (intracavitary)? A. Cyclophosphamide B. Thiotepa C. Carmustine D. Lomustine E. None of the above

89 B. Medscape Expect questions on never before heard drugs.

Which of the following is incorrectly matched? A. Menotropin - FSH and LH analog B. Ocreotide - Somatostatin analog C. Desmopressin - ADH analog D. Ganirelix - GnRH analog E. Levothyroxine - T4

9. Bonus. No correct answer Menotropin - FSH and LH analog Ocreotide - Somatostatin analog Desmopressin - ADH analog Ganirelix - GnRH antagonist Levothyroxine - T4 Source: Topnotch handout on Pharmacology

Which of the following is a mixed FSH and LH agonist and is used to stimulate gonadal function including spermatogenesis and ovulation? A. Menotropin B. Urofollitropin C. Lutropin D. Somatropin E. Gonadorelin

90 A. B - FSH only C - LH only D - GH agonist E - GnRH agonist Katzung Pharmacology Examination and Board Review 10th edition p.327

Which of the following is not likely used for treatment of Herpes Simplex virus? A. Acyclovir B. Penciclovir C. Famciclovir D. Ganciclovir E. none of the above

91 D. D is used for CMV Katzung Pharmacology Examination and Board Review 10th edition p.429

A neonate in the NICU was observed by the Neonatologist fellow to have epicanthal folds, low nasal bridge, short palpebral fissures, flat midface, and indistinct philtrum. Which of the following medications did the mother likely take during pregnancy? A. Phenytoin B. Carbamazepine C. Valproic acid D. Ethyl alcohol E. Warfarin

92 D.

Which of the following is the chemoprophylactic drug of choice for pregnant patients visiting falciparum malaria endemic places? A. chloroquine B. mefloquine C. doxycycline D. atovaquone - proguanil E. artemether - lumefantrine

93 B. http://www.who.int/ith/ITH_chapter_7.pdf

Which of the following is NOT true of phenytoin? A. Metabolism is nonlinear B. Drug binds extensively to proteins in plasma C. Fosphenytoin in the water soluble prodrug for parenteral administration D. Metabolism is induced by isoniazid E. Drug of choice for grand mal seizures

94 D. metabolism is enhanced by presence of inducers of liver metabolism (phenobarbital, rifampin) and inhibited by other drugs (cimetidine, isoniazid) Katzung Pharmacology Examination and Board Review 10th edition p.217

A 54 year old male presents to the clinic with a circular lesion on the arm and anesthesia on pin prick of the center of the lesion. Patient was then brought to Cebu Skin Clinic for further evaluation. Dr. AM impression was leprosy tuberculoid type and decides to start the patient with antimycobacterials. Which of the following is true of Dapsone? A. Fluorodapsone is the repository form B. It is used as an alternative drug for treatment of PCP pneumonia C. It is not the most active drug for Leprosy D. Mechanism of action is to inhibit arabinosyl transferase E. Can be given to G6PD patients

95 B. Acedapsone is the repository form that provides inhibitory plasma concentrations for several months. It is the most active drug for leprosy. Mechanism of action is inhibition of folic acid synthesis. It is contraindicated to G6PD patients. Katzung Pharmacology Examination and Board Review 10th edition p.415

DC, a 69 year old female, presented with resting tremors and shuffling gait. Upon cleaning her bathroom, she accidentally damaged the bathroom sink thus prompting consult. Having a good understanding of the pathophysiology of Parkinson's disease, Dr. MC starts the patient with dopamine agonists. Which of the following can be given as monotherapy for mild parkinsonism and has high affinity for D3 receptor? A. Bromocriptine B. Pramipexole C. Ropinirole D. Apomorphine E. Cabergoline

96 B. Katzung Pharmacology Examination and Board Review 10th edition p.249

Which of the following is the regimen for a 29 year old male with a testicular mass? A. ABVD regimen B. paclitaxel + carboplatin C. gemcitabine + erlotinib D. fluorouracil and leucovorin + oxaliplatin E. PEB regimen

97 E A - hodgkin's lymphoma B - ovarian C - pancreatic D - colorectal Katzung Pharmacology Examination and Board Review 10th edition p.468

NT a 39 year old male with adult polycystic kidney disease underwent kidney transplant. Which of the following immunophilin inhibitors bind to FK - binding protein 12 and inhibit the mTOR pathway which inhibits the T - cell proliferation response to IL - 2? A. sirolimus B. tacrolimus C. cyclosporin D. cyclophiline E. none of the above

98 A. Cyclosporine binds to cyclophilin and tacrolimus binds to FKBP. Both complexes inhibit calcineurin, a cytoplasmic phosphatase. Calcineurin regulates the ability of the nuclear factor of activated T cells to translocate to the nucleus and increase the production of key cytokines such as IL 2, 3, and IFN gamma. Cyclophilin and tacrolimus prevent the increased production of cytokines that normally occurs in response to T cell receptor activation. Katzung Pharmacology Examination and Board Review 10th edition p.482

A 23 year old female came in for unilateral throbbing headache associated with nausea and photophobia. Which of the following ergot alkaloids has no effects on the dopamine receptor D2 but stimulates the uterine smooth muscle? A. bromocriptine B. ergonovine C. ergotamine D. LSD E. methylergometrine

99 C. Ergot alkaloids can produce marked and prolonged alpha receptor mediated vasoconstriction. Ergotamine is the prototype and has been a mainstay of treatment of acute attacks of migraine. Katzung Pharmacology Examination and Board Review 10th edition p.160


Related study sets

The General Theory of Employment, Interest & Money

View Set

MHS Unit 2 Body and Behavior Review

View Set

115 PrepU Ch.17: Assessment of Respiratory Function

View Set

2010 AP Environmental Science Test

View Set

CH 7 International Arbitrage and Interest Rate Parity

View Set

Chapter 7 - Language of Medicine Online Questions & Answers & TB

View Set

Unit 3: Financial Statements: Module 3: Financial Statements Overview, Lesson 9: The Income Statement, Lesson: 10: The Statement of Retained Earnings, Lesson 11: The Statement of Cash Flows, Lesson: 12: The Statement of Cash Flows, Module 3 Quiz

View Set